CAT Previous Year Paper 2009

CAT 2009

Section

Quantitative Aptitude

Questions

25 Questions

Marks

75

DILR

15 Questions (21 – 35)

45

English

20 Questions (41 – 60)

60

Q. 1 If mxᵐ-nxⁿ = 0 , then what is the value of 1/(xᵐ+xⁿ) + 1/(xᵐ+xⁿ) in terms of xⁿ?

A. 2mn/xⁿ(n²-m²)

B. 2mn/xⁿ(n² + m²)

C. 2mn/xⁿ(m² – n²)

D. 2mn/xⁿ(m² + n²)

 

Q. 2 If log (0.57) = 1(bar).756 , then the value of

log 57 + log (0.57)³ + log √0.57 is

A. 0.902

B. 2(bar).146

C. 1.902

D. 1(bar).146

 

Q. 3 In a certain zoo, there are 42 animals in one sector, 34 in the second sector and 20 in the third sector. Out of this, 24 graze in sector one and also in sector two. 10 graze in sector two and sector three, 12 graze in sector one and sector three. These figures also include four animals grazing in all the three sectors are now transported to another zoo, find the total number of animals.

A. 38

B. 56

C. 54

D. None of the above

 

Q. 4 The ratio of the roots of bx² + nx + n = 0 is p :q , then

A. √(q/p) + √(p/q) + √(l/n) = 0

B. √(p/q) + √(q/p) + √(n/l) = 0

C. √(q/p) + √(p/q) + √(n/l) = 0

D. √(p/q) + √(q/p) + √(l/n) = 0

 

Q. 5 The average age of a couple is 25 years. The average age of the family just after the birth of the first child was 18 years. The average age of the family just after the second child was born was 15 years. The average age of the family after the third and the fourth children (who are twins) were born was 12 years. If the present average age of the family of six persons is 16 years, how old is the eldest child?

A. 6 years

B. 7 years

C. 8 years

D. 9 years

 

Q. 6 10% of the voters did not cast their vote in an election between two candidates. 10% of the votes polled were found invalid. The successful candidate got 54% of the valid votes and won by a majority of 1620 votes. The number of voters enrolled on the voters list was:

A. 25000

B. 33000

C. 35000

D. 40000

 

Q. 7 The resistance of a wire is proportional to its length and inversely proportional to the square of its radius. Two wires of the same material have the same resistance and their radii are in the ratio 9:8. If the length of the first wire is 162 cms, find the length of the other.

A. 64 cm

B. 120 cm

C. 128 cm

D. 132 cm

 

Q. 8 A 20 litre vessel is filled with alcohol. Some of the alcohol is poured out into another vessel of an equal capacity, which is then completely filled by adding water. The mixture thus obtained is then poured into the first vessel to capacity. Then 623 litres is poured from the first vessel into the second. Both vessels now contain an equal amount of alcohol. How much alcohol was originally poured from the first vessel into the second?

A. 9 litres

B. 10 litres

C. 12 litres

D. 12.5 litres

 

Q. 9 The shaded portion of figure shows the graph of which of the following ?

A. x (y – 2x) ≥ 0

B. x (y – 2x) ≤ 0

C. x (y + 1/2x) ≥ 0

D. x (y – 1/2x) ≤ 0

 

Q. 10 If f (x + y/8 , x – y/8) = xy , then f(m , n) + f(n , m) = 0

A. Only when m = n

B. Only when m ≠ n

C. Only when m = −n

D. For all m and n

 

Q. 11 A person closes his account in an investment scheme by withdrawing Rs. 10000. One year ago he had withdrawn Rs. 6000. Two years ago he had withdrawn Rs. 5000. Three years ago he had not withdrawn any money. How much money he had deposited approximately at the time of opening of the account 4 years ago , if the annual simple interest is 10% ?

A. Rs. 15600

B. Rs. 16500

C. Rs. 17280

D. None of these

 

Q. 12 It takes 6 technicians a total of 10 hours to build a new server from direct computer , with each working at the same rate. If six technicians start to build the server at 11:00 am , and one technician per hour is added beginning at 5:00 pm , at what time will the server be completed ?

A. 6:40 pm

B. 7:00 pm

C. 7:20 pm

D. 8:00 pm

 

Q. 13 A ship 55 kms from the shore springs a leak which admits 2 tonnes of water in 6 min ; 80 tonnes of water would suffer to sink her , but the pumps can throw out 12 tonnes an hour. Find the average rate of sailing that she may just reach the shore as she begins to sink.

A. 5.5 km/hr

B. 6.5 km/hr

C. 7.5 km/hr

D. 8.5 km/hr

 

Q. 14 In a 400 meter race around a circular stadium having a circumference of 1000 meters , the fastest runner and the slowest runner reach the same point at the end of the 5th minute , for the first time after the start of the race. All the runners have the same starting point and each runner maintains a uniform speed throughout the race. If the fastest runner runs at twice the speed of the slowest runner , what is the time taken by the fastest runner to finish the race ?

A. 20 mins

B. 15 mins

C. 10 mins

D. 5 mins

 

Q. 15 A train crosses a platform 100 meters long in 60 seconds at a speed of 45 km/hr. The time taken by the train to cross an electric pole is

A. 8 sec

B. 1 min

C. 52 sec

D. data inadequate

 

Q. 16 If x = 1 + 2a + 3a² + 4a² +………….. (−1 < a < 1) and

y = 1 + 3b + 6b² + 10b³ +………….. (−1 < b < 1) ,

then find 1 + ab + (ab)² + (ab)³ +………………… in terms of x and y

A. x½ y⅓ / x½ + y⅓ − 1

B. xy / x + y − 1

C. x⅓ y⅔ / x⅓ + y½ − 1

D. None of the above

 

Q. 17 Two vertical lamp-posts of equal heights stand on either side of a road 50 m wide. At a point P on the road between them , the elevation of the tops of the lamp-post are 60°and 30°. Find the distance of P from the lamp post which makes angle of 60°.

A. 25 m

B. 12.5 m

C. 16.5 m

D. 20.5 m

 

Q. 18 M is the centre of the circle. l(QS) = 10√2 ; l(RS) = l(RS) and PR ∥ QS. Find the area of the shaded region. (use π = 3)

A. 100 sq. units

B. 114 sq. units

C. 50 sq. units

D. 200 sq. units

 

Q. 19 There are three coplanar parallel lines. If any p points are taken on each of the lines , then find the maximum number of triangles with the vertices of these points.

A. p² (4p − 3)

B. p³ (4p − 3)

C. p (4p − 3)

D. p³

 

Q. 20 A and B throw with one dice for a stake of Rs. 11 which is to be won by the player who first throws 6. If A has the first throw , what are their respective expectations.

A. Rs.7 , Rs. 4

B. Rs. 6 , Rs. 5

C. Rs. 4 , Rs. 7

D. Rs. 5 , Rs. 6

 

Questions: 21 – 25

DIRECTIONS (Q 21 – 25) : These are based on the following Line Chart : The sales and net profit of XPL Electronics in Rs. crores is given below.

 

Q. 21 What is the net profit percentage of XPL in 1998 ?

A. 4.6%

B. 4.8%

C. 5.1%

D. 6.2%

 

Q. 22 Which year showed the maximum percentage increase in sales ?

A. 1999

B. 1998

C. 1997

D. 1996

 

Q. 23 By how much percentage has the net profit dropped in 1996 ?

A. 1.1%

B. 2.27%

C. 2.53%

D. cannot be determined

 

Q. 24 If XPL sold 20,000 units in both 1998 and 1999 , by what percentage has the price per unit changed ?

A. 8.7%

B. 10.96%

C. 9.86%

D. cannot be determined

 

Q. 25 The year in which the expenses of XPL Electronic are highest is

A. 1999

B. 1998

C. 1997

D. cannot be determined

 

Questions: 26 – 30

DIRECTIONS (Q 26 – 30) : These are based on the following data.

The following chart gives the production of three major crops (in million tons) across five Indian states in the year 1998.

 

Q. 26 If Tamil Nadu registers an annual increase of 22% in rice production , what was the rice production in Tamil Nadu in 1998 ?

A. 1900 million tons

B. 2300 million tons

C. 2180 million tons

D. 2520 million tons

 

Q. 27 What is the ratio of total wheat production in the five states states to total sugar production ?

A. 0.6

B. 1.4

C. 1.0

D. 0.75

 

Q. 28 If sugar costs Rs. 700 per ton and wheat costs Rs. 400 ton , what was the total worth of sugar and wheat production in India ?

A. 42 million

B. 21 million

C. 10.5 million

D. data sufficient

 

Q. 29 If the yield per hectare of sugar is 3.86 tons , what is the ratio of area employed to produce sugar in Bihar to Gujarat ?

A. 1.15

B. 1.45

C. 1.35

D. data insufficient

 

Q. 30 If these five states constitute 77% of the country’s wheat production and 23% of country’s wheat consumption is imported , what is the amount of wheat imported in 1998 ?

A. 4700 million tons

B. 3900 million tons

C. 3500 million tons

D. data insufficient

 

Questions: 31 – 35

Directions (Q 31 – 35) : Read the following information ans answer the questions that follow.

The cars at a dealership come with a choice of the following options : air conditioning , a cassette deck , leather seats , power windows , a sunroof and tinted glass. None of the cars has any other optional equipment. The following conditions apply :

If a car has leather seats , it also has a cassette deck. If a car has a cassette deck, it also has power windows. If a car has power windows , it also has a cassette deck. Cars with tinted glass have a sunroof , but no air-conditioning. Cars that have air conditioning , have at most , two other options.

 

Q. 31 If a car has both tinted glass and leather seats , what is the greatest number of additional options that the car could have ?

A. 1

B. 2

C. 3

D. 4

 

Q. 32 Which one of the following could be a complete and accurate list of options on a car ?

A. air conditioning , cassette deck , leather seats , power windows

B. air conditioning , cassette deck , leather seats , sunroof

C. cassette , leather seats , sunroof , tinted roof

D. cassette deck , power windows , sunroof , tinted glass

 

Q. 33 If a car has power windows and a sunroof, how many different sets of options, at most, can the car have ?

A. 2

B. 3

C. 4

D. 5

 

Q. 34 If a car has exactly two options , which one of the following could they be ?

A. air conditioning and cassette deck

B. tinted glass and sunroof

C. cassette deck and leather seats

D. power windows and sunroof

 

Q. 35 If a car has tinted glass , which one of the following CANNOT be true ?

A. The car has 3 options

B. The car has 4 options

C. The car has power windows and a sunroof

D. The car does not have both leather seats and cassette deck

 

Questions: 36 – 40

DIRECTIONS (Q 36 – 40) : Each of the following problems consists of a question and the statements labelled (1) and (2). You must decide whether the data given for the statements are sufficient to answer the question.

 

Q. 36 If both ABDC and CDFE are parallelograms , what is q + r ?

(1) r = 70

(2) p = 110

A. Statement (1) ALONE is sufficient , but statement (2) alone is not sufficient

B. Statement (2) ALONE is sufficient , but statement (1) alone is not sufficient

C. BOTH statements TOGETHER are sufficient , but NEITHER statement alone is sufficient

D. EACH statement ALONE is sufficient

 

Q. 37 A certain stadium is currently full to 13/16 of its maximum seating capacity. What is the maximum seating capacity of the stadium ?

(1) If 1250 people were to enter the stadium , the stadium would be full to 15/16 of its maximum seating capacity

(2) If 2500 people were to leave the stadium , the stadium would be full to 9/16 of its maximum seating capacity

A. Statement (1) ALONE is sufficient , but statement (2) alone is not sufficient

B. Statement (2) ALONE is sufficient , but statement (1) alone is not sufficient

C. BOTH statements TOGETHER are sufficient , but NEITHER statement alone is sufficient

D. EACH statement ALONE is sufficient

 

Q. 38 If n is an integer , is n even ?

(1) 2n is an even integer

(2) n -1 is an odd integer

A. Statement (1) ALONE is sufficient , but statement (2) alone is not sufficient

B. Statement (2) ALONE is sufficient , but statement (1) alone is not sufficient

C. BOTH statements TOGETHER are sufficient , but NEITHER statement alone is sufficient.

D. EACH statement ALONE is sufficient

 

Q. 39 What is the value of x ?

(1) x² − 5x + 4 = 0

(2) x is not prime

A. Statement (1) ALONE is sufficient , but statement (2) alone is not sufficient

B. Statement (2) ALONE is sufficient , but statement (1) alone is not sufficient

C. BOTH statements TOGETHER are sufficient , but NEITHER statement alone is sufficient

D. EACH statement ALONE is sufficient

 

Q. 40 A fish tank contains a number of fish , including 5 Fantails. If two fish are selected from the tank at random , what is the probability that both will be Fantails ?

(1) The probability that the first fish chosen will be Fantail is 1/2

(2) The probability that the second fish chosen will be a Fantail is 4/9

A. Statement (1) ALONE is sufficient , but statement (2) alone is not sufficient

B. Statement (2) ALONE is sufficient , but statement (1) alone is not sufficient

C. BOTH statements TOGETHER are sufficient , but NEITHER statement alone is sufficient

D. EACH statement ALONE is sufficient

 

Questions: 41 – 44

Directions (Q.41 to 44) : Read the following passage and give answer based on it. History has shaped academic medical centers (AMCs) to perform 3 functions:

patient care, research, and teaching. These 3 missions are now fraught with problems because the attempt to combine them has led to such inefficiencies as duplication of activities and personnel, inpatient procedures that could and should have been out-patient procedures, and unwieldy administrative bureaucracies.

One source of inefficiency derives from mixed lines of authority. Clinical chiefs and practitioners in AMCs are typically responsible to the hospital for practice issues but to the medical school for promotion, marketing, membership in a faculty practice plan, and educational accreditation. Community physicians with privileges at a university hospital add more complications. They have no official affiliationwith the AMC’s medical school connected, but their cooperation with faculty members is essential for proper patient treatment. The fragmented accountability is heightened by the fact that 3 different groups often vie for the loyalty of physicians who receive research. The medical school may wish to. capitalize on the research for its educational value to students; the hospital may desire the state-of-the-art treatment methods resulting from the research; and the grant administrators may, focus on the researchers’ humanitarian motives. Communication among these groups is rarely coordinated, and the physicians may serve whichever group promises the best perks and ignore the rest – which inevitably strains relationships.

Another source of inefficiency is the fact that physicians have obligations to many different groups: patients, students, faculty members, referring physicians, thirdparty payers, and staff members, all of whom have varied expectations. Satisfying the interests of one group may alienate others. Patient care provides a common example. For the benefit of medical students, physicians may order too many tests, prolong patient visits, or encourage experimental studies of a patient. If AMC faculty physicians were more aware of how much treatments of specific illnesses cost, and of how other institutions treat patient conditions, they would be better practitioners, and the educational and clinical care missions of AMCs would both be better served.

A bias toward specialization adds yet more inefficiency. AMCs are viewed as. institutions serving the gravest cases in need of the most advanced treatments. The high number of specialty residents and the presence of bum units, blood banks, and, transplant centers validate this belief. Also present at AMCs, though less conspicuous, are facilities for ordinary primary care patients. In fact, many patients choose to visit an AMC for primary care because they realize that any necessary follow-up can occur almost instantaneously. While AMCs have emphasized cutting-edge specialty medicine, their more routine medical services need development and enhancement. A final contribution to inefficiency is organizational complacency. Until recently, most academic medical centers drew the public merely by existing. The rising presence, however, of tertiary hospitals with patient care as their only goal has immersed AMCs in a very competitive market. It is only in the past several years that AMCs have started to recognize and develop strategies to address competition.

 

Q. 41 The author’s attitude toward the inefficiencies at academic medical centers is one of

A. reluctant acquiescence

B. strident opposition

C. agonized indecision

D. reasoned criticism

 

Q. 42 The author of the passage would most likely agree with which of the following statements about primary care at AMCs?

A. AMCs would make more money if they focused mainly on primary care.

B. Bum and transplant patients need specialty care more than primary care.

C. AMCs offer the best primary care for most patients.

D. Inefficiencies at AMCs would be reduced if better primary care were offered.

 

Q. 43 The author’s primary purpose in this passage is to

A. discuss the rise and fall of academic medical centers

B. explain that multiple lines of authority in a medical centre create inefficiencies

C. delineate conflicts occurring in academic medical facilities

D. examine the differences between-academic and other health care entities

 

Q. 44 The author implies which of the following about faculty physicians at AMCs?

A. Most of them lack good business sense.

B. They put patients’ physical health above their hospitals’ monetary concerns.

C. They sometimes focus on education at the expense of patient care.

D. They lack official affiliation with the medical schools connected to AMCs.

 

Questions: 45 – 47

Directions (Q.45 to 47) : Read the following passage and give answer based on it. Founded at the dawn of the modem industrial era, the nearly forgotten Women’s Trade Union League (WTUL) played an instrumental Line role in advancing the cause of working women throughout.the early part of the twentieth century. In the face of considerable adversity, the WTUL made a contribution far greater than did most historical footnotes. The organization’s successes did not come easily; conflict beset the WTUL in many forms. During those early days of American unions, organized labour was aggressively opposed by both industry and government. The WTUL, which represented a largely unskilled labour force, had little leverage against these powerful opponents. Also, because of the skill level of its workers as well as inherent societal gender bias, the WTUL had great difficulty finding allies among other unions. Even the large and powerful American Federation of Labour (AFL), which nominally took the WTUL under its wing, kept it at a distance. Because the AFL’S power stemmed from its highly skilled labour force, the organization saw little economic benefit in working with the WTUL. The – affiliation provided the AFL with political cover, allowing it to claim support for women workers; in return, the WTVL gained a potent but largely absent ally. The WTUL also had to overcome internal discord. While the majority of the group’s members were working women, a sizeable and powerful minority consisted of middle- and upperclass social reformers whose goals extended beyond labour reform. While workers argued that the WTUL should focus its efforts on collective bargaining and working conditions, the reformers looked beyond the workplace, seeking state and national legislation aimed at education reform and urban poverty relief as well as workplace issues. Despite these obstacles, the WTUL accomplished a great deal. The organization was instrumental in the passage of state laws mandating an eight-hour workday, a minimum wage for women, and a ban on child labour. It provided seed money to women who organized workers in specific plants and industries, and also established strike funds and soup kitchens to support striking unionists. After the tragic Triangle Shirtwaist Company fire of 1911, the WTUL launched a four-year investigation whose conclusions formed the basis of much subsequent workplace safety legislation. The organization also offered a political base for all reform-minded women, and thus helped develop the next generation of American leaders. Eleanor Roosevelt was one of many prominent figures to emerge from the WTUL. The organization began a slow death in the late I920s, when the Great Depression choked off its funding. The organization limped through the I940s; the death knell eventually rang in 1950, at the onset of the McCarthy era. A turn of-the-century labor organization dedicated to. social reform, one that during its heyday was regarded by many as “radical,” stood little chance of weathering that storm. This humble ending, however, does nothing to diminish the accomplishments of an organization that is yet to receive its historical due.

 

Q. 45 The primary purpose of this passage is to.

A. describe the barriers confronting women in the contemporary workplace

B. call readers’ attention to an overlooked contributor to American history

C. critique the methods employed by an important labor union

D. rebuke historians for failing to cover the women’s labor movement adequately

 

Q. 46 Which of the following best characterizes the American Federation of Labor’s view of the Women’s Trade Union League, as it is presented in the passage?

A. The WTUL was an important component of the AFLs multi front assault on industry and its treatment of workers-

B. Because of Eleanor Roosevelt’s affiliation with the organization, the WTUL was a vehicle through which the AFL could gain access to the White House.

C. The WTUL was to. be avoided because the radical element within it attracted unwanted government scrutiny.

D. The WTUL offered the AFL some political capital but little that would assist it in labour negotiations.

 

Q. 47 Each ofthe following is cited in the passage as an accomplishment of the Women’s Trade Union League EXCEPT

A. It organized a highly skilled workforce to increase its bargaining power.

B. It contributed to the development of a group of leaders in America.

C. It provided essential support to striking women.

D. It helped fund start-up unions for women.

 

Questions: 48 – 53

Directions (Q.48 to 53) : Read the following passage and give answer based on it. The function of strategic planning is to. position a company for long-term growth and expansion in a variety of markets by analyzing its strengths and weaknesses and examining current and potential opportunities. Based on this information, the company develops strategy for itself. That strategy then becomes the basis for supporting strategies for its various departments. This is where all too. many strategic plans go astray-at implementation. Recent business management surveys show that most CEOs who have a strategic plan are concerned with the potential breakdown in the implementation of the plan. Unlike 1980s corporations that blindly followed their 5-year plans, even when they were misguided, today’s corporations tend to. Second-guess. Outsiders can help facilitate the process, but in the final analysis, ifthe company doesn’t make the plan, the company won’t follow the plan. This was one of the problems with strategic planning in the 1980s. In that era, it was an abstract, top down process involving only a few top corporate officers and hired guns. Number crunching experts came into a company and generated tome-like volumes filled with a mixture of abstruse facts and grand theories which had little to do with the day- to-day realities of the company. Key middle managers were left out of planning sessions, resulting in lost opportunities and ruffled feelings. However, more hands-on strategic planning can produce startling results. A recent survey queried more than a thousand small-to- jnedium sized businesses to compare companies with a strategic plan to companies without one. The survey found that companies with strategic plans had annual revenue growth of 6.2 percent as opposed to 3.8 percent for the other companies. Perhaps most important, a strategic plan helps companies anticipate-and survive-change. New technology and the mobility of capital mean that markets can shift faster than ever before. Some financial analysts wonder why they should bother planning two. years ahead when market dynamics might be transformed by next quarter. The fact is that it’s the very pace of change that makes planning so crucial. Now, more than ever, companies have to stay alert to the marketplace, In an environment of continual and rapid change, long range planning expands options and organizational flexibility.

 

Q. 48 The primary purpose of the passage is to

A. refute the idea that change is bad for a corporation’s long-term health

B. describe how long-term planning, despite some potential pitfalls, can help a corporation to. grow

C. compare and contrast two styles of corporate planning

D. evaluate the strategic planning goals of corporate America today

 

Q. 49 It can be inferred from the passage that, in general, strategic planning during the 1980s had all of the following shortcomings EXCEPT

A. a reliance on outside consultants who did not necessarily understand the nuts and bolts of the business

B. a dependence on theoretical models that did not always perfectly describe the workings of the company

C. an inherent weakness in the company’s own ability to implement the strategic plan

D. an excess of information and data that made it difficult to get to key concepts

 

Q. 50 The author most likely mentions the results of the survey of 1,000 companies in order to  

A. put forth an opposing view on strategic plans so that she can then refute it

B. illustrate that when strategic planning is “hands-on,” it produces uninspiring results

C. give a concrete example of why strategic planning did not work during the 1980s

D. support her contention that strategic planning when done correctly can be very successful

 

Q. 51 Over the last 20 years the rate of increase in total production in Workland has been second to none in the world. However, the growth is more modest when calculated per capita of total population. Over the last ten years progress has been much slower. If the information above is accurate, which of the following must be true?

A. Work land has a very large population.

B. Productivity per capita has not grown as fast during the past ten years.

C. Total production has increased faster than population growth ..

D. The birth rate has declined.

 

Q. 52 Between 1979 and 1983, the number of unincorporated business self-employed women increased five times faster than the number of self employed men and more than three times faster than women wage-and-salary workers. Part-time self-employment among women increased more than full-time self-employment. Each of the following, if true, could help to account for this trend except:

A. Owning a business affords flexibility to combine work and family responsibilities.

B. The proportion of women studying business administration courses has grown considerable.

C. There are more self-employed women than men.

D. Uninorporated service industries have grown by 300 percent over the period; the ratio of women to men in this industry is three to one.

 

Q. 53 There is no clear line between health and illness; it is easy to forget what it feels like to be really well and to get gradually used to often having a headache, feeling irritable, or tired. There is an unrecognized proportion of the population that has been tipped over the brink into ill health by ubiquitous contaminants.

Which of the following statements best describes the purpose of the abvoe?

A. The public must be encouraged to have regular medical examinations.

B. The public must be warned ~o be aware of various physical and chemical hazards.

C. The public must be warned to treat seriously such symptoms as headaches, irritability, and tiredness.

D. The medical professional is not always capable of diagnosing illness.

 

Questions: 54 – 55

Directions (Q.54 – 55) : In each question, there are five sentences. Each sentence has a pair of words that are italicized and highlighted. From the italicized and highlighted words, select the most appropriate words (A or B) to form correct sentences. The sentences are followed by options that indicate the words, which may be selected to correctly complete the set of sentences. From the options given, choose the most appropriate one.

 

Q. 54 The further [A] /farther [B] he pushed himself, the more disillusioned he grew.

For the crowds it was more ofa historical [A] / historic [B] event; for their leader, it was just another day.

The old man has a healthy distrust [A] / mistrust [B] for all new technology. This film is based on a real [A] / true [B] story .

One suspects that the compliment [A] / complement [B] was backhanded.

A. BABAB

B. ABABA

C. BAABA

D. BBAAB

 

Q. 55 Regrettably [A] / Regretfully [B] I have to decline your invitation.

I am drawn to the poetic, sensual [A] / sensuous [B] quality of her paintings.

He was besides [A] / beside [B] himself with age when I told him what J had done.

After brushing against a stationary [A] / stationery [B] truck my car turned turtle .

As the water began to rise over [A] / above [B] the danger mark, the signs of an imminent flood were clear.

A. BAABA

B. BBBAB

C. AAABA

D. BBAAB

 

Questions: 56 – 57

Directions (Q.56 – 57) : In each question, the word at the top is used in four different ways, numbered (a) to (d). Choose the option in which the usage of the word is Incorrect or Inappropriate.

 

Q. 56 Choose the correct answer.

SORT

A. Let’s sort these boys into four groups

B. They serve tea of a sort on these trains.

C. Farmers of all sort attended the rally.

D. Farmers of all sort attended the rally.

 

Q. 57 Choose the correct answer.

HOST

A. A virus has infected the host computer

B. Ranchi will play the host to the next national film festival

C. Kerala’ s forests are host to a range of snakes

D. If you host the party, who will foot the bill

 

Q. 58 Select the one which would best fill the blanks.

Football evokes a ________ response in India compared to cricket, the almost _________ the nation

A. tepid boiling

B. lukewarm, electrifies

C. turbid, fascinating

D. apocryphal, genuinely fascinates

 

Questions: 59 – 60

Directions (Q.59 – 60): Each question consists of four sentences on a topic. Some sentences are grammatically incorrect or inappropriate. Select the option that indicates the grammatically correct and. appropriate sentence(s).

 

Q. 59 A. Last Sunday, Archana had nothing to do.

B. After waking up, she lay on the bed thinking of what to do.

C. At 11 o’clock she took shower and got ready.

D. She spent most of the day shopping.

A. B and C

B. C

C. A and B

D. B , C and D

 

Q. 60 A. Large reductions in the ozone layer, which sits about 15-30 km above the Earth, take place each winter over the polar regions, especially the Antarctic, as low temperatures allow the formation of stratospheric clouds that assist chemical reactions breaking down ozone.

B. Industrial chemicals containing chlorine and bromine have-been blamed for thinning the layer because they attack the ozone molecules, making them to break apart.

C. Many an offending chemicals have now been banned.

D. It will still take several decades before these substances have disappeared from the atmosphere.

A. D

B. B and D

C. A and D

D. A and C

 

 

Answer Sheet
Question 1 2 3 4 5 6 7 8 9 10
Answer A D C B D A C B A D
Question 11 12 13 14 15 16 17 18 19 20
Answer A D A C C A B C A B
Question 21 22 23 24 25 26 27 28 29 30
Answer C D B B D D B D B D
Question 31 32 33 34 35 36 37 38 39 40
Answer C D C B A D D B D D
Question 41 42 43 44 45 46 47 48 49 50
Answer D D D C B D A B C D
Question 51 52 53 54 55 56 57 58 59 60
Answer B C B B B C C B D C

CAT Previous Year Paper 2008

CAT 2008

Section

Questions

Marks

Quantitative Aptitude

19 Questions

76

Logical Reasoning

24 Questions

96

Data Interpretation

7 Questions

28

Verbal Ability

20 Questions (51 – 70)

80

Reading Comprehension

20 Questions (71 – 90)

80

Q. 1 The integers 1, 2, …, 40 are written on a blackboard. The following operation is then repeated 39 times : In each repetition, any two numbers, say a and b, currently on the blackboard are erased and a new number a + b – 1 is written. What will be the number left on the board at the end ?

A. 820

B. 821

C. 781

D. 819

E. 780

 

Q. 2 What are the last two digits of 7²⁰⁰⁸ ?

A. 21

B. 61

C. 01

D. 41

E. 81

 

Q. 3 If the roots of the equation x³ – ax² + bx – c = 0 are three consecutive integers, then what is the smallest possible value of b ?

A. -1/√3

B. –1

C. 0

D. 1

E. 1/√3

 

Q. 4 A shop stores x kg of rice. The first customer buys half this amount plus half a kg of rice. The second customer buys half the remaining amount plus half a kg of rice. Then the third customer also buys half the remaining amount plus half a kg of rice. Thereafter, no rice is left in the shop. Which of the following best describes the value of x?

A. 2 ≤ x ≤ 6

B. 5 ≤ x ≤ 8

C. 9 ≤ x ≤ 12

D. 11≤ x ≤ 14

E. 13 ≤ x ≤ 18

 

Questions: 5 – 6

Directions for Questions 5 and 6:

Let f(x) = ax² + bx + c, where a, b and c are certain constants and a ≠ 0 It is known that f = – 3f and that 3 is a root of f(x) = 0.

 

Q. 5 What is the other root of f(x) = 0?

A. –7

B. – 4

C. 2

D. 6

E. cannot be determined

 

Q. 6 What is the value of a + b + c?

A. 9

B. 14

C. 13

D. 37

E. cannot be determined

 

Q. 7 The number of common terms in the two sequences 17, 21, 25,…, 417 and 16, 21, 26,…, 466 is

A. 78

B. 19

C. 20

D. 77

E. 22

 

Q. 8 How many integers, greater than 999 but not greater than 4000, can be formed with the digits 0, 1, 2, 3 and 4, if repetition of digits is allowed?

A. 499

B. 500

C. 375

D. 376

E. 501

 

Questions: 9 – 10

The figure below shows the plan of a town. The streets are at right angles to each other. A rectangular park (P) is situated inside the town with a diagonal road running through it. There is also a prohibited region (D) in the town.

 

Q. 9 Neelam rides her bicycle from her house at A to her office at B, taking the shortest path. Then the number of possible shortest paths that she can choose is

A. 60

B. 75

C. 45

D. 90

E. 72

 

Q. 10 Neelam rides her bicycle from her house at A to her club at C, via B taking the shortest path. Then the number of possible shortest paths that she can choose is

A. 1170

B. 630

C. 792

D. 1200

E. 936

 

Q. 11 Let f(x) be a function satisfying f(x)f(y) = f(xy) for all real x, y. If f = 4, then what is the value of f(1/2)?

A. 0

B. 1/4

C. 1/2

D. 1

E. cannot be determined

 

Q. 12 Suppose, the seed of any positive integer n is defined as follows:

seed(n) = n, if n < 10 = seed(s(n)), otherwise,

where s(n) indicates the sum of digits of n. For example, seed(7) = 7, seed(248) = seed(2 + 4 + 8) = seed(14) = seed(1 + 4) = seed = 5 etc. How many positive integers n, such that n < 500, will have seed (n) = 9?

A. 39

B. 72

C. 81

D. 108

E. 55

 

Q. 13 In a triangle ABC, the lengths of the sides AB and AC equal 17.5 cm and 9 cm respectively. Let D be a point on the line segment BC such that AD is perpendicular to BC. If AD = 3 cm, then what is the radius (in cm) of the circle circumscribing the triangle ABC?

A. 17.05

B. 27.85

C. 22.45

D. 32.25

E. 26.25

 

Q. 14 Consider obtuse-angled triangles with sides 8 cm, 15 cm and x cm. If x is an integer, then how many such triangles exist?

A. 5

B. 21

C. 10

D. 15

E. 14

 

Q. 15 Consider a square ABCD with midpoints E, F, G, H of AB, BC, CD and DA respectively. Let L denote the line passing through F and H. Consider points P and Q, on L and inside ABCD, such that the angles APD and BQC both equal 120°. What is the ratio of the area of ABQCDP to the remaining area inside ABCD?

A. 4√2/3

B. 2+√3

C. 10-3√3/9

D. 1+1/√3

E. 2√3−1

 

Q. 16 What is the number of distinct terms in the expansion of (a + b + c)²⁰?

A. 231

B. 253

C. 242

D. 210

E. 228

 

Questions: 17 – 18

Directions for Questions 17 and 18:

Five horses, Red, White, Grey, Black and Spotted participated in a race. As per the rules of the race, the persons betting on the winning horse get four times the bet amount and those betting on the horse that came in second get thrice the bet amount. Moreover, the bet amount is returned to those betting on the horse that came in third, and the rest lose the bet amount. Raju bets Rs. 3000, Rs. 2000 and Rs. 1000 on Red, White and Black horses respectively and ends up with no profit and no loss.

 

Q. 17 Which of the following cannot be true?

A. At least two horses finished before Spotted

B. Red finished last

C. There were three horses between Black and Spotted

D. There were three horses between White and Red

E. Grey came in second

 

Q. 18 Suppose, in addition, it is known that Grey came in fourth. Then which of the following cannot be true?

A. Spotted came in first

B. Red finished last

C. White came in second

D. Black came in second

E. There was one horse between Black and White

 

Questions: 19 – 20

Directions for Questions 19-20:

Mark (1) if Q can be answered from A alone but not from B alone.

Mark (2) if Q can be answered from B alone but not from A alone.

Mark (3) if Q can be answered from A alone as well as from B alone.

Mark (4) if Q can be answered from A and B together but not from any of them alone.

Mark (5) if Q cannot be answered even from A and B together.

In a single elimination tournament, any player is eliminated with a single loss.

The tournament is played in multiple rounds subject to the following rules :

(a) If the number of players, say n, in any round is even, then the players are grouped into n/2 pairs. The players in each pair play a match against each other and the winner moves on to the next round.

(b) If the number of players, say n, in any round is odd, then one of them is given a bye, that is he automatically moves on to the next round. The remaining (n–1) players are grouped into (n–1)/2 pairs. The players in each pair play a match against each other and the winner moves on to the next round. No player gets more than one bye in the entire tournament.

Thus, if n is even, then n/2 players move on to the next round while if n is odd, then (n+1)/2 players move on to the next round. The process is continued till the final round, which obviously is played between two players. The winner in the final round is the champion of the tournament.

 

Q. 19 What is the number of Matches played by the champion?

A. The entry list for the tournament consists of 83 players.

B. The champion received one bye.

A. Q can be answered from A alone but not from B alone.

B. Q can be answered from B alone but not from A alone.

C. Q can be answered from A alone as well as from B alone.

D. Q can be answered from A and B together but not from any of them alone.

E. Q cannot be answered even from A and B together.

 

Q. 20 If the number of players, say n, in the first round was between 65 and 128, then what is the exact value of n?

A. Exactly one player received a bye in the entire tournament.

B. One player received a bye while moving on to the fourth round from the third round.

A. Q can be answered from A alone but not from B alone.

B. Q can be answered from B alone but not from A alone.

C. Q can be answered from A alone as well as from B alone.

D. Q can be answered from A and B together but not from any of them alone.

E. Q cannot be answered even from A and B together.

 

Q. 21 Two circles, both of radii 1 cm, intersect such that the circumference of each one passes through the centre of the other. What is the area (in sq. cm.) of the intersecting region?

A. π/3-√3/4

B. 2π/3+√3/2

C. 4π/3-√3/2

D. 4π/3+√3/2

E. 2π/3-√3/2

 

Q. 22 Rahim plans to drive from city A to station C, at the speed of 70 km per hour, to catch a train arriving there from B. He must reach C at least 15 minutes before the arrival of the train. The train leaves B, located 500 km south of A, at 8:00 am and travels at a speed of 50 km per hour. It is known that C is located between west and northwest of B, with BC at 60° to AB. Also, C is located between south and southwest of A with AC at 30° to AB. The latest time by which Rahim must leave A and still catch the train is closest to

A. 6 : 15 am

B. 6 : 30 am

C. 6 :45 am

D. 7 : 00 am

E. 7 : 15 am

 

Q. 23 Three consecutive positive integers are raised to the first, second and third powers respectively and then added. The sum so obtained is perfect square whose square root equals the total of the three original integers. Which of the following best describes the minimum, say m, of these three integers?

A. 1≤ m ≤ 3

B. 4 ≤ m ≤ 6

C. 7 ≤ m ≤ 9

D. 10 ≤ m ≤ 12

E. 13 ≤ m ≤ 15

 

Q. 24 Find the sum of the following

A. 2008-1/2008

B. 2007-1/2007

C. 2007-1/2008

D. 2008-1/2007

E. 2008-1/2009

 

Q. 25 Consider a right circular cone of base radius 4 cm and height 10 cm. A cylinder is to be placed inside the cone with one of the flat surfaces resting on the base of the cone. Find the largest possible total surface area (in sq. cm) of the cylinder.

A. 100π/3

B. 80π/3

C. 120π/7

D. 130π/9

E. 110π/7

 

Questions: 26 – 28

Answer the following questions based on the statements given below:

(i) There are three houses on each side of the road.

(ii) These six houses are labelled as P, Q, R, S, T and U.

(iii) The houses are of different colours, namely, Red, Blue, Green, Orange, Yellow and White.

(iv) The houses are of different heights.

(v) T, the tallest house, is exactly opposite to the Red coloured house.

(vi) The shortest house is exactly opposite to the Green coloured house.

(vii) U, the Orange coloured house, is located between P and S.

(viii) R, the Yellow coloured house, is exactly opposite to P.

(ix) Q, the Green coloured house, is exactly opposite to U.

(x) P, the White coloured house, is taller than R, but shorter than S and Q.

 

Q. 26 What is the colour of the house diagonally opposite to the Yellow coloured house?

A. White

B. Blue

C. Green

D. Red

E. none of these

 

Q. 27 Which is the second tallest house?

A. P

B. S

C. Q

D. R

E. cannot be determined

 

Q. 28

What is the colour of the tallest house?

A. Red

B. Blue

C. Green

D. Yellow

E. none of these

 

Questions: 29 – 31

Directions for questions 29 to 31:

Answer the following questions based on the information given below:

Telecom operators get revenue from transfer of data and voice. Average revenue received from transfer of each unit of data is known as ARDT. In the diagram below, the revenue received from data transfer as percentage of total revenue received and the ARDT in US Dollars (USD) are given for various countries.

 

Q. 29 It was found that the volume of data transfer in India is the same as that of Singapore. Then which of the following statements is true?

A. Total revenue is the same in both countries.

B. Total revenue in India is about 2 times that of Singapore.

C. Total revenue in India is about 4 times that of Singapore.

D. Total revenue in Singapore is about 2 times that of India.

E. Total revenue in Singapore is about 4 time that of India.

 

Q. 30 It is expected that by 2010, revenue from data transfer as a percentage of total revenue will triple for India and double for Sweden. Assume that in 2010, the total revenue in India is twice that of Sweden and that the volume of data transfer is the same in both the countries. What is the percentage increase of ARDT in India if there is no change in ARDT in Sweden?

A. 400%

B. 550%

C. 800%

D. 950%

E. cannot be determined

 

Q. 31 If the total revenue received is the same for the pairs of countries listed in the choices below, choose the pair that has approximately the same volume of data transfer.

A. Philippines and Austria

B. Canada and Poland

C. Germany and USA

D. UK and Spain

E. Denmark and Mexico

 

Questions: 32 – 34

Answer the following questions based on the information given below:

For admission to various affiliated colleges, a university conducts a written test with four different sections, each with a maximum of 50 marks. The following table gives the aggregate as well as the sectional cut-off marks fixed by six different colleges affiliated to the university. A student will get admission only if he/she gets marks greater than or equal to the cut-off marks in each of the sections and his/her aggregate marks are at least equal to the aggregate cut-off marks as specified by the college.

 

Q. 32 Bhama got calls from all colleges. What could be the minimum aggregate marks obtained by her?

A. 180

B. 181

C. 196

D. 176

E. 184

 

Q. 33 Charlie got calls from two colleges. What could be the minimum marks obtained by him in a section?

A. 0

B. 21

C. 25

D. 35

E. 41

 

Q. 34 Aditya did not get a call from even a single college. What could be the maximum aggregate marks obtained by him?

A. 181

B. 176

C. 184

D. 196

E. 190

 

Questions: 35 – 38

Directions for Questions 35 to 38:

Answer the following questions based on the information given below:

In a sports event, six teams (A, B, C, D, E and F) are competing against each other. Matches are scheduled in two stages. Each team plays three matches in stage – I and two matches in Stage – II. No team plays against the same team more than once in the event. No ties are permitted in any of the matches. The observations after the completion of Stage – I and Stage – II are as given below.

Stage-I:

• One team won all the three matches.

• Two teams lost all the matches.

• D lost to A but won against C and F.

• E lost to B but won against C and F.

• B lost at least one match.

• F did not play against the top team of Stage-I.

Stage-II:

• The leader of Stage-I lost the next two matches.

• Of the two teams at the bottom after Stage-I, one team won both matches, while the other lost both matches.

• One more team lost both matches in Stage-II.

 

Q. 35 The two teams that defeated the leader of Stage-I are:

A. F & D

B. E & F

C. B & D

D. E & D

E. F & D

 

Q. 36 The only team(s) that won both matches in Stage-II is (are):

A. B

B. E & F

C. A, E & F

D. B, E & F

E. B & F

 

Q. 37 The teams that won exactly two matches in the event are:

A. A, D & F

B. D & E

C. E & F

D. D, E & F

E. D & F

 

Q. 38 The team(s) with the most wins in the event is (are):

A. A

B. A & C

C. F

D. E

E. B & E

 

Questions: 39 – 42

Directions for Questions 39 to 42:

Answer the following questions based on the information given below:

The bar chart below shows the revenue received in million US Dollars (USD), from subscribers to a particular Internet service. The data covers the period 2003 to 2007 for the United States (US) and Europe. The bar chart also shows the estimated revenues from subscription to this service for the period 2008 to 2010.

 

Q. 39 The difference between the estimated subscription in Europe in 2008 and what it would have been if it were computed using the percentage growth rate of 2007 (over 2006), is closest to:

A. 50

B. 80

C. 20

D. 10

E. 0

 

Q. 40 In 2003, sixty percent of subscribers in Europe were men. Given that women subscribers increase at the rate of 10 percent per annum and men at the rate of 5 percent per annum, what is the approximate percentage growth of subscribers between 2003 and 2010 in Europe? The subscription prices are volatile and may change each year.

A. 62

B. 15

C. 78

D. 84

E. 50

 

Q. 41 Consider the annual percent change in the gap between subscription revenues in the US and Europe. What is the year in which the absolute value of this change is the highest?

A. 03 – 04

B. 05 – 06

C. 06 – 07

D. 08 – 09

E. 09 – 10

 

Q. 42 While the subscription in Europe has been growing steadily towards that of the US, the growth rate in Europe seems to be declining. Which of the following is closest to the percent change in growth rate of 2007 (over 2006) relative to the growth rate of 2005 (over 2004)?

A. 17

B. 20

C. 35

D. 60

E. 100

 

Questions: 43 – 45

Answer the following questions based on the information given below:

Abdul, Bikram and Chetan are three professional traders who trade in shares of a company XYZ Ltd. Abdul follows the strategy of buying at the opening of the day at 10 am and selling the whole lot at the close of the day at 3 pm. Bikram follows the strategy of buying at hourly intervals: 10 am, 11 am, 12 noon, 1 pm, and 2 pm, and selling the whole lot at the close of the day. Further, he buys an equal number of shares in each purchase. Chetan follows a similar pattern as Bikram but his strategy is somewhat different. Chetan’s total investment amount is divided equally among his purchases. The profit or loss made by each investor is the difference between the sale value at the close of the day less the investment in purchase. The “return” for each investor is defined as the ratio of the profit or loss to the investment amount expressed as a percentage.

 

Q. 43 On a day of fluctuating market prices, the share price of XYZ Ltd. ends with a gain, i.e, it is higher at the close of the day compared to the opening value. Which trader got the maximum return on that day?

A. Bikram

B. Chetan

C. Abdul

D. Bikram or Chetan

E. cannot be determined

 

Q. 44 Which one of the following statements is always true?

A. Abdul will not be one with the minimum return

B. Return for Chetan will be higher than that of Bikram

C. Return for Bikram will be higher than that of Chetan

D. Return for Chetan cannot be higher than that of Abdul

E. none of the above

 

Q. 45 On a “boom” day the share price of XYZ Ltd. keeps rising throughout the day and peaks at the close of the day. Which trader got the minimum return on that day?

A. Bikram

B. Chetan

C. Abdul

D. Abdul or Chetan

E. cannot be determined

 

Questions: 46 – 47

Answer the following questions based on the information given below:

One day, two other traders, Dane and Emily joined Abdul, Bikram and Chetan for trading in the shares of XYZ Ltd. Dane followed a strategy of buying equal numbers of shares at 10 am. 11 am and 12 noon, and selling the same numbers at 1 pm, 2 pm and 3 pm. Emily, on the other hand, followed the strategy of buying shares using all her money at 10 am and selling all of them at 12 noon and again buying the shares for all the money at 1 pm and again selling all of them at the close of the day at 3 pm. At the close of the day the following was observed.

i. Abdul lost money in the transactions.

ii. Both Dane and Emily made profits.

iii. There was an increase in share price during the closing hour compared to the price at 2 pm.

iv. Share price at 12 noon was lower than the opening price

 

Q. 46 Share price was at its highest at

A. 10 am

B. 11 am

C. 12 noon

D. 1 pm

E. cannot be determined

 

Q. 47 Which of the following is necessarily false?

A. Share price was at its lowest at 2 pm

B. Share price was at its lowest at 11 am

C. Share price at 1 pm was higher than the share price at 2 pm

D. Share price at 1 pm was higher than the share price at 12 noon

E. none of the above

 

Questions: 48 – 50

Answer the following questions based on the information given below:

There are 100 employees in an organization across five departments. The following table gives the department- wise distribution of average age, average basic pay and allowances. The gross pay of an employee is the sum of his/her basic pay and allowances.

There are limited numbers of employees considered for transfer/promotion across departments. Whenever a person is transferred/promoted from a department of lower average age to a department of higher average age, he/she will get an additional allowance of 10% of basic pay over and above his/her current allowance. There will not be any change in pay structure if a person is transferred/promoted from a department with higher average age to a department with lower average age.

Questions below are independent of each other.

 

Q. 48 What is the approximate percentage change in the average gross of the HR department due to transfer of a 40-year old person with basic pay of Rs. 8000 from the Marketing department?

A. 9%

B. 11%

C. 13%

D. 15%

E. 17%

 

Q. 49 There was a mutual transfer of an employee between Marketing and Finance departments and transfer of one employee from Marketing to HR. As a result, the average age of Finance department increased by one year and that of Marketing department remained the same. What is the new average age of HR department?

A. 30

B. 35

C. 40

D. 45

E. cannot be determined

 

Q. 50 If two employees (each with a basic pay of Rs. 6000) are transferred from Maintenance department to HR department and one person (with a basic pay of Rs. 8000) was transferred from Marketing department to HR department, what will be the percentage change in average basic pay of HR department?

A. 10.5%

B. 12.5%

C. 15%

D. 30%

E. 40%

 

Questions: 51 – 54

In each of the following questions there are sentences that form a paragraph.

Identify the sentence(s) or part(s) of sentence(s) that is/are correct in terms of grammar and usage (including spelling, punctuation and logical consistency). Then, choose the most appropriate option.

 

Q. 51 A. In 1849, a poor Bavarian immigrant named Levi Strauss

B. landed in San Francisco, California,

C. at the invitation of his brother-in-law David Stern

D. owner of dry goods business.

E. This dry goods business would later became known as Levi Strauss & Company.

A. B only

B. B and C

C. A and B

D. A only

E. A, B and D

 

Q. 52 A. In response to the allegations and condemnation pouring in,

B. Nike implemented comprehensive changes in their labour policy.

C. Perhaps. sensing the rising tide of global labour concerns,

D. from the public would become a prominent media issue,

E. Nike sought to be a industry leader in employee relations.

A. D and E

B. D only

C. A and E

D. A and D

E. B, C and E

 

Q. 53 A. Charges and counter charges mean nothing

B. to the few million who have lost their home.

C. The nightmare is far from over, for the government

D. is still unable to reach hundreds who are marooned.

E. The death count have just begun.

A. A only

B. C only

C. A and C

D. A, C and D

E. D only

 

Q. 54 A. I did not know what to make of you.

B. Because you’d lived in India, I associate you more with my parents than with me.

C. And yet you were unlike my cousins in Calcutta, who seem so innocent and obedient when I visited them.

D. You were not curious about me in the least.

E. Although you did make effort to meet me.

A. A only

B. A and B

C. A and E

D. D only

E. A and D

 

Questions: 55 – 58

In each question, there are five sentences. Each sentence has a pair of words that are italicized and highlighted. From the italicized and highlighted words, select the most appropriate words (A or B) to form correct sentences. The sentences are followed by options that indicate the words, which may be selected to correctly complete the set of sentences. From the options given, choose the most appropriate one.

 

Q. 55 Anita wore a beautiful broach (A)/brooch (B) on the lapel of her jacket.

If you want to complain about the amenities in your neighbourhood, please meet your councillor(A)/counsellor(B).

I would like your advice(A)/advise(B) on which job I should choose.

The last scene provided a climactic(A)/climatic(B) ending to the film.

Jeans that flair(A)/flare(B) at the bottom are in fashion these days.

A. BABAA

B. BABAB

C. BAAAB

D. ABABA

E. BAABA

 

Q. 56 The cake had lots of currents(A)/currants(B) and nuts in it.

If you engage in such exceptional(A)/exceptionable(B) behaviour, I will be forced to punish you.

He has the same capacity as an adult to consent(A)/assent(B) to surgical treatment. The minister is obliged (A)/compelled(B) to report regularly to a parliamentary board. His analysis of the situation is far too sanguine(A)/genuine(B).

A. BBABA

B. BBAAA

C. BBBBA

D. ABBAB

E. BABAB

 

Q. 57 She managed to bite back the ironic(A)/caustic(B) retort on the tip of her tongue. He gave an impassioned and valid(A)/cogent(B) plea for judicial reform.

I am not adverse(A)/averse(B) to helping out. The coupé(A)/coup(B) broke away as the train climbed the hill. They heard the bells peeling(A)/pealing(B) far and wide.

A. BBABA

B. BBBAB

C. BAABB

D. ABBAA

E. BBBBA

 

Q. 58 We were not successful in defusing(A)/diffusing(B) the Guru’s ideas.

The students baited(A)/bated(B) the instructor with irrelevant questions.

The hoard(A)/horde(B) rushed into the campus.

The prisoner’s interment(A)/interment(B) came to an end with his early release.

The hockey team could not deal with his unsociable(A)/unsocial(B) tendencies.

A. BABBA

B. BBABB

C. BABAA

D. ABBAB

E. AABBA

 

Questions: 59 – 62

In each of the questions, a word has been used in sentences in five different ways. Choose the option corresponding to the sentence in which the usage of the word is incorrect or inappropriate.

 

Q. 59 The word is – Run

A. I must run fast to catch up with him.

B. Our team scored a goal against the run of play.

C. You can’t run over him like that.

D. The newly released book is enjoying a popular run.

E. This film is a run-of-the-mill production.

 

Q. 60 The word is – Round

A. The police fired a round of tear gas shells.

B. The shop is located round the corner.

C. We took a ride on the merry-go-round.

D. The doctor is on a hospital round.

E. I shall proceed further only after you come round to admitting it.

 

Q. 61 The word is – Buckle

A. After the long hike our knees were beginning to buckle.

B. The horse suddenly broke into a buckle.

C. The accused did not buckle under police interrogation.

D. Sometimes, an earthquake can make a bridge buckle.

E. People should learn to buckle up as soon as they get into a car.

 

Q. 62 The word is – File

A. You will find the paper in the file under C.

B. I need to file an insurance claim.

C. The cadets were marching in a single file.

D. File your nails before you apply nail polish.

E. When the parade was on, a soldier broke the file.

 

Questions: 63 – 66

Directions for Questions 63 to 66:

Each of the following questions has a sentence with two blanks. Given below each question are five pairs of words. Choose the pair that best completes the sentence.

 

Q. 63 The genocides in Bosnia and Rwanda, apart from being mis-described in the most sinister and _________ manner as ‘ethnic cleansing’, were also blamed, in further hand-washing rhetoric, on something dark and interior to __________ and perpetrators alike.

A. innovative; communicator

B. enchanting; leaders

C. disingenuous; victims

D. exigent; exploiters

E. tragic; sufferers

 

Q. 64 As navigators, calendar makers, and other __________ of the night sky accumulated evidence to the contrary, ancient astronomers were forced to __________ that certain bodies might move in circles about points, which in turn moved in circles about the earth.

A. scrutinizers; believe

B. observers; agree

C. scrutinizers; suggest

D. observers; concede

E. students; conclude

 

Q. 65 Every human being, after the first few days of his life, is a product of two factors: on the one hand, there is his __________ endowment; and on the other hand, there is the effect of environment, including _____

A. constitutional; weather

B. congenital; education

C. personal; climate

D. economic; learning

E. genetic; pedagogy

 

Q. 66 Exhaustion of natural resources, destruction of individual initiative by governments, control over men’s minds by central _______ of education and propaganda are some of the major evils which appear to be on the increase as a result of the impact of science upon minds suited by __________ to an earlier kind of world.

A. tenets; fixation

B. aspects; inhibitions

C. institutions; inhibitions

D. organs; tradition

E. departments; repulsion

 

Questions: 67 – 70

Directions for Questions 67 to 70:

Each of the following questions has a paragraph from which the last sentence has been deleted. From the given options, choose the sentence that completes the paragraph in the most appropriate way.

 

Q. 67 Most people at their first consultation take a furtive look at the surgeon’s hands in the hope of reassurance. Prospective patients look for delicacy, sensitivity, steadiness, perhaps unblemished pallor. On this basis, Henry Perowne loses a number of cases each year. Generally, he knows it’s about to happen before the patient does: the downward glance repeated, the prepared questions beginning to falter, the overemphatic thanks during the retreat to the door.

A. Other people do not communicate due to their poor observation.

B. Other patients don’t like what they see but are ignorant of their right to go elsewhere.

C. But Perowne himself is not concerned.’

D. But others will take their place, he thought.

E. These hands are steady enough, but they are large.

 

Q. 68 Trade protectionism, disguised as concern for the climate, is raising its head. Citing competitiveness concerns, powerful industrialized countries are holding out threats of a levy on imports of energy-intensive products from developing countries that refuse to accept their demands. The actual source of protectionist sentiment in the OECD countries is, of course, their current lacklustre economic performance, combined with the challenges posed by the rapid economic rise of China and India – in that order.

A. Climate change is evoked to bring trade protectionism through the back door.

B. OECD countries are taking refuge in climate change issues to erect trade barriers against these two countries.

C. Climate change concerns have come as a convenient stick to beat the rising trade power of China and India.

D. Defenders of the global economic status quo are posing as climate change champions.

E. Today’s climate change champions are the perpetrators of global economic inequity.

 

Q. 69 Mattancherry is Indian Jewry’s most famous settlement. Its pretty streets of pastel coloured houses, connected by first-floor passages and home to the last twelve saree-and-sarongwearing, whiteskinned Indian Jews are visited by thousands of tourists each year. Its synagogue, built in 1568, with a floor of blue-and-white Chinese tiles, a carpet given by Haile Selassie and the frosty Yaheh selling tickets at the door, stands as an image of religious tolerance.

A. Mattancherry represents, therefore, the perfect picture of peaceful co-existence.

B. India’s Jews have almost never suffered discrimination, except for European colonizers and each other.

C. Jews in India were always tolerant.

D. Religious tolerance has always been only a façade and nothing more.

E. The pretty pastel streets are, thus, very popular with the tourists.

 

Q. 70 Given the cultural and intellectual interconnections, the question of what is ‘Western’ and what is ‘Eastern’ (or ‘Indian’) is often hard to decide, and the issue can be discussed only in more dialectical terms. The diagnosis of a thought as ‘purely Western’ or ‘purely Indian’ can be very illusory.

A. Thoughts are not the kind of things that can be easily categorized.

B. Though ‘occidentalism’ and ‘orientalism’ as dichotomous concepts have found many adherents.

C. ‘East is East and West is West’ has been a discredited notion for a long time now.

D. Compartmentalizing thoughts is often desirable.

E. The origin of a thought is not the kind of thing to which ‘purity’ happens easily.

 

Questions: 71 – 75

The passage given below is followed by a set of five questions. Choose the most appropriate answer to each question.

When I was little, children were bought two kinds of ice cream, sold from those white wagons with canopies made of silvery metal: either the two-cent cone or the four-cent ice-cream pie. The two-cent cone was very small, in fact it could fit comfortably into a child’s hand, and it was made by taking the ice cream from its container with a special scoop and piling it on the cone. Granny always suggested I eat only a part of the cone, then throw away the pointed end, because it had been touched by the vendor’s hand (though that was the best part, nice and crunchy, and it was regularly eaten in secret, after a pretence of discarding it). The four-cent pie was made by a special little machine, also silvery, which pressed two disks of sweet biscuit against a cylindrical section of ice cream. First you had to thrust your tongue into the gap between the biscuits until it touched the central nucleus of ice cream; then, gradually, you ate the whole thing, the biscuit surfaces softening as they became soaked in creamy nectar. Granny had no advice to give here: in theory the pies had been touched only by the machine; in practice, the vendor had held them in his hand while giving them to us, but it was impossible to isolate the contaminated area. I was fascinated, however, by some of my peers, whose parents bought them not a four-cent pie but two two-cent cones. These privileged children advanced proudly with one cone in their right hand and one in their left; and expertly moving their head from side to side, they licked first one, then the other. This liturgy seemed to me so sumptuously enviable, that many times I asked to be allowed to celebrate it. In vain. My elders were inflexible: a four-cent ice, yes; but two two-cent ones, absolutely no. As anyone can see, neither mathematics nor economy nor dietetics justified this refusal. Nor did hygiene, assuming that in due course the tips of both cones were discarded. The pathetic, and obviously mendacious, justification was that a boy concerned with turning his eyes from one cone to the other was more inclined to stumble over stones, steps, or cracks in the pavement. I dimly sensed that there was another secret justification, cruelly pedagogical, but I was unable to grasp it. Today, citizen and victim of a consumer society, a civilization of excess and waste (which the society of the thirties was not), I realize that those dear and now departed elders were right. Two two-cent cones instead of one at four cents did not signify squandering, economically speaking, but symbolically they surely did. It was for this precise reason, that I yearned for them: because two ice creams suggested excess. And this was precisely why they were denied to me: because they looked indecent, an insult to poverty, a display of fictitious privilege, a boast of wealth. Only spoiled children ate two cones at once, those children who in fairy tales were rightly punished, as Pinocchio was when he rejected the skin and the stalk. And parents who encouraged this weakness, appropriate to little parvenus, were bringing up their children in the foolish theatre of “I’d like to but I can’t.” They were preparing them to turn up at tourist-class check-in with a fake Gucci bag bought from a street peddler on the beach at Rimini. Nowadays the moralist risks seeming at odds with morality, in a world where the consumer civilization now wants even adults to be spoiled, and promises them always something more, from the wristwatch in the box of detergent to the bonus bangle sheathed, with the magazine it accompanies, in a plastic envelope. Like the parents of those ambidextrous gluttons I so envied, the consumer civilization pretends to give more, but actually gives, for four cents, what is worth four cents. You will throwaway the old transistor radio to purchase the new one, that boasts an alarm clock as well, but some inexplicable defect in the mechanism will guarantee that the radio lasts only a year. The new cheap car will have leather seats, double side mirrors adjustable from inside, and a panelled dashboard, but it will not last nearly so long as the glorious old Fiat 500, which, even when it broke down, could be started again with a kick. The morality of the old days made Spartans of us all, while today’s morality wants all of us to be Sybarites.

 

Q. 71 Which of the following cannot be inferred from the passage?

A. Today’s society is more extravagant than the society of the 1930s.

B. The act of eating two ice cream cones is akin to a ceremonial process.

C. Elders rightly suggested that a boy turning eyes from one cone to the other was more likely to fall.

D. Despite seeming to promise more, the consumer civilization gives away exactly what the thing is worth.

E. The consumer civilization attempts to spoil children and adults alike.

 

Q. 72 In the passage, the phrase “little parvenus” refers to

A. naughty midgets.

B. old hags.

C. arrogant people.

D. young upstarts.

E. foolish kids.

 

Q. 73 The author pined for two two-cent cones instead of one four-cent pie because

A. it made dietetic sense.

B. it suggested intemperance.

C. it was more fun.

D. it had a visual appeal.

E. he was a glutton.

 

Q. 74 What does the author mean by “nowadays the moralist risks seeming at odds with morality”?

A. The moralists of yesterday have become immoral today.

B. The concept of morality has changed over the years.

C. Consumerism is amoral.

D. The risks associated with immorality have gone up.

E. The purist’s view of morality is fast becoming popular.

 

Q. 75 According to the author, the justification for refusal to let him eat two cones was plausibly

A. didactic.

B. dietetic.

C. dialectic.

D. diatonic.

E. diastolic.

 

Questions: 76 – 80

The passage given below is followed by a set of five questions. Choose the most appropriate answer to each question.

Language is not a cultural artifact that we learn the way we learn to tell time or how the federal government works. Instead, it is a distinct piece of the biological makeup of our brains. Language is a complex, specialized skill, which develops in the child spontaneously, without conscious effort or formal instruction, is deployed without awareness of its underlying logic, is qualitatively the same in every individual, and is distinct from more general abilities to process information or behave intelligently. For these reasons some cognitive scientists have described language as a psychological faculty, a mental organ, a neural system, and a computational module. But I prefer the admittedly quaint term “instinct”. It conveys the idea that people know how to talk in more or less the sense that spiders know how to spin webs. Web-spinning was not invented by some unsung spider genius and does not depend on having had the right education or on having an aptitude for architecture or the construction trades. Rather, spiders spin spider webs because they have spider brains, which give them the urge to spin and the competence to succeed. Although there are differences between webs and words, I will encourage you to see language in this way, for it helps to make sense of the phenomena we will explore.  Thinking of language as an instinct inverts the popular wisdom, especially as it has been passed down in the canon of the humanities and social sciences. Language is no more a cultural invention than is upright posture. It is not a manifestation of a general capacity to use symbols: a three-year-old, we shall see, is a grammatical genius, but is quite incompetent at the visual arts, religious iconography, traffic signs, and the other staples of the semiotics curriculum. Though language is a magnificent ability unique to Homo sapiens among living species, it does not call for sequestering the study of humans from the domain of biology, for a magnificent ability unique to a particular living species is far from unique in the animal kingdom. Some kinds of bats home in on flying insects using Doppler sonar. Some kinds of migratory birds navigate thousands of miles by calibrating the positions of the constellations against the time of day and year. In nature’s talent show, we are simply a species of primate with our own act, a knack for communicating information about who did what to whom by modulating the sounds we make when we exhale. Once you begin to look at language not as the ineffable essence of human uniqueness hut as a biological adaptation to communicate information, it is no longer as tempting to see language as an insidious shaper of thought, and, we shall see, it is not. Moreover, seeing language as one of nature’s engineering marvels — an organ with “that perfection of structure and co-adaptation which justly excites our admiration,” in Darwin’s words – gives us a new respect for your ordinary Joe and the much-maligned English language (or any language). The complexity of language, from the scientist’s point of view, is part of our biological birthright; it is not something that parents teach their children or something that must be elaborated in school — as Oscar Wilde said, “Education is an admirable thing, but it is well to remember from time to time that nothing that is worth knowing can be taught.” A preschooler’s tacit knowledge of grammar is more sophisticated than the thickest style manual or the most state-of-the-art computer language system, and the same applies to all healthy human beings, even the notorious syntax fracturing professional athlete and the, you know, like, inarticulate teenage skateboarder. Finally, since language is the product of a well engineered biological instinct, we shall see that it is not the nutty barrel of monkeys that entertainer columnists make it out to be.

 

Q. 76 According to the passage, which of the following does not stem from popular wisdom on language?

A. Language is a cultural artifact.

B. Language is a cultural invention.

C. Language is learnt as we grow.

D. Language is unique to Homo sapiens.

E. Language is a psychological faculty.

 

Q. 77 Which of the following can be used to replace the “spiders know how to spin webs” analogy as used by the author?

A. A kitten learning to jump over a wall

B. Bees collecting nectar

C. A donkey carrying a load

D. A horse running a Derby

E. A pet clog protecting its owner’s property

 

Q. 78 According to the passage, which of the following is unique to human beings?

A. Ability to use symbols while communicating with one another.

B. Ability to communicate with each other through voice modulation.

C. Ability to communicate information to other members of the species.

D. Ability to use sound as means of communication.

E. All of the above.

 

Q. 79 According to the passage, complexity of language cannot be taught by parents or at school to children because

A. children instinctively know language.

B. children learn the language on their own.

C. language is not amenable to teaching.

D. children know language better than their teachers or parents.

E. children are born with the knowledge of semiotics.

 

Q. 80 Which of the following best summarizes the passage?

A. Language is unique to Homo sapiens.

B. Language is neither learnt nor taught.

C. Language is not a cultural invention or artifact as it is made out.

D. Language is instinctive ability of human beings.

E. Language is use of symbols unique to human beings.

 

Questions: 81 – 85

The passage given below is followed by a set of five questions. Choose the most appropriate answer to each question.

To summarize the Classic Maya collapse, we can tentatively identify five strands. I acknowledge, however, that Maya archaeologists still disagree vigorously among themselves in part, because the different strands evidently varied in importance among different parts of the Maya realm; because detailed archaeological studies are available for only some Maya sites; and because it remains puzzling why most of the Maya heartland remained nearly empty of population and failed to recover after the collapse and after re-growth of forests. With those caveats, it appears to me that one strand consisted of population growth outstripping available resources: a dilemma similar to the one foreseen by Thomas Malthus in 1798 and being played out today in Rwanda, Haiti and elsewhere. As the archaeologist David Webster succinctly puts it, “Too many farmers grew too many crops on too much of landscape.” Compounding that mismatch between population and resources was the second strand: the effects of deforestation and hillside erosion, which caused a decrease in the amount of useable farmland at a time when more rather than less farmland was needed, and possibly exacerbated by an anthropogenic drought resulting from deforestation, by soil nutrient depletion and other soil problems, and by the struggle to prevent bracken ferns from overrunning the fields. The third strand consisted of increased fighting, as more and more people fought over fewer resources. Maya warfare, already endemic, peaked just before the collapse. That is not surprising when one reflects that at least five million people, perhaps many more, were crammed into an area smaller than the US state of Colorado (104,000 square miles). That warfare would have decreased further the amount of land available for agriculture, by creating no-man’s lands between principalities where it was now unsafe to farm. Bringing matters to a head was the strand of climate change. The drought at the time of the Classic collapse was not the first drought that the Maya had lived through, but it was the most severe. At the time of previous droughts, there were still uninhabited parts of the Maya landscape, and people at a site affected by drought could save themselves by moving to another site. However, by the time of the Classic collapse the landscape was now full, there was no useful unoccupied land in the vicinity on which to begin anew, and the whole population could not be accommodated in the few areas that continued to have reliable water supplies. As our fifth strand, we have to wonder why the kings and nobles failed to recognize and solve these seemingly obvious problems undermining their society. Their attention was evidently focused on their short-term concerns of enriching themselves, waging wars, erecting monuments, competing with each other, and extracting enough food from the peasants to support all those activities. Like most leaders throughout human history, the Maya kings and nobles did not heed long-term problems, insofar as they perceived them. Finally, while we still have some other past societies to consider before we switch our attention to the modern world, we must already he struck by some parallels between the Maya and the past societies. As on Mangareva, the Maya environmental and population problems led to increasing warfare and civil strife. Similarly, on Easter Island and at Chaco Canyon, the Maya peak population numbers were followed swiftly by political and social collapse. Paralleling the eventual extension of agriculture from Easter Island’s coastal lowlands to its uplands, and from the Mimbres floodplain to the hills, Copan’s inhabitants also expanded from the floodplain to the more fragile hill slopes, leaving them with a larger population to feed when the agricultural boom in the hills went bust. Like Easter Island chiefs erecting ever larger statues, eventually crowned by pukao, and like Anasazi elite treating themselves to necklaces of 2,000 turquoise beads, Maya kings sought to outdo each other with more and more impressive temples, covered with thicker and thicker plaster — reminiscent in turn of the extravagant conspicuous consumption by modern American CEOs. The passivity of Easter chiefs and Maya kings in the face of the real big threats to their societies completes our list of disquieting parallels.

 

Q. 81 According to the passage, which of the following best represents the factor that has been cited by the author in the context of Rwanda and Haiti?

A. Various ethnic groups competing for land and other resources

B. Various ethnic groups competing for limited land resources

C. Various ethnic groups fighting with each other

D. Various ethnic groups competing for political power

E. Various ethnic groups fighting for their identity

 

Q. 82 By an anthropogenic drought, the author means

A. a drought caused by lack of rains.

B. a drought caused due to deforestation.

C. a drought caused by failure to prevent bracken ferns from overrunning the fields.

D. a drought caused by actions of human beings.

E. a drought caused by climate changes.

 

Q. 83 According to the passage, the drought at the time of Maya collapse had a different impact compared to the droughts earlier because

A. The Maya kings continued to be extravagant when common people were suffering.

B. it happened at the time of collapse of leadership among Mayas.

C. it happened when the Maya population had occupied all available land suited for agriculture.

D. it was followed by internecine warfare among Mayans.

E. irreversible environmental degradation led to this drought.

 

Q. 84 According to the author, why is it difficult to explain the reasons for Maya collapse?

A. Copan inhabitants destroyed all records of that period.

B. The constant deforestation and hillside erosion have wiped out all traces of the Maya kingdom.

C. Archaeological sites of Mayas do not provide any consistent evidence.

D. It has not been possible to ascertain which of the factors best explains as to why the Maya civilization collapsed.

E. At least five million people were crammed into a small area.

 

Q. 85 Which factor has not been cited as one of the factors causing the collapse of Maya society?

A. Environmental degradation due to excess population

B. Social collapse due to excess population

C. Increased warfare among Maya people

D. Climate change

E. Obsession of Maya population with their own short-term concerns

 

Questions: 86 – 90

The passage given below is followed by a set of five questions. Choose the most appropriate answer to each question.

A remarkable aspect of art of the present century is the range of concepts and ideologies which it embodies. It is almost tempting to see a pattern emerging within the art field – or alternatively imposed upon it a posteriori – similar to that which exists under the umbrella of science where the general term covers a whole range of separate, though interconnecting, activities. Any parallelism is however – in this instance at least – misleading. A scientific discipline develops systematically once its bare tenets have been established, named and categorized as conventions. Many of the concepts of modern art, by contrast, have resulted from the almost accidental meetings of groups of talented individuals at certain times and certain places. The ideas generated by these chance meetings had twofold consequences. Firstly, a corpus of work would be produced which, in great part, remains as a concrete record of the events. Secondly, the ideas would themselves be disseminated through many different channels of communication – seeds that often bore fruit in contexts far removed from their generation. Not all movements were exclusively concerned with innovation. Surrealism, for instance, claimed to embody a kind of insight which can be present in the art of any period. This claim has been generally accepted so that a sixteenth century painting by Spranger or a mysterious photograph by Atget can legitimately be discussed in surrealist terms. Briefly, then, the concepts of modern art are of many different (often fundamentally different) kinds and resulted from the exposures of painters, sculptors and thinkers to the more complex phenomena of the twentieth century, including our ever increasing knowledge of the thought and products of earlier centuries. Different groups of artists would collaborate in trying to make sense of a rapidly changing world of visual and spiritual experience. We should hardly be surprised if no one group succeeded completely, but achievements, though relative, have been considerable. Landmarks have been established – concrete statements of position which give a pattern to a situation which could easily have degenerated into total chaos. Beyond this, new language tools have been created for those who follow – semantic systems which can provide a springboard for further explorations. The codifying of art is often criticized. Certainly one can understand that artists are wary of being pigeonholed since they are apt to think of themselves as individuals – sometimes with good reason. The notion of self-expression, however, no longer carries quite the weight it once did; objectivity has its defenders. There is good reason to accept the ideas codified by artists and critics, over the past sixty years or so, as having attained the status of independent existence – an independence which is not without its own value. The time factor is important here. As an art movement slips into temporal perspective, it ceases to be a living organism – becoming, rather, a fossil. This is not to say that it becomes useless or uninteresting. Just as a scientist can reconstruct the life of a prehistoric environment from the messages codified into the structure of a fossil, so can an artist decipher whole webs of intellectual and creative possibility from the recorded structure of a ‘dead’ art movement. The artist can match the creative patterns crystallized into this structure against the potentials and possibilities of his own time. As T.S. Eliot observed, no one starts anything from scratch; however consciously you may try to live in the present, you are still involved with a nexus of behaviour patterns bequeathed from the past. The original and creative person is not someone who ignores these patterns, but someone who is able to translate and develop them so that they conform more exactly to his – and our – present needs.

 

Q. 86 Many of the concepts of modern art have been the product of

A. ideas generated from planned deliberations between artists, painters and thinkers.

B. the dissemination of ideas through the state and its organizations.

C. accidental interactions among people blessed with creative muse.

D. patronage by the rich and powerful that supported art.

E. systematic investigation, codification and conventions.

 

Q. 87 In the passage, the word ‘fossil’ can be interpreted as

A. an art movement that has ceased to remain interesting or useful.

B. an analogy from the physical world to indicate a historic art movement.

C. an analogy from the physical world to indicate the barrenness of artistic creations in the past.

D. an embedded codification of pre-historic life.

E. an analogy from the physical world to indicate the passing of an era associated with an art movement.

 

Q. 88 In the passage, which of the following similarities between science and art may lead to erroneous conclusions?

A. Both, in general, include a gamut of distinct but interconnecting activities.

B. Both have movements not necessarily concerned with innovation.

C. Both depend on collaborations between talented individuals.

D. Both involve abstract thought and dissemination of ideas.

E. Both reflect complex priorities of the modern world.

 

Q. 89 The range of concepts and ideologies embodied in the art of the twentieth century is explained by

A. the existence of movements such as surrealism.

B. landmarks which give a pattern to the art history of the twentieth century.

C. new language tools which can be used for further explorations into new areas.

D. the fast changing world of perceptual and transcendental understanding.

E. the quick exchange of ideas and concepts enabled by efficient technology.

 

Q. 90 The passage uses an observation by T.S. Eliot to imply that

A. creative processes are not ‘original’ because they always borrow from the past.

B. we always carry forward the legacy of the past.

C. past behaviours and thought processes recreate themselves in the present and get labeled as ‘original’ or ‘creative’.

D. ‘originality’ can only thrive in a ‘greenhouse’ insulated from the past biases.

E. ‘innovations’ and ‘original thinking’ interpret and develop on past thoughts to suit contemporary needs.

 

 

Answer Sheet
Question 1 2 3 4 5 6 7 8 9 10
Answer C C B B B E C D D A
Question 11 12 13 14 15 16 17 18 19 20
Answer B E E C E A D C D D
Question 21 22 23 24 25 26 27 28 29 30
Answer E B A A A E D B C E
Question 31 32 33 34 35 36 37 38 39 40
Answer D C B C E E B E D C
Question 41 42 43 44 45 46 47 48 49 50
Answer A A E E A A D C C B
Question 51 52 53 54 55 56 57 58 59 60
Answer A E D E C B B A D D
Question 61 62 63 64 65 66 67 68 69 70
Answer B E C D B D B D B E
Question 71 72 73 74 75 76 77 78 79 80
Answer C D B B A E B B A D
Question 81 82 83 84 85 86 87 88 89 90
Answer B D C D E C E A D E

CAT Previous Year Paper 2007

CAT 2007

Section

Questions

Marks

Quantitative Aptitude

25 Questions (1 – 25)

75

Data Interpretation

21 Questions

63

Logical Reasoning

4 Questions (30 – 33)

12

Reading Comprehension

12 Questions

36

Verbal Ability

13 Questions

39

Q. 1 Consider the set S = {2, 3, 4, ……, 2n+1}, where ‘n’ is a positive integer larger than 2007. Define X as the average of the odd integers in S and Y as the average of the even integers in S. What is the value of X – Y?

A. 0

B. 1

C. (1/2)n

D. (n+1)/2n

E. 2008

 

Q. 2 Ten years ago, the ages of the members of a joint family of eight people added up to 231 years. Three years later, one member died at the age of 60 years and a child was born during the same year. After another three years, one more member died, again at 60, and a child was born during the same year. The current average age of this eight-member joint family is nearest to

A. 23 years

B. 22 years

C. 21 years

D. 25 years

E. 24 years

 

Q. 3 A function f(x) satisfies f(1) = 3600 and f(1) + f(2) + ………. + f(n) = n^2 f(n), for all positive integers n > 1. What is the value of f(9)?

A. 80

B. 240

C. 200

D. 100

E. 120

 

Q. 4 Suppose you have a currency, named Miso, in three denominations: 1 Miso, 10 Misos and 50 Misos. In how many ways can you pay a bill of 107 Misos?

A. 17

B. 16

C. 18

D. 15

E. 19

 

Q. 5 A confused bank teller transposed the rupees and paise when he cashed a cheque for Shailaja giving her rupees instead of paise and paise instead of rupees. After buying a toffee for 50 paise, Shailaja noticed that she was left with exactly three times as much as the amount on the cheque. Which of the following is a valid statement about the cheque amount?

A. Over Rupees 13 but less than Rupees 14

B. Over Rupees 7 but less than Rupees 8

C. Over Rupees 22 but less than Rupees 23

D. Over Rupees 18 but less than Rupees 19

E. Over Rupees 4 but less than Rupees 5

 

Q. 6 How many pairs of positive integers m, n satisfy 1/m +4/n = 1/12, where, ‘n’ is an odd integer less than 60?

A. 6

B. 4

C. 7

D. 5

E. 3

 

Questions: 7 – 10

Each question is followed by two statements A and B. Indicate your response based on the following directives.

Mark (1) if the questions can be answered using A alone but not using B alone. Mark (2) if the question can be answered using B alone but not using A alone. Mark (3) if the question can be answered using A and B together, but not using either A or B alone. Mark (4) if the question cannot be answered even using A and B together.

 

Q. 7 The average weight of a class of 100 students is 45 kg. The class consists of two sections, I and II, each with 50 students. The average weight, WI , of Section I is smaller than the average weight WII , of the Section II. If the heaviest student say Deepak, of section II is moved to Section I, and the lightest student, say Poonam, of Section I is moved to Section II, then the average weights of the two sections are switched, i.e., the average weight of Section I becomes WII and that of Section II becomes WI . What is the weight of Poonam?

A: WII – WI = 1.0 .

B: Moving Deepak from Section II to I (without any move I to II) makes the average weights of the two sections equal.

A. (1)

B. (2)

C. (3)

D. (4)

 

Q. 8 ABC Corporation is required to maintain at least 400 Kilolitres of water at all times in its factory, in order to meet safety and regulatory requirements. ABC is considering the suitability of a spherical tank with uniform wall thickness for the purpose. The outer diameter of the tank is 10 meters. Is the tank capacity adequate to met ABC’s requirements?

A: The inner diameter of the tank is at least 8 meters.

B: The tank weights 30,000 kg when empty, and is made of a material with density of 3 gm/cc.

A. (1)

B. (2)

C. (3)

D. (4)

 

Q. 9 Consider integers x, y, z. What is the minimum possible value of x^2 + y^2 + z^2 = ?

A: x + y + z = 89.

B: Among x, y, z two are equal.

A. (1)

B. (2)

C. (3)

D. (4)

 

Q. 10 Rahim plans to draw a square JKLM with point O on the side JK but is not successful. Why is Rahim unable to draw the square?

A: The length of OM is twice that of OL.

B: The length of OM is 4 cm.

A. (1)

B. (2)

C. (3)

D. (4)

 

Questions: 11 – 12

Cities A and B are in different time zones. A is located 3000 km east of B. The table given in figure (1) describes the schedule of an airline operating non-stop flights between A and B. All the times indicated are local and on the same day. Assume that planes cruise at the same speed in both directions. However, the effective speed is influenced by a steady wind blowing from east to west at 50 km per hour.

 

Q. 11 What is the time difference between A and B?

A. 1 hour and 30 minutes

B. 2 hours

C. 2 hours and 30 minutes

D. 1 hour

E. Cannot be determined

 

Q. 12 What is the plane’s cruising speed in km per hour?

A. 700

B. 550

C. 600

D. 500

E. Cannot be determined.

 

Questions: 13 – 14

Shabnam is considering three alternatives to invest her surplus cash for a week. She wishes to guarantee maximum returns on her investment. She has three options, each of which can be utilized fully or partially in conjunction with others

 

Q. 13 The maximum guaranteed return to Shabnam is

A. 0.25%

B. 0.10%

C. 0.20%

D. 0.15%

E. 0.30%

 

Q. 14 What strategy will maximize the guaranteed return to Shabnam?

A. 100% in option A

B. 36% in option B and 64% in option C

C. 64% in option B and 36% in option C

D. 1/3 in each of the three options

E. 30% in option A, 32% in option B and 38% in option C

 

Questions: 15 – 16

Let S be the set of all pairs (i, j) where, 1 ≤ i < j ≤ n and n ≥ 4 . Any two distinct members of S are called “friends” if they have one constituent of the pairs in common and “enemies” otherwise. For example, if n = 4, then S = {(1, 2), (1, 3), (1, 4), (2, 3), (2, 4), (3, 4)}. Here, (1, 2) and (1, 3) are friends, (1, 2) and (2, 3) are also friends, but (1, 4) and (2, 3) are enemies.

 

Q. 15 For general ‘n’, how many enemies will each member of S have?

A. n – 3

B. 1/2 (n^2 – 3n – 2)

C. 2n – 7

D. 1/4 (n^2 – 5n + 6)

E. 1/2 (n^2 – 7n + 14)

 

Q. 16 For general ‘n’, consider any two members of S that are friends. How many other members of S will be common friends of both these members?

A. 1/2 (n^2 – 5n + 8)

B. 2n – 6

C. 1/2(n)(n – 3)

D. n – 2

E. 1/2 (n^2 – 7n + 14)

 

Q. 17 In a tournament, there are n teams T1, T2, …., Tn, with n > 5. Each team consists of ‘k’ players, k > 3. The following pairs of teams have one player in common: T1 & T2, T2 & T3, ……….., Tn-1 & Tn, and Tn & T1.

No other pair of teams has any player in common. How many players are participating in the tournament, considering all the ‘n’ teams together?

A. n(k-1)

B. k(n-1)

C. n(k-2)

D. k(n-2)

E. (n-1)(k-1)

 

Q. 18 Consider four-digit numbers for which the first two digits are equal and the last two digits are also equal. How many such numbers are perfect squares?

A. 3

B. 2

C. 4

D. 0

E. 1

 

Questions: 19 – 20

Mr. David manufactures and sells a single product at a fixed price in a niche market. The selling price of each unit is Rs. 30. On the other hand, the cost, in rupees, of producing ‘x’ units is 240 + bx + cx^2, where ‘b’ and ‘c’ are some constants. Mr. David noticed that doubling the daily production from 20 to 40 units increases the daily production cost by 66 2/3%. However, an increase in daily production from 40 to 60 units results in an increase of only 50% in the daily production cost. Assume that demand is unlimited and that Mr. David can sell as much as he can produce. His objective is to maximize the profit.

 

Q. 19 How many units should Mr. David produce daily?

A. 130

B. 100

C. 70

D. 150

E. Cannot be determined

 

Q. 20 What is the maximum daily profit, in rupees, that Mr. David can realize from his business?

A. 620

B. 920

C. 840

D. 760

E. Cannot be determined

 

Q. 21 The price of Darjeeling tea (in rupees per kilogram) is 100 + 0.10n, on the nth day of 2007 (n = 1, 2, …, 100), and then remains constant. On the other hand, the price of Ooty tea (in rupees per kilogram) is 89 + 0.15n, on the nth day of 2007 (n = 1, 2, …, 365). On which date in 2007 will the prices of these two varieties of tea be equal?

A. May 21

B. April 11

C. May 20

D. April 10

E. June 30

 

Q. 22 Two circles with centres P and Q cut each other at two distinct points A and B. The circles have the same radii and neither P nor Q falls within the intersection of the circles. What is the smallest range that includes all possible values of the angle AQP in degrees?

A. Between 0 and 90

B. Between 0 and 30

C. Between 0 and 60

D. Between 0 and 75

E. Between 0 and 45

 

Q. 23 A quadratic function ƒ(x) attains a maximum of 3 at x = 1. The value of the function at x = 0 is 1. What is the value ƒ(x) at x = 10?

A. –119

B. –159

C. –110

D. –180

E. –105

 

Questions: 24 – 25

Let a1 = p and b1 = q, where p and q are positive quantities. Define a n = pb n–1, b n = qb n–1, for even n > 1, and a n = pa n-1, b n = qa n-1, for odd n > 1.

 

Q. 24 Which of the following best describes an + bn for even ‘n’?

A. q(pq)^1/2(n-1) (p + q)

B. qp^1/2(n-1) (p + q)

C. q^1/2(n) (p + q)

D. q^1/2(n) (p + q)^1/2(n)

E. q(pq)^1/2(n-1) (p + q)^1/2(n)

 

Q. 25 If p = 1/3 and q = 2/3, then what is the smallest odd ‘n’ such that an + bn < 0.01

A. 7

B. 13

C. 11

D. 9

E. 15

 

Questions: 26 – 29

A health-drink company’s R&D department is trying to make various diet formulations, which can be used for certain specific purposes. It is considering a choice of 5 alternative ingredients (O, P, Q, R, and S), which can be used in different proportions in the formulations. The table given in figure (1) gives the composition of these ingredients. The cost per unit of each of these ingredients is O: 150, P: 50. Q: 200, R: 500, S: 100. .

 

Q. 26 For a recuperating patient, the doctor recommended a diet containing 10% minerals and at least 30% protein. In how many different ways can we prepare this diet by mixing at least two ingredients?

A. One

B. Two

C. Three

D. Four

E. None

 

Q. 27 Which among the following is the formulation having the lowest cost per unit for a diet having 10% fat and at least 30% protein? (The diet has to be formed by mixing two ingredients).

A. P and Q

B. P and S

C. P and R

D. Q and S

E. R and S

 

Q. 28 In what proportion P, Q and S should be mixed to make a diet having at least 60% carbohydrate at the lowest cost per unit?

A. 2:1:3

B. 4:1:2

C. 2:1:4

D. 3:1:2

E. 4:1:1

 

Q. 29 The company is planning to launch a balanced diet required for growth needs of adolescent children. This diet must contain at least 30% each of carbohydrate and protein, no more than 25% fat and at least 5% minerals. Which one of the following combinations of equally mixed ingredients is feasible?

A. O and P

B. R and S

C. P and S

D. Q and R

E. O and S

 

Questions: 30 – 33

Each question is followed by two statements, A and B. Answer each question using the following instructions:

Mark (1) if the question can be answered by using the statement A alone but not by using the statement B alone.

Mark (2) if the question can be answered by using the statement B alone but not by using the statement A alone.

Mark (3) if the question can be answered by using either of the statements alone.

Mark (4) if the question can be answered by using both the statements together but not by either of the statements alone.

Mark (5) if the question cannot be answered on the basis of the two statements.

 

Q. 30 In a particular school, sixty students were athletes. Ten among them were also among the top academic performers. How many top academic performers were in the school?

A. Sixty per cent of the top academic performers were not athletes.

B. All the top academic performers were not necessarily athletes.

A. (1)

B. (2)

C. (3)

D. (4)

E. (5)

 

Q. 31 Five students Atul, Bala, Chetan, Dev and Ernesto were the only ones who participated in a quiz contest. They were ranked based on their scores in the contest. Dev got a higher rank as compared to Ernesto, while Bala got a higher rank as compared to Chetan. Chetan’s rank was lower than the median. Who among the five got the highest rank?

A. Atul was the last rank holder.

B. Bala was not among the top two rank holders.

A. (1)

B. (2)

C. (3)

D. (4)

E. (5)

 

Q. 32 Thirty per cent of the employees of a call centre are males. Ten per cent of the female employees have an engineering background. What is the percentage of male employees with engineering background?

A. Twenty five per cent of the employees have engineering background.

B. Number of male employees having an engineering background is 20% more than the number of female employees having an engineering background.

A. (1)

B. (2)

C. (3)

D. (4)

E. (5)

 

Q. 33 ln a football match, at the half-time, Mahindra and Mahindra Club was trailing by three goals. Did it win the match?

A. In the second-half Mahindra and Mahindra Club scored four goals.

B. The opponent scored four goals in the match.

A. (1)

B. (2)

C. (3)

D. (4)

E. (5)

 

Questions: 34 – 37

The following table given in figiure (1) shows the break-up of actual costs incurred by a company in last five years (year 2002to year 2006) to produce a particular product.

The production capacity of the company is 2000 units. The selling price for the year 2006 was Rs. 125 per unit. Some costs change almost in direct proportion to the change in volume of production, while others do not follow any obvious pattern of change with respect to the volume of production and hence are considered fixed. Using the information provided for the year 2006 as the basis for projecting the figures for the year 2007, answer the following questions:

 

Q. 34 What is the approximate cost per unit in rupees, if the company produces and sells 1400 units in the year 2007?

A. 104

B. 107

C. 110

D. 115

E. 116

 

Q. 35 What is the minimum number of units that the company needs to produce and sell to avoid any loss?

A. 313

B. 350

C. 384

D. 747

E. 928

 

Q. 36 If the company reduces the price by 5%, it can produce and sell as many units as it desires. How many units the company should produce to maximize its profit?

A. 1400

B. 1600

C. 1800

D. 1900

E. 2000

 

Q. 37 Given that the company cannot sell more than 1700 units, and it will have to reduce the price by Rs.5 for all units, if it wants to sell more than 1400 units, what is the maximum profit, in rupees, that the company can earn?

A. 25,400

B. 24,400

C. 31,400

D. 32,900

E. 32,000

 

Questions: 38 – 41

The proportion of male students and the proportion of vegetarian students in a school are given in figure (1). The school has a total of 800 students, 80% of whom are in the Secondary Section and rest are equally divided between Class 11 and 12.

 

Q. 38 What is the percentage of male students in the secondary section?

A. 40

B. 45

C. 50

D. 55

E. 60

 

Q. 39 In Class 12, twenty five per cent of the vegetarians are male. What is the difference between the number of female vegetarians and male non-vegetarians?

A. less than 8

B. 10

C. 12

D. 14

E. 16

 

Q. 40 What is the percentage of vegetarian students in Class 12?

A. 40

B. 45

C. 50

D. 55

E. 60

 

Q. 41 In the Secondary Section, 50% of the students are vegetarian males. Which of the following statements is correct?

A. Except vegetarian males, all other groups have same number of students.

B. Except non-vegetarian males, all other groups have same number of students.

C. Except vegetarian females, all other groups have same number of students.

D. Except non-vegetarian females, all other groups have same number of students.

E. All of the above groups have the same number of students

 

Questions: 42 – 45

The Table given in figure (1) shows the comparative costs, in US Dollars, of major surgeries in USA and a select few Asian countries.

The equivalent of one US Dollar in the local currencies is given in figure (2): A consulting firm found that the quality of the health services were not the same in all the countries above. A poor quality of a surgery may have significant repercussions in future, resulting in more cost in correcting mistakes. The cost of poor quality of surgery is given in the table in figure (3):

 

Q. 42  A US citizen is hurt in an accident and requires an angioplasty, hip replacement and a knee replacement. Cost of foreign travel and stay is not a consideration since the government will take care of it. Which country will result in the cheapest package, taking cost of poor quality into account?

A. India

B. Thailand

C. Malaysia

D. Singapore

E. USA

 

Q. 43 Taking the cost of poor quality into account, which country/countries will be the most expensive for knee replacement?

A. India

B. Thailand

C. Malaysia

D. Singapore

E. India and Singapore

 

Q. 44 Approximately, what difference in amount in Bahts will it make to a Thai citizen if she were to get a hysterectomy done in India instead of in her native country, taking into account the cost of poor quality? (It costs 7500 Bahts for one-way travel between Thailand and India).

A. 23500

B. 40500

C. 57500

D. 67500

E. 75000

 

Q. 45 The rupee value increases to Rs.35 for a US Dollar, and all other things including quality, remain the same. What is the approximate difference in cost, in US Dollars, between Singapore and India for a Spinal Fusion, taking this change into account?

A. 700

B. 2500

C. 4500

D. 8000

E. No difference

 

Questions: 46 – 50

A low-cost airline company connects ten Indian cities, A to J. The table given in figure (1) shows the distance between a pair of airports and the corresponding price charged by the company. Travel is permitted only from a departure airport to an arrival airport. The customers do not travel by a route where they have to stop at more than two intermediate airports.

 

Q. 46  What is the lowest price, in rupees, a passenger has to pay for travelling by the shortest route from A to J?

A. 2275

B. 2850

C. 2890

D. 2930

E. 3340

 

Q. 47 The company plans to introduce a direct flight between A and J. The market research results indicate that all its existing passengers travelling between A and J will use this direct flight if it is priced 5% below the minimum price that they pay at present. What should the company charge approximately, in rupees, for this direct flight?

A. 1991

B. 2161

C. 2707

D. 2745

E. 2783

 

Q. 48 If the airports C, D and H are closed down owing to security reasons, then what would be the minimum price, in rupees, to be paid by a passenger travelling from A to J?

A. 2275

B. 2615

C. 2850

D. 2945

E. 3190

 

Q. 49 If the prices include a margin of 10% over the total cost that the company incurs, then what is the minimum cost per kilometer that the company incurs in flying from A to J?

A. 0.77

B. 0.88

C. 0.99

D. 1.06

E. 1.08

 

Q. 50 If the prices include a margin of 15% over the total cost that the company incurs, then which among the following is the distance to be covered in flying from A to J that minimizes the total cost per kilometer for the company?

A. 2170

B. 2180

C. 2315

D. 2350

E. 2390

 

Questions: 51 – 53

Human Biology does nothing to structure human society: age may enfeeble us all, but cultures vary considerably in the prestige and power they accord to the elderly. Giving birth is a necessary condition for being a mother, but it is not sufficient. We expect mothers to behave in maternal ways and to display appropriately maternal sentiments. We prescribe a clutch of norms or rules that govern the role of a mother. That the social role is independent of the biological base can be demonstrated by going back three sentences. (giving birth is certainly not sufficient to be a mother but, as adoption and fostering show, it is not even necessary!

The fine detail of what is expected of a mother or a father or a dutiful son differs from culture to culture, but everywhere behaviour is coordinated by the reciprocal nature of roles. Husbands and wives, parents and children, employers and employees, waiters and customers, teachers and pupils, warlords and followers: each makes sense only in its relation to the other. The term ‘role’ is an appropriate one. because the metaphor of an actor in a play neatly expresses the rule-governed nature or scripted nature of much of social life and the sense that society is a joint production. Social life occurs only because people play their parts (and that is as true for war and conflicts as for peace and love) and those parts make sense only in the context of the overall show. The drama metaphor also reminds us of the artistic licence available to the players. We can play a part straight or, as the following from J.P. Sartre conveys, we can ham it up. Let us consider this waiter in the café. His movement is quick and forward, a little too precise, a little too rapid. He comes towards the patrons with a step a little too quick. He bends forward a little too eagerly: his voice, his eyes express an interest a little too solicitous for the order of the customer. Finally there he

returns, trying to imitate in his walk the inflexible stiffness of some kind of automation while carrying his tray with the recklessness of a tightropewalker…..All his behaviour seems to us a game….But what is he playing? We need not watch long before we can explain it: he is playing at being a waiter in a café.

The American sociologist Frying Goffman built an influential body of social analysis on elaborations of themetaphor of social life as drama. Perhaps his most telling point was that it is only through acting out a part that we express character. It is not enough to be evil or virtuous: we have to be seen to be evil or virtuous.

There is distinction between the roles we play and some underlying self. Here we might note that some roles are more absorbing than others. We would not be surprised by the waitress who plays the part in such a way as to signal to us that she is much more than her occupation. We would be surprised and offended by the father who played his part ‘tongue in cheek’. Some roles are broader and more far-reaching than others. Describing someone as a clergyman or faith healer would say far more about that person than describing someone as a bus driver.

 

Q. 51 What is the thematic highlight of the passage?

A. In the absence of strong biological linkages, reciprocal roles provide the mechanism for coordinating human behaviour.

B. In the absence of reciprocal roles, biological linkages provide the mechanism for coordinating human behaviour

C. Human behaviour is independent of biological linkages and reciprocal roles.

D. Human behaviour depends on biological linkages and reciprocal roles.

E. Reciprocal roles determine normative human behaviour in society.

 

Q. 52 Which of the following would have been true if biological linkages structured human society?

A. The role of mother would have been defined through her reciprocal relationship with her children.

B. We would not have been offended by the father playing his role ‘tongue in cheek’.

C. Women would have adopted and fostered children rather than giving birth to them.

D. Even if warlords were physically weaker than their followers, they would still dominate them.

E. Waiters would have stronger motivation to serve their customers.

 

Q. 53 It has been claimed in the passage that “some roles are more absorbing than others”. According to the passage, which of the following seem(s) appropriate reason(s) for such a claim?

A. Some roles carry great expectations from the society preventing manifestation of the true self.

B. Society ascribes so much importance to some roles that the conception of self may get aligned with the roles being performed.

C. Some roles require development of skill and expertise leaving little time for manifestation of self.

A. A only

B. B only

C. C only

D. A and B

E. B and C

 

Questions: 54 – 56

In each question, there are five sentences or parts of sentences that form a paragraph. Identify the sentence(s) or part(s) of sentence(s) that is/are correct in terms of grammar and usage.

 

Q. 54 A. When I returned to home, I began to read.

B. everything I could get my hand on about Israel.

C. That same year Israel’s Jewish Agency sent

D. a Shaliach a sort of recruiter to Minneapolis.

E. I became one of his most active devotees.

A. C and E

B. C only

C. E only

D. B, C and E

E. C, D and E

 

Q. 55 A. So once an economy is actually in a recession,

B. he authorities can, in principle, move the economy

C. out of slump – assuming hypothetically.

D. that they know how to – by a temporary stimuli.

E. In the longer term, however, such polices have no affect on the overall behaviour of the economy.

A. A, B & E

B. B, C & E

C. C & D

D. E only

E. B only

 

Q. 56 A. It is sometimes told that democratic

B. government originated in the city-states

C. of ancient Greece. Democratic ideals have been handed to us from that time.

D. In truth, however, this is an unhelpful assertion.

E. The Greeks gave us the word, hence did not provide us with a model.

A. A, B & D

B. B, C & D

C. B & D

D. B only

E. D only

 

Questions: 57 – 59

Every civilized society lives and thrives on a silent but profound agreement as to what is to be accepted as the valid mould of experience. Civilization is a complex system of dams, dykes. and canals warding off, directing, and articulating the influx of the surrounding fluid element: a fertile fenland, elaborately drained and protected from the high tides of chaotic, unexercised, and inarticulate experience. In such a culture, stable and sure of itself within the frontiers of ‘naturalized’ experience, the arts wield their creative power not so much in width as in depth. They do not create new experience, but deepen and purify the old. Their works do not differ from one another like a new horizon from a new horizon, but like a madonna from a Madonna.

The periods of art which are most vigorous in creative passion seem to occur when the established pattern of experience loosens its rigidity without as yet losing its force. Such a period was the Renaissance, and Shakespeare its poetic consummation. Then it was as though the discipline of the old order gave depth to the excitement of the breaking away, the depth of job and tragedy, of incomparable conquests and irredeemable losses. Adventurers of experience set out as though in lifeboats to rescue and bring back to the shore treasures of knowing and feeling which the old order had left floating on the high seas. The works of the early Renaissance and the poetry of Shakespeare vibrate with the compassion for live experience in danger of dying from exposure and neglect. In this compassion was the creative genius of the age. Yet, it was a genius of courage, not of desperate audacity. For, however elusively, it still knew of harbours and anchors, of homes to which to return, and of barns in which to store the harvest. The exploring spirit of art was in the depths of its consciousness still aware of a scheme of things into which to fit its exploits and creations. But the more this scheme of things loses its stability, the more boundless and uncharted appears the ocean of potential exploration. In the blank confusion of infinite potentialities flotsam of significance gets attached to jetsam of experience: for everything is sea, everything is at sea-

…The sea is all about us;

The sea is the land’s edge also, the granite

Into which it reaches, the beaches where it tosses

Its hints of earlier and other creation…

– and Rilke tells a story in which, as in T.S. Eliot’s poem, it is again the sea and the distance of ‘other creation’ that becomes the image of the poet’s reality. A rowing boat sets out on a difficult passage. The oarsmen labour in exact rhythm. There is no sign yet of the destination. Suddenly a man. seemingly idle, breaks out into song. And if the labour of the oarsmen meaninglessly defeats the real resistance of the real waves, it is the idle single who magically conquers the despair of apparent aimlessness. While the people next to him try to come to grips with the element that is next to them, his voice seems to bind the boat to the farthest distance so that the farthest distance draws it towards itself. ‘I don’t know why and how,’ is Rilke’s conclusion, ‘but suddenly I understood the situation of the poet, his place and function in this age. It does not matter if one denies him every place — except this one. There one must tolerate him.’

 

Q. 57 In the passage, the expression “like a madonna from a madonna” alludes to

A. The difference arising as a consequence of artistic license.

B. The difference between two artistic interpretations.

C. The difference between ‘life’ and ‘interpretation of life’.

D. The difference between ‘width’ and ‘depth’ of creative power.

E. The difference between the legendary character and the modern day singer.

 

Q. 58 The sea and ‘other creation’ leads Rilke to

A. Define the place of the poet in his culture.

B. Reflect on the role of the oarsman and the singer.

C. Muse on artistic labour and its aimlessness.

D. Understand the elements that one has to deal with.

E. Delve into natural experience and real waves.

 

Q. 59 According to the passage, the term “adventurers of experience” refers to

A. Poets and artists who are driven by courage.

B. Poets and artists who create their own genre.

C. Poets and artists of the Renaissance.

D. Poets and artists who revitalize and enrich the past for us.

E. Poets and artists who delve in flotsam and jetsam in sea.

 

Q. 60 Each of the following questions has a paragraph from which the last sentence has been deleted. From the given options, choose the sentence that completes the paragraph in the most appropriate way.

Characters are also part of deep structure. Characters tie events in a story together and provide a thread of continuity and meaning. Stories can be about individuals, groups, projects, or whole organizations, so from an organizational studies perspective, the focal actor(s) determine the level and unit of analysis used in a study. Stories of mergers and acquisitions, for example, are commonplace. In these stories whole organizations are personified as actors. But these macrolevel stories usually are not told from the perspective of the macro-level participants, because whole organizations cannot narrate their experiences in the first person.

A. More generally, data concerning the identities and relationships of the characters in the story are required, if one is to understand role structure and social networks in which that process is embedded.

B. Personification of a whole organization abstracts away from the particular actors and from traditional notions of level of analysis.

C. The personification of a whole organization is important because stories differ depending on who is enacting various events.

D. Every story is told from a particular point of view, with a particular narrative voice, which is not regarded as part of the deep structure.

E. The personification of a whole organization is a textual device we use to make macrolevel theories more comprehensible. 61. Nevertheless, photographs still retain

 

Q. 61 Each of the following questions has a paragraph from which the last sentence has been deleted. From the given options, choose the sentence that completes the paragraph in the most appropriate way.

Nevertheless, photographs still retain some of the magical allure that the earliest daguerreotypes inspired. As objects, our photographs have changed; they have become physically flimsier as they have become more technologically sophisticated. Daguerre produced pictures on copper plates: today many of our photographs never become tangible things, but instead remain filed away on computers and cameras, part of the digital ether that envelops the modern world. At the same time, our patience for the creation of images has also eroded. Children today are used to being tracked from birth by digital cameras and video recorders and they expect to see the results of their poses and performances instantly. The space between life as it is being lived and life as it is being displayed shrinks to a mere second.

A. Yet, despite these technical developments, photographs still remain powerful because they are reminders of the people and things we care about.

B. Images, after all, are surrogates carried into battle by a soldier or by a traveller on holiday.

C. Photographs, be they digital or traditional, exist to remind us of the absent, the beloved, and the dead.

D. In the new era of the digital image, the images also have a greater potential for fostering falsehood and trickery, perpetuating fictions that seem so real we cannot tell the difference.

E. Anyway, human nature being what it is, little time has passed after photography’s invention became means of living life through images.

 

Q. 62 Each of the following questions has a paragraph from which the last sentence has been deleted. From the given options, choose the sentence that completes the paragraph in the most appropriate way.

Mma Ramotswe had a detective agency in Africa, at the foot of Kgale Hill. These were its assets: a tiny white van, two desks, two chairs, a telephone, and an old typewriter. Then there was a teapot, in which Mma Ramotswe – the only private lady detective in Botswana – brewed redbush tea. And three mugs – one for herself, one for her secretary, and one for the client. What else does a detective agency really need? Detective agencies rely on human intuition and intelligence, both of which Mma Ramotswe had in abundance.

A. But there was also the view, which again would appear on no inventory.

B. No inventory would ever include those, of course.

C. She had an intelligent secretary too.

D. She was a good detective and a good woman.

E. What she lacked in possessions was more than made up by a natural shrewdness.

 

Questions: 63 – 65

To discover the relation between rules, paradigms, and normal science, consider first how the historian isolates the particular loci of commitment that have been described as accepted rules. Close historical investigation of a given specialty at a given time discloses a set of recurrent and quasi-standard illustrations of various theories in their conceptual, observational, and instrumental applications. These are the community’s paradigms, revealed in its textbooks, lectures, and laboratory exercises. By studying them and by practicing with them, the members of the corresponding community learn their trade. The historian, of course, will discover in addition a penumbral area occupied by achievements whose status is still in doubt, but the core of solved problems and techniques will usually be clear. Despite occasional ambiguities, the paradigms of a mature scientific community can be determined with relative ease. That demands a second step and one of a somewhat different kind. When undertaking it, the historian must compare the community’s paradigms with each other and with its current research reports. In doing so, his object is to discover what isolable elements, explicit or implicit, the members of that community may have abstracted from their more global paradigms and deploy it as rules in their research. Anyone who has attempted to describe or analyze the evolution of a particular scientific tradition will necessarily have sought accepted principles and rules of this sort. Almost certainly, he will have met with at least partial success. But, if his experience has been at all like my own, he will have found the search for rules both more difficult and less satisfying than the search for paradigms. Some of the generalizations he employs to describe the community’s shared beliefs will present more problems. Others, however, will seem a shade too strong. Phrased in just that way, or in any other way he can imagine, they would almost certainly have been rejected by some members of the group he studies. Nevertheless, if the coherence of the research tradition is to be understood in terms of rules, some specification of common ground in the corresponding area is needed. As a result, the search for a body of rules competent to constitute a given normal research tradition becomes a source of continual and deep frustration. Recognizing that frustration, however, makes it possible to diagnose its source. Scientists can agree that a Newton, Lavoisier, Maxwell, or Einstein has produced an apparently permanent solution to a group of outstanding problems and still disagree, sometimes without being aware of it, about the particular abstract characteristics that make those solutions permanent. They can, that is, agree in their identification of a paradigm without agreeing on, or even attempting to produce, a full interpretation or rationalization of it. Lack of a standard interpretation or of an agreed reduction to rules will not prevent a paradigm from guiding research. Normal science can be determined in part by the direct inspection of paradigms, a process that is often aided by but does not depend upon the formulation of rules and assumption. Indeed, the existence of a paradigm need not even imply that any full set of rules exists.

 

Q. 63 What is the author attempting to illustrate through this passage?

A. Relationships between rules, paradigms, and normal science

B. How a historian would isolate a particular ‘loci of commitment’

C. How a set of shared beliefs evolves into a paradigm

D. Ways of understanding a scientific tradition

E. The frustrations of attempting to define a paradigm of a tradition

 

Q. 64 The term ‘loci of commitment’ as used in the passage would most likely correspond with which of the following?

A. Loyalty between a group of scientists in a research laboratory

B. Loyalty between groups of scientists across research laboratories

C. Loyalty to a certain paradigm of scientific inquiry

D. Loyalty to global patterns of scientific inquiry

E. Loyalty to evolving trends of scientific inquiry

 

Q. 65 The author of this passage is likely to agree with which of the following?

A. Paradigms almost entirely define a scientific tradition.

B. A group of scientists investigating a phenomenon would benefit by defining a set of rules.

C. Acceptance by the giants of a tradition is a sine qua non for a paradigm to emerge

D. Choice of isolation mechanism determines the type of paradigm that may emerge from a tradition.

E. Paradigms are a general representation of rules and beliefs of a scientific tradition.

 

Q. 66 In each question, there are four sentences. Each sentence has pairs of words/phrases that are italicized and highlighted. From the italicized and highlighted word(s)/phrase(s), select the most appropriate word(s)/phrase(s) to form correct sentences. Then, from the options given, choose the best one.

The cricket council that was [A] / were [B] elected last March is [A] are [B] at sixes and sevens over new rules.

The critics censored [A] / censured [B] the new movie because of its social unacceptability.

Amit’s explanation for missing the meeting was credulous [A] I credible [B].

She coughed discreetly [A] / discretely [B] to announce her presence.

A. BBAAA

B. AAABA

C. BBBBA

D. AABBA

E. BBBAA

 

Q. 67 In each question, there are four sentences. Each sentence has pairs of words/phrases that are italicized and highlighted. From the italicized and highlighted word(s)/phrase(s), select the most appropriate word(s)/phrase(s) to form correct sentences. Then, from the options given, choose the best one.

The further [A] / farther [B] he pushed himself, the more disillusioned he grew. For the crowds it was more of a historical [A] / historic [B] event; for their leader, it was just another day.

The old man has a healthy distrust [A] / mistrust [B] for all new technology.

This film is based on a real [A] I true [B] story.

One suspects that the compliment [A] / complement [B] was backhanded.

A. BABAB

B. ABBBA

C. BAABA

D. BBAAB

E. ABABA

 

Q. 68 In each question, there are four sentences. Each sentence has pairs of words/phrases that are italicized and highlighted. From the italicized and highlighted word(s)/phrase(s), select the most appropriate word(s)/phrase(s) to form correct sentences. Then, from the options given, choose the best one.

Regrettably [A] / Regretfully [B] I have to decline your invitation.

I am drawn to the poetic, sensual [A] / sensuous [B] quality of her paintings.

He was besides [A] / beside [B] himself with rage when I told him what I had done.

After brushing against a stationary [A] / stationery [B] truck my car turned turtle.

As the water began to rise over [A] / above [B] the danger mark, the signs of an imminent flood were clear.

A. BAABA

B. BBBAB

C. AAABA

D. BBAAB

E. BABAB

 

Questions: 69 – 71

The difficulties historians face in establishing cause-and-effect relations in the history of human societies are broadly similar to the difficulties facing astronomers, climatologists, ecologists, evolutionary biologists; geologists, and palaeontologists. To varying degrees each of these fields is plagued by the impossibility of performing replicated, controlled experimental interventions, the complexity arising from enormous numbers of variables, the resulting uniqueness of each system, the consequent impossibility of formulating universal laws, and the difficulties of predicting emergent properties and future behaviour. Prediction in history, as in other historical sciences, is most feasible on large spatial scales and over long times, when the unique features of millions of small-scale brief events become averaged out. Just as I could predict the sex ratio of the next 1,000 newborns but not the sexes of my own two children. The historian can recognize factors that made inevitable the broad outcome of the collision between American and Eurasian societies after 13,000 years of separate developments, but not the outcome of the 1960 U.S. presidential election. The details of which candidate said what during a single televised debate in October 1960 could have given the electoral victory to Nixon instead of to Kennedy, but no details of who said what could have blocked the European conquest of Native Americans. How can students of human history profit from the experience of scientists in other historical sciences? A methodology that has proved useful involves the comparative method and so-called natural experiments. While neither astronomers studying galaxy formation nor human historians can manipulate their systems in controlled laboratory experiments, they both can take advantage of natural experiments, by comparing systems differing in the presence or absence (or in the strong or weak effect) of some putative causative factor. For example, epidemiologists, forbidden to feed large amounts of salt to people experimentally, have still been able to identify effects of high salt intake by comparing groups of humans who already differ greatly in their salt intake: and cultural anthropologists, unable to provide human groups experimentally with varying resource abundances for many centuries, still study long-term effects of resource abundance on human societies by comparing recent Polynesian populations living on islands differing naturally in resource abundance. The student of human history can draw on many more natural experiments than just comparisons among the five inhabited continents. Comparisons can also utilize large islands that have developed complex societies in a considerable degree of isolation (such as Japan, Madagascar. Native American Hispaniola, New Guinea, Hawaii, and many others), as well as societies on hundreds of smaller islands and regional societies within each of the continents. Natural experiments in any field, whether in ecology or human history, are inherently open to potential methodological criticisms. Those include confounding effects of natural variation in additional variables besides the one of interest, as well as problems in inferring chains of causation from observed correlations between variables. Such methodological problems have been discussed in great detail for some of the historical sciences. In particular, epidemiology, the science of drawing inferences about human diseases by comparing groups of people (often by retrospective historical studies), has for a long time successfully employed formalized procedures for dealing with problems similar to those facing historians of human societies. In short, I acknowledge that it is much more difficult to understand human history than to understand problems in fields of science where history is unimportant and where fewer individual variables operate. Nevertheless, successful methodologies for analyzing historical problems have been worked out in several fields. As a result, the histories of dinosaurs, nebulae, and glaciers are generally acknowledged to belong to fields of science rather than to the humanities.

 

Q. 69 Why do islands with considerable degree of isolation provide valuable insights into human history?

A. Isolated islands may evolve differently and this difference is of interest to us

B. Isolated islands increase the number of observations available to historians.

C. Isolated islands, differing in their endowments and size may evolve differently and this difference can be attributed to their endowments and size.

D. Isolated islands. differing in their endowments and size, provide a good comparison to large islands such as Eurasia, Africa, Americas and Australia.

E. Isolated islands, in so far as they are inhabited, arouse curiosity about how human beings evolved there.

 

Q. 70 According to the author, why is prediction difficult in history?

A. Historical explanations are usually broad so that no prediction is possible.

B. Historical outcomes depend upon a large number of factors and hence prediction is difficult for each case.

C. Historical sciences, by their very nature, are not interested in a multitude of minor factors, which might be important in a specific historical outcome.

D. Historians are interested in evolution of human history and hence are only interested in long-term predictions.

E. Historical sciences suffer from the inability to conduct controlled experiments and therefore have explanations based on a few long-term factors.

 

Q. 71 According to the author, which of the following statements would be true?

A. Students of history are missing significant opportunities by not conducting any natural experiments.

B. Complex societies inhabiting large islands provide great opportunities for natural experiments.

C. Students of history are missing significant opportunities by not studying an adequate variety of natural experiments.

D. A unique problem faced by historians is their inability to establish cause and effect relationships.

E. Cultural anthropologists have overcome the problem of confounding variables through natural experiments.

 

Q. 72 The sentence/paragraph labelled A is in its correct place. The four that follow are labelled B, C, D and E, and need to be arranged in the logical order to form a coherent paragraph/passage. From the given options, choose the most appropriate one.

A. In America, highly educated women, who are in stronger position in the labour market than less qualified ones, have higher rates of marriage than other groups.

B. Some work supports the Becker thesis, and some appears to contradict it.

C. And, as with crime, it is equally inconclusive.

D. But regardless of the conclusion of any particular piece of work, it is hard to establish convincing connections between family changes and economic factors using conventional approaches.

E. Indeed, just as with crime, an enormous academic literature exists on the validity of the pure economic approach to the evolution of family structures.

A. BCDE

B. DBEC

C. BDCE

D. ECBD

E. EBCD

 

Q. 73 The sentence/paragraph labelled A is in its correct place. The four that follow are labelled B, C, D and E, and need to be arranged in the logical order to form a coherent paragraph/passage. From the given options, choose the most appropriate one.

A. Personal experience of mothering and motherhood are largely framed in relation to two discernible or “official” discourses: the “medical discourse and natural childbirth discourse”. Both of these tend to focus on the “optimistic stories” of birth and mothering and underpin stereotypes of the “good mother”.

B. At the same time, the need for medical expert guidance is also a feature for contemporary reproduction and motherhood. But constructions of good mothering have not always been so conceived – and in different may exist in parallel to other equally dominant discourses.

C. Similarly, historical work has shown how what are now taken-for-granted aspects of reproduction and mothering practices result from contemporary “pseudoscientific directives” and “managed constructs”. These changes have led to a reframing of modern discourses that pattern pregnancy and motherhood leading to an acceptance of the need for greater expert management.

D. The contrasting, overlapping, and ambiguous strands within these frameworks focus to varying degrees on a woman’s biological tie to her child and predisposition to instinctively know and be able to care for her child.

E. In addition, a third, “unofficial popular discourse” comprising “old wives” tales and based on maternal experiences of childbirth has also been noted. These discourses have also been acknowledged in work exploring the experiences of those who apparently do not “conform” to conventional stereotypes of the “good mother”.

A. EDBC

B. BCED

C. DBCE

D. EDCB

E. BCDE

 

Q. 74 The sentence/paragraph labelled A is in its correct place. The four that follow are labelled B, C, D and E, and need to be arranged in the logical order to form a coherent paragraph/passage. From the given options, choose the most appropriate one.

A. Indonesia has experienced dramatic shifts in its formal governance arrangements since the fall of President Soeharto and the close of his centralized, authoritarian “New Order” regime in 1997.

B. The political system has taken its place in the nearly 10 years since Reformasi began. It has featured the active contest for political office among a proliferation of parties at central, provincial and district levels; direct elections for the presidency (since 2004); and radical changes in centre-local government relations towards administrative, fiscal, and political decentralization.

C. The mass media, once tidily under Soeharto’s thumb, has experienced significant liberalization, as has the legal basis for non-governmental organizations, including many dedicated to such controversial issues as corruption control and human rights.

D. Such developments are seen optimistically by a number of donors and some external analysts, who interpret them as signs of Indonesia’s political normalization.

E. A different group of analysts paint a picture in which the institutional forms have changed, bitt power relations have not. Vedi Hadiz argues that Indonesia’s “democratic transition” has been anything but linear.

A. BDEC

B. CBDE

C. CEBD

D. BCDE

 

Q. 75 The sentence/paragraph labelled A is in its correct place. The four that follow are labelled B, C, D and E, and need to be arranged in the logical order to form a coherent paragraph/passage. From the given options, choose the most appropriate one.

A. I had six thousand acres of land, and had thus got much spare land besides the coffee plantation. Part of the farm was native forest, and about one thousand acres were squatters’ land, what [the Kikuyu] called their shambas.

B. The squatters’ land was more intensely alive than the rest of the farm, and was changing with the seasons the year round. The maize grew up higher than your head as you walked on the narrow hard-trampled footpaths in between the tall green rustling regiments.

C. The squatters are Natives, who with their families hold a few acres on a white man’s farm, and in return have to work for him a certain number of days in the year. My squatters, I think, saw the relationship in a different light, for many of them were born on the farm, and their fathers before them, and they very likely regarded me as a sort of superior squatter on their estates.

D. The Kikuyu also grew the sweet potatoes that have a vine like leaf and spread over the ground like a dense entangled mat, and many varieties of big yellow and green speckled pumpkins.

E. The beans ripened in the fields, were gathered and thrashed by the women, and the maize stalks and coffee pods were collected and burned, so that in certain seasons thin blue columns of smoke rose here and there all over the farm

A. CBDE

B. BCDE

C. CBED

D. DBCE

E. EDBC

 

 

Answer Sheet
Question 1 2 3 4 5 6 7 8 9 10
Answer B E A C D E C B A A
Question 11 12 13 14 15 16 17 18 19 20
Answer D B C B E D A E B D
Question 21 22 23 24 25 26 27 28 29 30
Answer C C B B D A D E E A
Question 31 32 33 34 35 36 37 38 39 40
Answer D C E B C E A B E A
Question 41 42 43 44 45 46 47 48 49 50
Answer C C A D B D B C B D
Question 51 52 53 54 55 56 57 58 59 60
Answer E B B C E C C A C E
Question 61 62 63 64 65 66 67 68 69 70
Answer A B A C E D E B D E
Question 71 72 73 74 75
Answer B E A B C

CAT Previous Year Paper 2006

CAT 2006

Quant 

Instructions 

A punching machine is used to punch a circular hole of diameter two Units from a square sheet of aluminium of width 2 units, as shown below. The hole is punched such that the circular hole touches one corner P of the square sheet and the diameter of the hole originating at P is in line with a diagonal of the square: 

Q. 1 The proportion of the sheet area that remains after punching is: 

A. (π +2)/ 8 

B. (6 –π )/ 8 

C. (4 – π)/4 

D. (π -2)/ 4 

E. (14 -3π)/ 6 

Answer: B. 

Explanation: 

The area of triangle ABC is 1/2 *√2 *√2 = 1 

Area of semicircle ABC = π/2

So, area of circle outside the square = π/2 – 1 = (π -2)/2 

So, area of circle inside the sheet =π/2 – (π – 1) = 1 + π/2

Area of original square = 2*2 = 4 

So, area of the sheet after punching = 4 – 1 – π/2 = 3 – π/2

So, proportion of sheet that remains after punching = (3 –π/2 )/4 = (6 – π)/8 

Q. 2 Find the area of the part of the circle (round punch) falling outside the square sheet. 

A. π /4 

B. (π  -1)/2 

C. (π  -1)/4 

D. (π -2)/2 

E. (π -2)/4 

Answer: D. 

Explanation: 

The area of triangle ABC is 1/2 *√2 *√2 = 1 

Area of semi-circle ABC = π/2 

So, area of circle outside the square = π/2 – 1 = ( π-2)/2 

 

Instructions 

An airline has a certain free luggage allowance and charges for excess luggage at a fixed rate per kg. Two passengers, Raja and Praja have 60 kg of luggage between them, and are charged Rs 1200 and Rs 2400 respectively for excess luggage. Had the entire luggage belonged to one of them, the excess luggage charge would have been Rs 5400. 

Q. 3 What is the weight of Praja’s luggage? 

A. 20 kg 

B. 25 kg 

C. 30 kg 

D. 35 kg 

E. 40 kg 

Answer: D. 

Explanation: 

Let the limit be x and the rate of charge be k per kg. 

Let the excess luggage with Raja be R kg. 

So, excess luggage with Praja = 2R kg 

Now, excess luggage with Raja + excess luggage with Praja = 60 – 2x 

So, 3R = 60 – 2x => R = 20 – 2x/3 which was charged 1200 Also, if one person had the entire luggage, excess luggage would have been 60 – x, which would have been charged 5400. 

=> (60-2x)/3*(60-x) = 1200/5400 

Solving this, x = 15 kg 

So, Praja’s luggage = 35 kg 

Q. 4 What is the free luggage allowance? 

A. 10 kg 

B. 15 kg 

C. 20 kg 

D. 25kg 

E. 30kg 

Answer: B. 

Explanation: 

Let the limit be x and the rate of charge be k per kg. 

Let the excess luggage with Raja be R kg. 

So, excess luggage with Praja = 2R kg 

Now, excess luggage with Raja + excess luggage with Praja = 60 – 2x 

So, 3R = 60 – 2x => R = 20 – 2x/3 which was charged 1200 Also, if one person had the entire luggage, excess luggage would have been 60 – x, which would have been charged 5400. 

=> (60-2x)/3*(60-x) = 1200/5400 

Solving this, x = 15 kg 

 

Instructions 

For the following questions answer them individually 

Q. 5 If x = -0.5, then which of the following has the smallest value? 

A. 21/x

B. 1/x

C. 1/x2

D. 2x

E. 1/√-x

Answer: B. 

Explanation: 

2P is always positive

x2 is always non negative

1/√-x is always positive

1/x is negative when x is negative.

In this case, x is negative =>1/x is smallest. 

Q. 6 Which among 21/2 , 31/3 , 41/4 , 61/6, and 121/12 is the largest? 

A. 21/2

B. 31/3

C. 41/4

D. 61/6

E. 121/12

Answer: B. 

Explanation: 

Make the power equal and compare the denominators. 

21/2 can be written as 641/12

31/3 can be written as  811/12

41/4can be written as 641/12

61/6 can be written as 361/12

Among these, 811/12 is the greatest =>31/3 is the greatest. 

Q. 7 If a/b = 1/3, b/c = 2, c/d = 1/2 , d/e = 3 and e/f = 1/4, then what is the value of abc/def ? 

A. 3/8 

B. 27/8 

C. 3/4 

D. 27/4 

E. 1/4 

Answer: A. 

Explanation: 

a/d = a/b * b/c * c/d = 1/3 * 2 * 1/2 = 1/3 

Similarly, b/e and c/f are 3 and 3/8 respectively. 

b/e = b/c*c/d*d/e = 3 

c/f = c/d*d/e*e/f = 3/8 

=> Value of abc/def = 1/3 * 3 * 3/8 = 3/8 

Q. 8 The length, breadth and height of a room are in the ratio 3:2:1. If the breadth and height are halved while the length is doubled, then the total area of the four walls of the room will 

A. remain the same 

B. decrease by 13.64% 

C. decrease by 15% 

D. decrease by 18.75% 

E. decrease by 30% 

Answer: E. 

Explanation: 

The area of the four walls is length*height*2 + breadth*height*2 

Initial area = 3*1*2 + 2*1*2 = 10 

Final area = 6*1/2*2 + 1*1/2*2 = 7 

So, the area decreased by 30% 

Q. 9 Consider a sequence where the nth term, tn = n/(n + 2), n = 1, 2, …. The value of t3t4t5 ∗ ….. ∗ t53 equals. 

A. 2/495 

B. 2/477 

C. 12/55 

D. 1/1485 

E. 1/2970 

Answer: A. 

Explanation: 

substituting 3,4…53 in the given function, we get 

t3 = 3/5 

t4 = 4/6 

t5 = 5/7 

t6 = 6/8 

Multiplying the values, we get ⅗ * 4/6 * 5/7 * ….. 52/54 * 53/55 which ultimately after cancellations give 3*4 /54*55 = 2/495 

Q.10 A group of 630 children is arranged in rows for a group photograph session. Each row contains three fewer children than the row in front of it. What number of rows is not possible? 

A.

B.

C.

D.

E.

Answer: D. 

Explanation: 

Let x be in the front row. 

So no. of children in next rows will be x-3,x-6,x-9,x-12,x-15,x-18,x-21…. 

Suppose there are 6 rows, then the sum is equal to x + x-3 + x-6 + x-9 + x-12 + x-15 = 6x – 45 This sum is equal to 630. 

=> 6x – 45 = 630 => 6x = 585 

Here, x is not an integer. 

Hence, there cannot be 6 rows. 

Q. 11 What are the values of x and y that satisfy both the equations? 

20.7x ∗ 3−1.25y = 8√6 /27

40.3x ∗ 90.2y = 8 ∗ 811/5

A. x = 2, y = 5 

B. x = 2.5, y = 6 

C. x =3, y = 5 

D. x = 3,y = 4 

E. x = 5,y = 2 

Answer: E. 

Explanation: 

20.7x ∗ 3−1.25y = 8√6 /27 

=> 20.7x ∗ 3−1.25y = 23.5 ∗ 3-2.5

=> 0.7x = 3.5 => x = 5 

=> -1.25y = -2.5 => y = 2

40.3x ∗ 90.2y = 8 ∗ 811/5

=> 40.3x ∗ 90.2y = 23 ∗ 30.8 

=> 0.6x = 3 => x = 5 

=> 0.4y = 0.8 => y = 2

=> (5,2) is the solution. 

Q. 12 The number of solutions of the equation 2x + y = 40 where both x and y are positive integers and x <= y is: 

A.

B. 13 

C. 14 

D. 18 

E. 20 

Answer: B. 

Explanation: 

y = 38 => x = 1 

y = 36 => x = 2 

… 

… 

y = 14 => x = 13 

y = 12 => x = 14 => Cases from here are not valid as x > y. 

Hence, there are 13 solutions. 

Q. 13 A survey was conducted of 100 people to find out whether they had read recent issues of Golmal, a monthly magazine. The summarized information regarding readership in 3 months is given below: 

Only September: 18; 

September but not August: 23; 

September and July: 8; 

September:28; 

July: 48; 

July and August: 10; 

none of the three months: 24 

What is the number of surveyed people who have read exactly two consecutive issues (out of the three)? 

A.

B.

C. 12 

D. 14 

E. 17 

Answer: B. 

Explanation: 

Let the areas be labelled as shown in the diagram above. 

The number of people corresponding to “none of the three months” is 24. So, H is 24. 

Only September is 18. So, G = 18 

September but not August is 23. So, G + D = 23. 

Hence, D = 23 – 18 = 5. 

We know that September and July are 8. So, D + E = 8 

This implies E = 3. 

September = 28. 

So, D + E + F + G = 28. 

So, F = 28 – 5 – 3 – 18 = 2. 

July and August = 10. 

So, B + E = 10. 

E = 3. So, B = 7. 

July = 48. 

So, A + B + D + E = 48 

A = 48 – 7 – 5 – 3 = 33. 

There are 100 people in total. So, C = 100 – A – B – D – E – F – G – H 

= 100 – 33 – 7 – 5 – 3 – 2 – 18 – 24 = 8 

So, number of people who read exactly two consecutive issues 

= (July & August) + (August & September) 

= B + F = 7 + 2 = 9 

Q. 14 The sum of four consecutive two-digit odd numbers, when divided by 10, becomes a perfect square. Which of the following can possibly be one of these four numbers? 

A. 21 

B. 25 

C. 41 

D. 67 

E. 73 

Answer: C. 

Explanation: 

Sum of the four numbers < 396 

396/10 = 39.6 

So, the perfect square is a number less than 39.6 

The possibilities are 36, 25, 16 and 9 

For the sum to be 360, the numbers can be 87, 89, 91 and 93 

The sum of four consecutive odd numbers cannot be 250 

For the sum to be 160, the numbers can be 37,39,41 and 43 

The sum of 4 consecutive odd numbers cannot be 90 

So, from the options, the answer is 41. 

Q. 15 The graph of y – x (on the y axis) against y + x (on the x axis) is as shown below. (All graphs in this question are drawn to scale and the same scale and the same scale has been used on each axis.) 

Which of the following shows the graph of y against x? 

Answer: D. 

Explanation: 

y-x = k(y+x) 

y = k(x+1)/(1-k) 

Since k>1 

Therefore y<0 for x>-1 and y>0 for x<-1 

Option d correctly satisfy this condition 

Q. 16 Consider the set S = { 1, 2, 3, .., 1000 }. How many arithmetic progressions can be formed from the elements of S that start with l and end with 1000 and have at least 3 elements? 

A.

B.

C.

D.

E.

Answer: D. 

Explanation: 

The nth term is a + (n-1)d 

1000 = 1 + (n-1)d 

So, (n-1)d = 999 

999 = 3^3 * 37 

So, the number of factors is 4*2 = 8 

Since there should be at least 3 terms in the series, d cannot be 999. 

So, the number of possibilities is 7 

Q. 17 What values of x satisfy x2/3 + x1/3 − 2 ≤ 0

A. −8 ≤ x ≤ 1

B.  −1 ≤ x ≤ 8

C. 1 ≤ x ≤ 8

D. 1 ≤ x ≤ 18

E. −8 ≤ x ≤ 8

Answer: A. 

Explanation: 

Try to solve this type of questions using the options. 

Substitute 0 first => We ger -2 <=0, which is correct. Hence, 0 must be in the solution set. 

Substitute 8 => 4 + 2 – 2 <=0 => 6 <= 0, which is false. Hence, 8 must not be in the solution set. 

=> Option 1 is the answer. 

Q. 18 Let f(x) = max (2x + 1, 3 – 4x), where x is any real number. Then the minimum possible value of f(x) is: 

A. 1/3 

B. 1/2 

C. 2/3 

D. 4/3 

E. 5/3 

Answer: E. 

Explanation: 

The minimum value is obtained when 2x+1 = 3-4x => 6x = 2 => x = 1/3 

So, f((x) = 2*1/3 + 1 = 5/3 

Q. 19 Arun, Barun and Kiranmala start from the same place and travel in the same direction at speeds of 30, 40 and 60 km per hour respectively. Barun starts two hours after Arun. If Barun and Kiranmala overtake Arun at the same instant, how many hours after Arun did Kiranmala start? 

A.

B. 3.5 

C.

D. 4.5 

E.

Answer: C. 

Explanation: 

Let the distance be D. 

Time taken by Arun = D/30 

Time taken by Barun = D/40 

Now, D/40 = D/30 – 2 

=> 3D. = 4D. – 240 

=> D. = 240 

Therefore time taken by Arun to cover 240 km = 240/30 = 8 hr 

Time Kiranmala takes to cover 240 km = 240/60 = 4 hr 

So, Kiranmala has to start 4 hours after Arun. 

Q. 20 When you reverse the digits of the number 13, the number increases by 18. How many other two-digit numbers increase by 18 when their digits are reversed? 

A.

B.

C.

D.

E. 10 

Answer: B. 

Explanation: 

Let the number be xy 

10y + x = 10x + y + 18 

=> 9y – 9x = 18 

=> y – x = 2 

So, y can take values from 9 to 4 (since 3 is already counted in 13) 

Number of possible values = 6 

Q. 21 A semicircle is drawn with AB as its diameter. From C, a point on AB, a line perpendicular to AB is drawn meeting the circumference of the semi-circle at D Given that AC = 2 cm and CD = 6 cm, the area of the semi-circle (in sq. cm) will be: 

A. 32 π

B. 50 π

C. 40.5 π

D. 81 π

E. Can’t be determined 

Answer: B. 

Explanation: 

In triangle CDO, which is a right angled triangle, we can use pythagoras theorem. 

62+ (r − 2)2 = r2 

=> 36 + r2 − 4r + 4 = r2

=> 4r = 40 

=> r = 10 

=> Area =π ∗ 10 ∗ 10/2 = 50π

Q. 22 There are 6 tasks and 6 persons. Task 1 cannot be assigned either to person 1 or to person 2; task 2 must be assigned to either person 3 or person 4. Every person is to be assigned one task. In how many ways can the assignment be done? 

A. 144 

B. 180 

C. 192 

D. 360 

E. 716 

Answer: A. 

Explanation: 

If the first task is assigned to either person 3 or person 4, the second task can be assigned in only 1 way. If the first task is assigned to either person 5 or person 6, the second task can be assigned in 2 ways. Therefore, the number of ways in which the first two tasks can be assigned is 2*1 + 2*2 = 6. 

The other 4 tasks can be assigned to 4 people in 4! ways. 

The total number of ways of assigning the 6 tasks is, therefore, 6*4! = 144. 

Q. 23 The number of employees in Obelix Menhir Co. is a prime number and is less than 300. The ratio of the number of employees who are graduates and above, to that of employees who are not, can possibly be: 

A. 101:88 

B. 87:100 

C. 110:111 

D. 85:98 

E. 97:84 

Answer: E. 

Explanation: 

The addition of numerator and denominator should give a prime no. Only option E. gives that. 3 is a factor of 189 and 183 => A. and D. eliminated 

17 is a factor of 187 and 221 => B. and C. eliminated 

181 is prime. 

Q. 24 If logyx = (a logzy) = (b logxz) = ab, then which of the following pairs of values for (a, b) is not possible? 

A. (-2, 1/2) 

B. (1,1) 

C. (0.4, 2.5)  

D. (π , 1/π

E. (2,2) 

Answer: E. 

Explanation: 

logyx = ab 

=> a logzy = ab logzy =

=> b logxz = ab logxz =

Logyx =logzy logxz =>logx/logy = logy/logz logz/logx 

=> logx /logy = logy /logx

=> (logx)2=(logy)2

=> logx = logy or logx = −logy

So, x = y or x = 1/y 

So, ab = 1 or -1 

Option 5) is not possible 

Q. 25 An equilateral triangle DOC is drawn inside a square ABCD What is the value of the angle AOB in degrees? 

A. 75 

B. 90 

C. 120 

D. 135 

E. 150 

Answer: E. 

Explanation: 

Triangle AOD. is isosceles. So, angle DAO = angle DOA. = 75. Similarly, angle BOC. = 75. So, angle AOB. = 150 

Data Interpretation 

Instructions 

In a Class X Board examination, ten papers are distributed over five Groups – PCB, Mathematics, Social Science, Vernacular and English. Each of the ten papers is evaluated out of 100. The final score of a student is calculated in the following manner. First, the Group Scores are obtained by averaging marks in the papers within the Group. The final score is the simple average of the Group Scores. The data for the top ten students are presented below. (Dipan’s score in English Paper II has been intentionally removed in the table.) 

Note: B or G against the name of a student respectively indicates whether the student is a boy or a girl. 

Q. 26 How much did Dipan get in English Paper II? 

A. 94 

B. 96.5 

C. 97 

D. 98 

E. 99 

Answer: C. 

Explanation: 

Since we know the average , we can calculate the marks Dipan got in English Paper II (x) as ; 98+95+95.5+95+ (96+x)/2 = 96*5 . We get 96+x = 96.5*2 . Thus x = 97 . Hence option C. . 

Q. 27 Students who obtained Group Scores of at least 95 in every group are eligible to apply for a prize. Among those who are eligible, the student obtaining the highest Group Score in Social Science Group is awarded this prize. The prize was awarded to: 

A. Shreya 

B. Ram 

C. Ayesha 

D. Dipan 

E. no one from the top ten 

Answer: D. 

Explanation: 

From the given data it is clear that Dipan is the only student who obtained Group Scores of at least 95 in every group and obtained the highest Group Score in Social Science Group. Hence option D. 

Q. 28 Among the top ten students, how many boys scored at least 95 in at least one paper from each of the groups? 

A.

B.

C.

D.

E.

Answer: A. 

Explanation: 

Among the top ten students, only Dipan scored at least 95 in at least one paper from each of the groups. Hence option A. 

Q. 29 Each of the ten students was allowed to improve his/her score in exactly one paper of choice with the objective of maximizing his /her final score. Everyone scored 100 in the paper in which he or she chose to improve. After that, the topper among the ten students was: 

A. Ram 

B. Agni 

C. Pritam 

D. Ayesha 

E. Dipan 

Answer: E. 

Explanation: 

Lets first consider ayesha’s marks , her 7 marks will increase in geography . So increase in her average marks is 7/(2*5) = 0.7. So new average is 96.9 . 

Now for dipan , his 5 marks will increase in mathematics . So increase in his average 5/5 = 1 . So his new average is 96+1 = 97 which is greater than ayesha’s . Other will clearly have a lower average than these both. Hence option E. 

Q. 30 Had Joseph, Agni, Pritam and Tirna each obtained Group Score of 100 in the Social Science Group,then their standing in decreasing order of final score would be; 

A. Pritam, Joseph, Tirna, Agni 

B. Joseph, Tirna, Agni, Pritam 

C. Pritam, Agni, Tirna, Joseph 

D. Joseph, Tirna, Pritam, Agni 

E. Pritam, Tirna, Agni, Joseph 

Answer: A. 

Explanation: 

Had Joseph, Agni, Pritam and Tirna each obtained Group Score of 100 in the Social Science Group , their final score would increase by 0.9, 0.9, 2.2, 2.1 respectively. Adding these to the final score then their standing in decreasing order of final score would be Pritam, Joseph, Tirna, Agni with scores of 96.1,95.9,95.8,95.2 respectively. Hence option A. 

 

Instructions 

Mathematicians are assigned a number called Erdos number (named after the famous mathematician, Paul Erdos). Only Paul Erdos himself has an Erdos number of zero. Any mathematician who has written a research paper with Erdos has an Erdos number of 1.For other mathematicians, the calculation of his/her Erdos number is illustrated below: 

Suppose that a mathematician X has co-authored papers with several other mathematicians. ‘From among them, mathematician Y has the smallest Erdos number. Let the Erdos number of Y be y. Then X has an Erdos number of y+1. Hence any mathematician with no co-authorship chain connected to Erdos has an Erdos number of infinity. : 

In a seven day long mini-conference organized in memory of Paul Erdos, a close group of eight mathematicians, call them A, B, C, D, E, F, G and H, discussed some research problems. At the beginning of the conference, A. was the only participant who had an infinite Erdos number. Nobody had an Erdos number less than that of F. 

On the third day of the conference F co-authored a paper jointly with A. and C. This reduced the average Erdos number of the group of eight mathematicians to 3. The Erdos numbers of B, D, E, G and H remained unchanged with the writing of this paper. Further, no other co-authorship among any three members would have reduced the average Erdos number of the group of eight to as low as 3. 

• At the end of the third day, five members of this group had identical Erdos numbers while the other three had Erdos numbers distinct from each other. 

• On the fifth day, E. co-authored a paper with F which reduced the group’s average Erdos number by 0.5. The Erdos numbers of the remaining six were unchanged with the writing of this paper. 

• No other paper was written during the conference. 

Q. 31 The person having the largest Erdos number at the end of the conference must have had Erdos number (at that time): 

A.

B.

C.

D. 14 

E15 

Answer: B. 

Explanation: 

Let us consider the Erdos number of A,B,C,D,E,F,G,H be a,b,c,d,e,f,g,h where f is the min, a is infinity. 

At the end of 3rd day, F co authored with A. and C. Since F has min Erdos number ,the values of c,a will change to f+1 and the Erdos number of F will remain the same. [Because according to Erdos principle if a person co-authors with some one who has higher Erdos number then the Erdos number of co-authors will be min Erdos value + 1] 

Average of the mathematicians is 3 

Sum of the Erdos number of eight mathematicians=24 

Erdos number at the third day:f+1,b,f+1,d,e,f,g,h 

At the end of the fifth day, F co-authors with E. thereby changing the average to 2.5 and the Erdos number of rest of the mathematicians remain unchanged. 

Sum of the Erdos numbers of eight mathematicians=20 

So here the difference of 4[24-20] arose, which means e will be f+5 initially and changed to f+1 after co-authoring with F. 

So the Erdos number at the third day:f+1,b,f+1,d,f+5,f,g,h 

At the end of the third day, five mathematicians had the same Erdos number and the rest had distinct Erdos number from each other. 

It cannot be f+5 because then there will be two mathematicians with the same Erdos number f+1. So five mathematicians will have f+1, one with f+5,one with f, one with some different value say x 5(f+1)+f+5+f+x=24 

7f+x=14 

The only value which satisfies the above equation is f=1,x=7 

Erdos number at the end of fifth day,f+1,b,f+1,d,f+1,f,g,h 

On tabulating, we get 

Hence the person having the largest Erdos number at the end of the conference must have had Erdos number 7 . Hence option B. 

Q. 32 How many participants in the conference did not change their Erdos number during the conference? 

A.

B.

C.

D.

E. Cannot be determined 

Answer: D. 

Explanation: 

Let us consider the Erdos number of A,B,C,D,E,F,G,H be a,b,c,d,e,f,g,h where f is the min, a is infinity. 

At the end of 3rd day, F co authored with A. and C. Since F has min Erdos number ,the values of c,a will change to f+1 and the Erdos number of F will remain the same. [Because according to Erdos principle if a person co-authors with some one who has higher Erdos number then the Erdos number of co-authors will be min Erdos value + 1] 

Average of the mathematicians is 3 Sum of the Erdos number of eight mathematicians=24 Erdos number at the third day:f+1,b,f+1,d,e,f,g,h 

At the end of the fifth day, F co-authors with E. thereby changing the average to 2.5 and the Erdos number of rest of the mathematicians remain unchanged. Sum of the Erdos numbers of eight mathematicians=20 So here the difference of 4[24-20] arose, which means e will be f+5 initially and changed to f+1 after co-authoring with F. 

So the Erdos number at the third day:f+1,b,f+1,d,f+5,f,g,h 

At the end of the third day, five mathematicians had the same Erdos number and the rest had distinct Erdos number from each other. 

It cannot be f+5 because then there will be two mathematicians with the same Erdos number f+1. So five mathematicians will have f+1, one with f+5, one with f, one with some different value say x 5(f+1)+f+5+f+x=24 

7f+x=14 

The only value which satisfies the above equation is f=1,x=7 

Erdos number at the end of fifth day,f+1,b,f+1,d,f+1,f,g,h On tabulating, we get 

So B,D. ,F,G,H are 5 participants in the conference who did not change their Erdos number during the conference. 

Q. 33 The Erdos number of C. at the end of the conference was: 

A.

B.

C.

D.

E.

Answer: B. 

Explanation: 

Let us consider the Erdos number of A,B,C,D,E,F,G,H be a,b,c,d,e,f,g,h where f is the min, a is infinity. 

At the end of 3rd day, F co authored with A. and C. Since F has min Erdos number ,the values of c,a will change to f+1 and the Erdos number of F will remain the same. [Because according to Erdos principle if a person co-authors with some one who has higher Erdos number then the Erdos number of co-authors will be min Erdos value + 1] 

Average of the mathematicians is 3 Sum of the Erdos number of eight mathematicians=24 Erdos number at the third day:f+1,b,f+1,d,e,f,g,h 

At the end of the fifth day, F co-authors with E. thereby changing the average to 2.5 and the Erdos number of rest of the mathematicians remain unchanged. Sum of the Erdos numbers of eight mathematicians=20 So here the difference of 4[24-20] arose, which means e will be f+5 initially and changed to f+1 after co-authoring with F. 

So the Erdos number at the third day:f+1,b,f+1,d,f+5,f,g,h 

At the end of the third day, five mathematicians had the same Erdos number and the rest had distinct Erdos number from each other. 

It cannot be f+5 because then there will be two mathematicians with the same Erdos number f+1. So five mathematicians will have f+1, one with f+5, one with f, one with some different value say x 5(f+1)+f+5+f+x=24 

7f+x=14 

The only value which satisfies the above equation is f=1,x=7 

Erdos number at the end of fifth day,f+1,b,f+1,d,f+1,f,g,h On tabulating, we get 

Erdos no. of C. at the end is f+1 = 1+1 = 2. Hence option B. 

Q. 34 The Erdos number of E. at the beginning of the conference was: 

A.

B.

C.

D.

E.

Answer: C. 

Explanation: 

Let us consider the Erdos number of A,B,C,D,E,F,G,H be a,b,c,d,e,f,g,h where f is the min, a is infinity. 

At the end of 3rd day, F co authored with A. and C. Since F has min Erdos number ,the values of c,a will change to f+1 and the Erdos number of F will remain the same. [Because according to Erdos principle if a person co-authors with some one who has higher Erdos number then the Erdos number of co-authors will be min Erdos value + 1] 

Average of the mathematicians is 3 Sum of the Erdos number of eight mathematicians=24 

Erdos number at the third day:f+1,b,f+1,d,e,f,g,h 

At the end of the fifth day, F co-authors with E. thereby changing the average to 2.5 and the Erdos number of rest of the mathematicians remain unchanged. Sum of the Erdos numbers of eight mathematicians=20 So here the difference of 4[24-20] arose, which means e will be f+5 initially and changed to f+1 after co-authoring with F. 

So the Erdos number at the third day:f+1,b,f+1,d,f+5,f,g,h 

At the end of the third day, five mathematicians had the same Erdos number and the rest had distinct Erdos number from each other. 

It cannot be f+5 because then there will be two mathematicians with the same Erdos number f+1. So five mathematicians will have f+1, one with f+5, one with f, one with some different value say x 5(f+1)+f+5+f+x=24 

7f+x=14 

The only value which satisfies the above equation is f=1,x=7 

Erdos number at the end of fifth day,f+1,b,f+1,d,f+1,f,g,h On tabulating, we get 

Hence erdos no. of E. at the beginning of conference would be f+5 = 6 . 

Q. 35 How many participants had the same Erdos number at the beginning of the conference? 

A.

B.

C.

D.

E. Cannot be determined 

Answer: B. 

Explanation: 

Since at the end of the 3rd day 5 people had identical erdos no.(f+1) so : 5*(f+1) +f+f+5+x = 24 ; Only f=1 and x = 7 satisfies the equation. So out of 5 people who had identical erdos no. at the end of day 3, 2 of them had different nos. at the beginning. So there were 5-2 = 3 participants who had the same Erdos number at the beginning of the conference. 

 

Instructions 

Two traders, Chetan and Michael, were involved in the buying and selling Of MCS shares over five trading days. At the beginning of the first day, the MCS share was priced at Rs 100, while at the end of the fifth day it was priced at Rs 110. At the end of each day, the MCS share price either went up by Rs 10, or else, it came down by Rs 10. Both Chetan and Michael took buying and selling decisions at the end of each trading day. The beginning price of MCS share on a given 

day was the same as the ending price of the previous day. Chetan and Michael started with the same number of shares and amount of cash, and had enough of both. Below are some additional facts about how Chetan and Michael traded over the five trading days. 

• Each day if the price went up, Chetan sold 10 shares of MCS at the closing price. On the other hand, each day if the price went down, he bought 10 shares at the closing price. 

• If on any day, the closing price was above Rs 110, then Michael sold 10 shares of MCS, while if it was below Rs 90, he bought 10 shares, all at the closing price. 

Q. 36 If Chetan sold 10 shares of MCS on three consecutive days, while Michael sold 10 shares only once during the five days, what was the price of MCS at the end of day 3? 

A. Rs 90 

B. Rs 100 

C. Rs 110 

D. Rs 120 

E. Rs 130 

Answer: C. 

Explanation: 

Let at start no. of shares of both be 0 (No. of shares can be negative) and M and C. be no. of shares at the end of 5th day. Following are the possibilities in the changes in share prices over days : 

In the 5th possibility we can see that Chetan sold 10 shares of MCS on three consecutive days, while Michael sold 10 shares only once during the five days, so price of MCS at the end of day 3 was 110. Hence option C. 

Q. 37 If Michael ended up with Rs 100 less cash than Chetan at the end of day 5, what was the difference in the number of shares possessed by Michael and Chetan (at the end of day 5)? 

A. Michael had l0 less shares than Chetan. 

B. Michael had 10 more shares than Chetan. 

C. Chetan had 10 more shares than Michael. 

D. Chetan had 20 more shares than Michael. 

E. Both had the same number of shares. 

Answer: E. 

Explanation: 

Let at start no. of shares of both be 0 (No. of shares can be negative) and M and C. be no. of shares at the end of 5th day. Following are the possibilities in the changes in share prices over days : 

For the 6th possibility we find that chetan earned 1300 after selling and earned 1200 after selling at 120. Hence when Michael ended up with Rs 100 less cash than Chetan at the end of day 5 the no. of shares possesed are same. 

Q. 38 If Chetan ended up with Rs 1300 more cash than Michael at the end of day 5, what was the price of MCS share at the end of day 4? 

A. Rs 90 

B. Rs 100 

C. Rs 110 

D. Rs 120 

E. Not uniquely determinable 

Answer: B. 

Explanation: 

Let at start no. of shares of both be 0 (No. of shares can be negative) and M and C. be no. of shares at the end of 5th day. Following are the possibilities in the changes in share prices over days : 

We can calculate in the 3rd possibility, Chetan ended up with Rs 1300 and Michael ended with 0 ( by selling no shares) at the end of day 5, the price of MCS share at the end of day 4 is 100 as from the table. 

Q. 39 What could have been the maximum possible increase in combined cash balance of Chetan and Michael at the end of the fifth day? 

A. Rs 3700 

B. Rs 4000 

C. Rs 4700 

D. Rs 5000 

E. Rs 6000 

Answer: D. 

Explanation: 

Let at start no. of shares of both be 0 (No. of shares can be negative) and M and C. be no. of shares at the end of 5th day. Following are the possibilities in the changes in share prices over days : 

In the 8th possibility we can calculate that chetan ended up with 1300 and michael ended with 3700 cash . So the combined cash balance of Chetan and Michael at the end of the fifth day is 5000. Hence option D. 

Q. 40 If Michael ended up with 20 more shares than Chetan at the end of day 5, what was the price of the share at the end of day 3? 

A. Rs 90 

B. Rs 100 

C. Rs 110 

D. Rs 120 

E. Rs 130 

Answer: A. 

Explanation: 

Let at start no. of shares of both be 0 (No. of shares can be negative) and M and C. be no. of shares at the end of 5th day. Following are the possibilities in the changes in share prices over days : 

In the second possibility we can see that difference of no. of shares in 20 and at the end of the 3rd day the price was 90 .Hence option A. 

 

Instructions 

A significant amount of traffic flows from point S to point T in the one-way street network shown below. Points A, B, C, and D are junctions in the network, and the arrows mark the direction of traffic flow. The fuel cost in rupees for travelling along a street is indicated by the number adjacent to the arrow representing the street. – 

Motorists traveling from point S to point T would obviously take the route for which the total cost of traveling is the minimum. If two or more routes have the same least travel cost, then motorists are indifferent between them. Hence, the traffic gets evenly distributed among all the least cost routes. 

The government can control the flow of traffic only by levying appropriate toll at each junction. For example, if a motorist takes the route S-A-T (using junction A alone), then the total cost of travel would be Rs 14 (i.e., Rs 9 + Rs 5) plus the toll charged at junction A 

Q. 41 If the government wants to ensure that all motorists travelling from S to T pay the same amount (fuel costs and toll combined) regardless of the route they choose and the street from B to C is under repairs (and hence unusable), then a feasible set of toll charged (in rupees) at junctions A, B, C, and D respectively to achieve this goal is: 

A. 2,5,3,2 

B. 0,5,3,2 

C. 1,5,3,2 

D. 2,3,5,1 

E. 1,3,5,1 

Answer: C. 

Explanation: 

Let the toll charged at junctions A, B, C, and D be a,b,c and d respectively. Then the so that equal amount is collected through all route we have, 9+a+5=2+b+2+a+5=10+d+c=13+d. Then from the options only option C satisfies the above equality. hence option C 

Q. 42 If the government wants to ensure that no traffic flows on the street from D to T, while equal amount of traffic flows through junctions A and C, then a feasible set of toll charged (in rupees) at junctions A, B, C,and D respectively to achieve this goal is: 

A. 1,5,3,3 

B. 1,4,4,3 

C. 1,5,4,2 

D. 0,5,2,3 

E. 0,5,2,2 

Answer: E. 

Explanation: 

Let the toll charged at junctions A, B, C, and D be a,b,c and d respectively. Now since we want equal traffic through A and C, total cost through routes passing from A. and C should be equal. So we have (9+a+5) + (2+b+2+a+5) = (2+3+b+c+2) + (7+d+1+c+2) . Only option E. satisfies the above equality. 

Q. 43 If the government wants to ensure that all routes from S to T get the same amount of traffic, then a feasible set of toll charged (in rupees) at junctions A, B, C, and D respectively to achieve this goal is: 

A. 0, 5, 2, 2 

B. 0,5,4,1 

C. 1,5,3,3 

D. 1, 5, 3,2 

E. 1,5,4,2 

Answer: D. 

Explanation: 

Now the fuel cost along different routes are : 

SAT = 14 

SBAT = 9 

SBCT = 7 

SDCT = 10 

SDT = 13 

Now , if we consider option D. Total cost for all routes comes out to be the same which is 15. Hence option D. 

Q. 44 If the government wants to ensure that the traffic at S gets evenly distributed along streets from S to A, from S to B, and from S to D, then a feasible set of toll charged (in rupees) at junctions A, B, C, and D respectively to achieve this goal is: 

A. 0,5,4,1 

B. 0,5,2,2 

C. 1,5,3,3 

D. 1,5,3,2 

E. 0,4,3,2 

Answer: A. 

Explanation: 

Total cost = fuel cost + toll 

Total cost along SAT : 14+toll A 

Total cost along SBAT : 9+toll A+toll B 

Total cost along SDT : 13+toll D 

Now when option A is considered , total costs come out to be same. 

Hence option A is correct. 

Q. 45 The government wants to devise a toll policy such that the total cost to the commuters per trip is minimized. The policy should also ensure that not more than 70 per cent of the total traffic passes through junction B. The cost incurred by the commuter travelling from point S to point T under this policy will be: 

A. Rs 7 

B. Rs 9 

C. Rs 10 

D. Rs 13 

E. Rs 14 

Answer: C. 

Explanation: 

The costs of the routes are as given below: 

S – B. – C. – T = 7 

S – B. – A. – T = 9 

S – D. – C. – T = 10 

S – D. – T = 13 

S – A. -T = 14 

Hence now 100% of the traffic flows through S – B – C – T 

Now if we make the cost of traveling through S – B – C – T same as some other route not going through B, then the traffic will be equally distributed between these two routes. The lowest such route is S-D-C-T. The difference in cost = 3. Hence if we levy a toll of Rs.3 at B, the costs of SBCT and SBAT become 10,12 respectively and other routes are not affected. So 50% traffic flows through SBCT and 50% flows through SDCT . Hence cost in this policy = 10. 

 

Instructions 

K, L, M, N, P, Q, R, S, U and W are the only ten members in a department. There is a proposal to form a team from within the members of the department, subject to the following conditions: 

1. A. team must include exactly one among P,R and S. 

2. A. team must include either M or Q, but not both. 

3. If a team includes K, then it must also include L, and vice versa. 

4. If a team includes one among S, U and W, then it should also include the other two. 

5. L and N cannot be members of the same team. 

6. L and U cannot be members of the same team. 

The size of a team is defined as the number of members in the team. 

Q. 46 What could be the size of a team that includes K? 

A. 2 or 3 

B. 2 or 4 

C. 3 or 4 

D. Only 2 

E. Only 4 

Answer: E. 

Explanation: 

A. team which has K should have L also. 

Since L is there in the team, N and U should not be there in the team. Since U is not there in the team, S and W should not be there in the team. 

So, the team will have K, L, one of P and R and one of M or Q. 

So, the team size will be 4. 

Q. 47 In how many ways a team can be constituted so that the team includes N? 

A.

B.

C.

D.

E.

Answer: E. 

Explanation: 

Since N is in the team, L and K cannot be in the team. 

The team can have one of M and Q. So, 2 ways of selection. 

If the team has S, then it should have U and W as well. 

If the team has no S, then it should have one of P or R. 

So, the number of ways of forming the team is 2*(1+2) = 6 ways 

Q. 48 What would be the size of the largest possible team? 

A.

B.

C.

D.

E. cannot be determined 

Answer: D. 

Explanation: 

Out of P, R and S only 1 can be in the team. If S is there, U and W will also be there. So, P and R should not be in the team for its size to be maximum. 

Out of M and Q, only 1 can be there. 

If L is there in the team, N and U cannot be in the team. 

If L is not there in the team, then K is also not there in the team but N and U can be in the team. 

So, the maximum team size is 5 consisting of S, U, W, (M or Q), N. 

Q. 49 Who can be a member of a team of size 5? 

A.

B.

C.

D.

E.

Answer: C. 

Explanation: 

Out of P, R and S only 1 can be in the team. If S is there, U and W will also be there. So, P and R should not be in the team for its size to be maximum. 

Out of M and Q, only 1 can be there. 

If L is there in the team, N and U cannot be in the team. 

If L is not there in the team, then K is also not there in the team but N and U can be in the team. So, the maximum team size is 5 consisting of S, U, W, (M or Q), N. 

So, M can be a member of team size 5. 

Q. 50 Who cannot be a member of a team of size 3? 

A.

B.

C.

D.

E.

Answer: A. 

Explanation: 

If L is in the team, the team should include K also. The team should have one among P, R and S and one among M and Q. 

So, the team size cannot be 3 if L is in the team. 

Verbal 

Instructions 

Fifteen years after communism was officially pronounced dead, its spectre seems once again to be haunting Europe. Last month, the Council of Europe’s parliamentary assembly voted to condemn the “crimes of totalitarian communist regimes,” linking them with Nazism and complaining that communist parties are still “legal and active in some countries.” Now Goran Lindblad, the conservative Swedish MP behind the resolution, wants to go further. Demands that European Ministers launch a continent-wide anti-communist campaign – including school textbook revisions, official memorial days, and museums – only narrowly missed the necessary two-thirds majority. Mr. Lindblad pledged to bring the wider plans back to the Council of Europe in the coming months: 

He has chosen a good year for his ideological offensive: this is the 50’h anniversary of Nikita Khrushchev’s denunciation of Josef Stalin and the subsequent Hungarian uprising, which will doubtless be the cue for further excoriation of the communist record. Paradoxically, given that there is no communist government left in Europe outside Moldova, the attacks have if anything, become more extreme as time has gone on. A. clue as to why that might be can be found in the rambling report by Mr. Lindblad that led to the Council of Europe declaration. Blaming class struggle and public ownership, he explained “different elements of communist ideology such as equality or social justice still seduce many” and “a sort of nostalgia for communism is still alive.” Perhaps the real problem for Mr. Lindblad and his right-wing allies in Eastern Europe is that communism is not dead enough – and they will only be content when they have driven a stake through its heart. 

The fashionable attempt to equate communism and Nazism is in reality a moral and historical nonsense. Despite the cruelties of the Stalin terror, there was no Soviet Treblinka or Sorbibor, no extermination camps built to murder millions. Nor did the Soviet Union launch the most devastating war in history at a cost of more than 50 million lives – in fact it played the decisive role in the defeat of the German war machine. Mr. Lindblad and the Council of Europe adopt as fact the wildest estimates of those “killed by communist regimes” (mostly in famines) from the fiercely contested Black Book of Communism, which also underplays the number of deaths attributable to Hitler. But, in any case, none of this explains why anyone might be nostalgic in former communist states, now enjoying the delights of capitalist restoration. 

The dominant account gives no sense of how communist regimes renewed themselves after 1956 or why Western leaders feared they might overtake the capitalist world well into the 1960s. For all its brutalities and failures, communism in the Soviet Union, Eastern Europe, and elsewhere delivered rapid industrialization, mass education, job security, and huge advances in social and gender equality. Its existence helped to drive up welfare standards in the west, and provided a powerful counterweight to western global domination. 

It would be easier to take the Council of Europe’s condemnation of communist state crimes seriously if it had also seen fit to denounce the far bloodier record of European colonialism – which only finally came to an end in the 1970s. This was a system of racist despotism, which dominated the globe in Stalin’s time. And while there is precious little connection between the ideas of fascism and communism, there is an intimate link between colonialism and Nazism. The terms lebensraum and konzentrationslager were both first used by the German colonial regime in South-West Africa (now Namibia), which committed genocide against the Herero and Nama peoples and bequeathed its ideas and personnel directly to the Nazi parry. 

Around 10 million Congolese died as a result of Belgian forced labor and mass murder in the early twentieth century; tens of millions perished in avoidable or enforced famines in British-ruled India; up to a million Algerians died in their war for independence, while controversy now rages in France about a new law requiring teachers to put a positive spin on colonial history. Comparable atrocities were carried out by all European colonialists, but not a word of condemnation from the Council of Europe. Presumably, European lives count for more. 

No major twentieth century political tradition is without blood on its hands, but battles over history are more about the future than the past. Part of the current enthusiasm in official Western circles for dancing on the grave of communism is no doubt about relations with today’s Russia and China. But it also reflects a determination to prove there is no alternative to the new global capitalist order – and that any attempt to find one is bound to lead to suffering. With the new imperialism now being resisted in the Muslim world and Latin America, growing international demands for social justice and ever greater doubts about whether the environmental crisis can be solved within the existing economic system, the pressure for alternatives will increase. 

Q. 51 Among all the apprehensions that Mr. Goran Lindblad expresses against communism, which one gets admitted, although indirectly, by the author? 

A. There is nostalgia for communist ideology even if communism has been abandoned by most European nations. 

B. Notions of social justice inherent in communist ideology appeal to critics of existing systems. 

C. Communist regimes were totalitarian and marked by brutalities and large scale violence. 

D. The existing economic order is wrongly viewed as imperialistic by proponents of communism: 

E. Communist ideology is faulted because communist regimes resulted in economic failures. 

Answer: C. 

Explanation: 

From the 3rd para we find the sentences like ‘ … Mr. Lindblad and the Council of Europe adopt as fact the wildest estimates of those “killed by communist regimes” (mostly in famines) from..’ From here we can infer that among all apprehensions that Mr. Goran Lindblad expresses against communism this option C. is admitted indirectly. 

Q. 52 What, according to the author, is the real reason for a renewed attack against communism? 

A. Disguising the unintended consequences of the current economic order such as social injustice and environmental crisis. 

B. Idealising the existing ideology of global capitalism. 

C. Making communism a generic representative of all historical atrocities, especially those perpetrated by the European imperialists. 

D. Communism still survives, in bits and pieces, in the minds and hearts of people. 

E. Renewal of some communist regimes has led to the apprehension that communist nations might overtake the capitalists. 

Answer: B. 

Explanation: 

Options A, C and E are irrelevant according to the passage. 

Both B and D seem to be the answers, but the question asks for the real reason. 

If communism is not a threat, then it is not required to destroy it. So, D. cannot be the real reason. Hence, option B is the answer. 

Q. 53 The author cites examples of atrocities perpetrated by European colonial regimes in order to 

A. compare the atrocities committed by colonial regimes with those of communist regimes. 

B. prove that the atrocities committed by colonial regimes were more than those of communist regimes. 

C. prove that, ideologically, communism was much better than colonialism and Nazism. 

D. Neutralise the arguments of Mr. Lindblad and to point out that the atrocities committed by colonial regimes were more than those of communist regimes. . 

E. Neutralize the arguments of Mr. Lindblad and to argue that one needs to go beyond and look at the motives of these regimes. 

Answer: E. 

Explanation: 

Option A,B,C. are clearly not the answers and are out of context . Out of D. and E. , option E. gives perfect reason of why author cites examples of atrocities perpetrated by European colonial regimes. 

Q. 54 Why, according to the author, is Nazism closer to colonialism than it is to communism? 

A. Both colonialism and Nazism were examples of tyranny of one race over another. 

B. The genocides committed by the colonial and the Nazi regimes were of similar magnitude. 

C. Several ideas of the Nazi regime were directly imported from colonial regimes. 

D. Both colonialism and Nazism are based on the principles of imperialism. 

E. While communism was never limited to Europe, both the Nazis and the colonialists originated in Europe. 

Answer: A. 

Explanation: 

Author compares the motives behind all the atrocities and according to him colonialism and Nazism were examples of tyranny of one race over another. Hence option A is the answer. 

Q. 55 Which of the following cannot be inferred as a compelling reason for the silence of the Council of Europe on colonial atrocities? 

A. The Council of Europe being dominated by erstwhile colonialists. 

B. Generating support for condemning communist ideology. 

C. Unwillingness to antagonize allies by raking up an embarrassing past. 

D. Greater value seemingly placed on European lives. 

E. Portraying both communism and Nazism as ideologies to be condemned. 

Answer: D. 

Explanation: 

Except D, all the other options are compelling reasons for the silence of the Council of Europer over colonial atrocities. Option D is the compelling reason for condemnation of communism. 

Hence, option D is the answer. 

 

Instructions 

My aim is to present a conception of justice which generalizes and carries to a higher level of abstraction the familiar theory of the social contract. In order to do this we are not to think of the original contract as one to enter a particular society or to set up a particular form of government. Rather, the idea is that the principles of justice for the basic structure of society are the object of the original agreement. They are the principles that free and rational persons concerned to further their own interests would accept in an initial position of equality. These principles are to regulate all further agreements; they specify the kinds of social cooperation that can be entered into and the forms of government that can be established. This way of regarding the principles of justice, I shall call justice as fairness. Thus, we are to imagine that those who engage in social cooperation choose together, in one joint act, the principles which are to assign basic rights and duties and to determine the division of social benefits. Just as each person must decide by rational reflection what constitutes his good, that is, the system of ends which it is rational for him to pursue, so a group of persons must decide once and for all what is to count among them as just and unjust. The choice which rational men would make in this hypothetical situation of equal liberty determines the principles of justice. 

In ‘justice as fairness’, the original position is not an actual historical state of affairs. It is understood as a purely hypothetical situation characterized so as to lead to a certain conception of justice. Among the essential features of this situation is that no one knows his place in society, his class position or social status, nor does anyone know his fortune in the distribution of natural assets and abilities, his intelligence, strength, and the like. I shall even assume that the parties do not know their conceptions of the good or their special psychological propensities. The principles of justice are chosen behind a veil of ignorance. This ensures that no one is advantaged or disadvantaged in the choice of principles by the outcome of natural chance or the contingency of social circumstances. Since all are similarly situated and no one is able to design principles to favor his particular condition, the principles of justice are the result of a fair agreement or bargain. 

Justice as fairness begins with one of the most general of all choices which persons might make together, namely, with the choice of the first principles of a conception of justice which is to regulate all subsequent criticism and reform of institutions. Then, having chosen a conception of justice, we can suppose that they are to choose a constitution and a legislature to enact laws, and so on, all in accordance with the principles of justice initially agreed upon. Our social situation is just if it is such that by this sequence of hypothetical agreements we would have contracted into the general system of rules which defines it. Moreover, assuming that the original position does determine a set of principles, it will then be true that whenever social institutions satisfy these principles, those engaged in them can say to one another that they are cooperating on terms to which they would agree if they were free and equal persons whose relations with respect to one another were fair. They could all view their arrangements as meeting the stipulations which they would acknowledge in an initial situation that embodies widely accepted and reasonable constraints on the choice of principles. The general recognition of this fact would provide the basis for a public acceptance of the corresponding principles of justice. No society can, of course, be a scheme of cooperation which men enter voluntarily in a literal sense; each person finds himself placed at birth in some particular position in some particular society, and the nature of this position materially affects his life prospects. Yet a society satisfying the principles of justice as fairness comes as close as a society can to being a voluntary scheme, for it meets the principles which free and equal persons would assent to under circumstances that are fair. 

Q. 56 A just society, as conceptualized in the passage, can be best described as: 

A. A Utopia in which everyone is equal and no one enjoys any privilege based on their existing positions and powers. ‘ 

B. A hypothetical society in which people agree upon principles of justice which are fair. 

C. A society in which principles of justice are not based on the existing positions and powers of the individuals. 

D. A society in which principles of justice are fair to all. 

E. A hypothetical society in which principles of justice are not based on the existing positions and powers of the individuals. 

Answer: C. 

Explanation: 

A. just society is not a utopia according to the passage => option A is wrong. 

The society mentioned in the passage is not hypothetical => options B and E are wrong. 

Between option C and option D, option C fits better to be the answer because it is similar to the authors views in the passage whereas option D is too generalized. 

Hence, C is the answer. 

Q. 57 The original agreement or original position in the passage has been used by the author as: 

A. A hypothetical situation conceived to derive principles of justice which are not influenced by position, status and condition of individuals in the society. 

B. A hypothetical situation in which every individual is equal and no individual enjoys any privilege based on the existing positions and powers. . 

C. A hypothetical situation to ensure fairness of agreements among individuals in society. 

D. An imagined situation in which principles of justice would have to be fair. 

E. An imagined situation in which fairness is the objective of the principles of justice to ensure that no individual enjoys any privilege based on the existing positions and powers. 

Answer: A. 

Explanation: 

It is clearly mentioned in the 2nd para of the passage that the original agreement is a hypothetical situation and is not influenced by position or status of the individual. Refer to the sentence “It is understood as a purely hypothetical situation………intelligence, strength and the like.” 

Q. 58 Which of the following best illustrates the situation that is equivalent to choosing ‘the principles of justice’ behind a ‘veil of ignorance’? 

A. The principles of justice are chosen by businessmen, who are marooned on an uninhabited island after a shipwreck, but have some possibility of returning. 

B. The principles of justice are chosen by a group of school children whose capabilities are yet to develop. 

C. The principles of justice are chosen by businessmen, who are marooned on an uninhabited island after a shipwreck and have no possibility of returning. 

D. The principles of justice are chosen assuming that such principles will govern the lives of the rule makers only in their next birth if the rule makers agree that they will be born again. 

E. The principles of justice are chosen by potential immigrants who are unaware of the resources necessary to succeed in a foreign country. 

Answer: D. 

Explanation: 

In this second paragraph, refer to the following lines:”This ensures that no one is advantaged or disadvantaged in the choice of principles by the outcome of natural chance or the contingency of social circumstances.” If the rule makers agree that they will be born again and these principals will govern in the next birth, then they are framing the laws in the veil of ignorance. 

Q. 59 Why, according to the passage, do principles of justice need to be based on an original agreement? 

A. Social institutions and laws can be considered fair only if they conform to principles of justice. 

B. Social institutions and laws can be fair only if they are consistent with the principles of justice as initially agreed upon. 

C. Social institutions and laws need to be fair in order to be just. 

D. Social institutions and laws evolve fairly only if they are consistent with the principles of justice as initially agreed upon. 

E. Social institutions and laws conform to the principles of justice as initially agreed upon. 

Answer: B. 

Explanation: 

Refer to the sentence “Our social situation is just…………general system of rules which define it.” Option B. justifies this whereas other options deviate from this statement. 

Q. 60 Which of the following situations best represents the idea of justice as fairness, as argued in the passage? 

A. All individuals are paid equally for the work they do. 

B. Everyone is assigned some work for his or her livelihood. 

C. All acts of theft are penalized equally. 

D. All children are provided free education in similar schools. 

E. All individuals are provided a fixed sum of money to take care of their health. 

Answer: D. 

Explanation: 

The author says that the initial equality and the veil of ignorance are the most important. 

Among the given options, only option D. fits to be the answer because of the initail equality it poses. Hence, option D. is the answer. 

 

Instructions 

Our propensity to look out for regularities, and to impose laws upon nature, leads to the psychological phenomenon of dogmatic thinking or, more generally, dogmatic behaviour: we expect regularities everywhere and attempt to find them even where there are none; events which do not yield to these attempts we are inclined to treat as a kind of `background noise’; and we stick to our expectations even when they are inadequate and we ought to accept defeat. This dogmatism is to some extent necessary. It is demanded by a situation which can only be dealt with by forcing our conjectures upon the world. Moreover, this dogmatism allows us to approach a good theory in stages, by way of approximations: if we accept defeat too easily, we may prevent ourselves from finding that we were very nearly right. 

It is clear that this dogmatic attitude; which makes us stick to our first impressions, is indicative of a strong belief; while a critical attitude, which is ready to modify its tenets, which admits doubt and demands tests, is indicative of a weaker belief. Now according to Hume’s theory, and to the popular theory, the strength of a belief should be a product of repetition; thus it should always grow with experience, and always be greater in less primitive persons. But dogmatic thinking, an uncontrolled wish to impose regularities, a manifest pleasure in rites and in repetition as such, is characteristic of primitives and children; and increasing experience and maturity sometimes create an attitude of caution and criticism rather than of dogmatism. 

My logical criticism of Hume’s psychological theory, and the considerations connected with it, may seem a little removed from the field of the philosophy of science. But the distinction between dogmatic and critical thinking, or the dogmatic and the critical attitude, brings us right back to our central problem. For the dogmatic attitude is clearly related to the tendency to verify our laws and schemata by seeking to apply them and to confirm them, even to the point of neglecting refutations, whereas the critical attitude is one of readiness to change them – to test them; to refute them; to falsify them, if possible. This suggests that we may identify the critical attitude with the scientific attitude, and the dogmatic attitude with the one which we have described as pseudo-scientific. It further suggests that genetically speaking the pseudo-scientific attitude is more primitive than, and prior to, the scientific attitude: that it is a pre-scientific attitude. And this primitivity or priority also has its logical aspect. For the critical attitude is not so much opposed to the dogmatic attitude as super-imposed upon it: criticism must be directed against existing and influential beliefs in need of critical revision – in other words, dogmatic beliefs. A. critical attitude needs for its raw material, as it were, theories or beliefs which are held more or less dogmatically. 

Thus, science must begin with myths, and with the criticism of myths; neither with the collection of observations, nor with the invention of experiments, but with the critical discussion of myths, and of magical techniques and practices. The scientific tradition is distinguished from the pre-scientific tradition in having two layers. Like the latter, it passes on its theories; but it also passes on a critical attitude towards them. The theories are passed on, not as dogmas, but rather with the challenge to discuss them and improve upon them. 

The critical attitude, the tradition of free discussion of theories with the aim of discovering their weak spots so that they may be improved upon, is the attitude of reasonableness, of rationality. From the point of view here developed, all laws, all theories, remain essentially tentative, or conjectural, or hypothetical, even when we feel unable to doubt them any longer. Before a theory has been refuted we can never know in what way it may have to be modified. 

Q. 61 In the context of science, according to the passage, the interaction of dogmatic beliefs and critical attitude can be best described as: 

A. A. duel between two warriors in which one has to die. 

B. The effect of a chisel on a marble stone while making a sculpture. 

C. The feedstock (natural gas) in fertilizer industry being transformed into fertilizers. 

D. A. predator killing its prey. 

E. The effect of fertilizers on a sapling. 

Answer: B. 

Explanation: 

It has been stated in the passage that “For the critical attitude is not so much opposed to the dogmatic attitude as super-imposed upon it: criticism must be directed against existing and influential beliefs in need of critical revision – in other words, dogmatic beliefs. A. critical attitude needs for its raw material, as it were, theories or beliefs which are held more or less dogmatically.” 

From this we can infer that critical attitude is not opposed to dogmatic beliefs. Rather it uses dogmatic beliefs as raw material. Hence, we can eliminate options A, D. and E. 

From the passage, we can infer that the critical attitude acts on dogmatic beliefs to refine them. Hence, option B. is more appropriate. 

Q. 62 According to the passage, the role of a dogmatic attitude of dogmatic behaviour in the development of science is 

A. critical and important, as, without it, initial hypotheses or conjectures can never be made. 

B. positive, as conjectures arising out of our dogmatic attitude become science. 

C. negative, as it leads to pseudo-science. 

D. neutral, as the development of science is essentially because of our critical attitude. 

E. inferior to critical attitude, as a critical attitude leads to the attitude of reasonableness and rationality. 

Answer: A. 

Explanation: 

Options C,D,E. do not hold true. 

Amongst option A. and B,option A. is more appropriate.Refer to the 3rd paragraph. It is written that “the pseudo-scientific attitude is more primitive than, and prior to, the scientific attitude: that it is a pre-scientific attitude. And this primitivity or priority also has its logical aspect. For the critical attitude is not so much opposed to the dogmatic attitude as super-imposed upon it: criticism must be directed against existing and influential beliefs in need of critical revision — in other words, dogmatic beliefs. A. critical attitude needs for its raw material, as it were, theories or beliefs which are held more or less dogmatically”. 

Q. 63 Dogmatic behaviour, in this passage, has been associated with primitives and children. Which of the following best describes the reason why the author compares primitives with children? 

A. Primitives are people who are not educated, and hence can be compared with children,who have not yet been through school. . 

B. Primitives are people who, though not modern, are as innocent as children. 

C. Primitives are people without a critical attitude, just as children are. 

D. Primitives are people in the early stages of human evolution; similarly, children are in the early stages of their lives. 

E. Primitives are people who are not civilized enough, just as children are not. . 

Answer: D. 

Explanation: 

Refer to the lines: 

It further suggests that genetically speaking the pseudo-scientific attitude is more primitive than, and prior to, the scientific attitude: that it is a pre-scientific attitude. And this primitivity or priority also has its logical aspect. For the critical attitude is not so much opposed to the dogmatic attitude as super-imposed upon it. It clearly illustates option 4. 

Data Interpretation 

Q. 64 Which of the following statements best supports the argument in the passage that a critical attitude leads to a weaker belief than a dogmatic attitude does? 

A. A critical attitude implies endless questioning, and, therefore, it cannot lead to strong beliefs. 

B. A critical attitude, by definition, is centered on an analysis of anomalies and “noise”. 

C. A critical attitude leads to questioning everything, and in the process generates “noise” without any conviction. 

D. A critical attitude is antithetical to conviction, which is required for strong beliefs. 

E. A critical attitude leads to questioning and to tentative hypotheses. . 

Answer: E. 

Explanation: 

Refer to the last 3 lines 

“From the point of view here developed, all laws, all theories, remain essentially tentative, or conjectural, or hypothetical, even when we feel unable to doubt them any longer. Before a theory has been refuted we can never know in what way it may have to be modified.” 

Option E clearly supports this argument. 

Option A contains a distortion of “endless questioning”. A critical attitude requires questioning but not endless questioning. 

Option B has not been implied anywhere in the passage. 

The first part of option C is true but the second part is false. The result of a critical attitude is not noise but tested beliefs that are subject to change. 

Option D. has not been implied anywhere in the passage. 

Q. 65 According to the passage, which of the following statements best describes the difference between science and pseudo-science? : 

A. Scientific theories or hypothesis are tentatively true whereas pseudo-sciences are always true. 

B. Scientific laws and theories are permanent and immutable whereas pseudo-sciences are contingent on the prevalent mode of thinking in a society. 

C. Science always allows the possibility of rejecting a theory or hypothesis, whereas pseudo-sciences seek to validate their ideas or theories. 

D. Science focuses on anomalies and exceptions so that fundamental truths can be uncovered, whereas pseudo sciences focus mainly on general truths. 

E. Science progresses by collection of observations or by experimentation, whereas pseudo-sciences do not worry about observations and experiments. 

Answer: C. 

Explanation: 

Refer to these lines 

“But the distinction between dogmatic and critical thinking, or the dogmatic and the critical attitude, brings us right back to our central problem. For the dogmatic attitude is clearly related to the tendency to verify our laws and schemata by seeking to apply them and to confirm them, even to the point of neglecting refutations, whereas the critical attitude is one of readiness to change them — to test them; to refute them; to falsify them, if possible. This suggests that we may identify the critical attitude with the scientific attitude, and the dogmatic attitude with the one which we have described as pseudo-scientific.” 

Option C. is the main point of the above paragraph. Hence, option C is the answer. 

 

Instructions 

For the following questions answer them individually 

Q. 66 Facts, which deal with pieces of information that one has heard, seen or read, and which are open to discovery or verification (the answer option indicates such a statement with an `F’). 

Inferences,which are conclusions drawn about the unknown, on the basis of the known (the answer option indicates such a statement with an `I’). 

Judgements which are opinions that imply approval or disapproval of persons, objects, situations and occurrences in the past, the present or the future (the answer option indicates such a statement with a J) 

Select the answer option that best describes the set of four statements. 

1.So much of our day-to-day focus seems to be on getting things done, trudging our way through the tasks of living – it can feel like a treadmill that gets you nowhere; where is the childlike joy? 

2.We are not doing the things that make us happy; that which brings us joy; the things that we cannot wait to do because we enjoy them so much. 

3.This is the stuff that joyful living is made of – identifying your calling and committing yourself wholeheartedly to it. 

4.When this happens, each moment becomes a celebration of you; there is a rush of energy that comes with feeling completely immersed in doing what you love most. 

A. IIIJ 

B. IFIJ 

C. JFJJ 

D. JJJJ 

E. JFII 

Answer: D. 

Explanation: 

The first statement says “It can feel like a treadmill that gets you nowhere”. This is an opinion and hence the statement is a judgement. 

Statement 2 is also a personal opinion and hence a judgement. 

Similarly, statements 3 and 4 are also opinions and hence can be categorised as judgements. 

Q. 67 Facts, which deal with pieces of information that one has heard, seen or read, and which are open to discovery or verification (the answer option indicates such a statement with an `F’). 

Inferences,which are conclusions drawn about the unknown, on the basis of the known (the answer option indicates such a statement with an `I’). 

Judgements which are opinions that imply approval or disapproval of persons, objects, situations and occurrences in the past, the present or the future (the answer option indicates such a statement with a J) 

Select the answer option that best describes the set of four statements. 

1.Given the poor quality of service in the public sector, the HIV/AIDS affected should be switching to private initiatives that supply antiretroviral drugs (ARVs) at a low cost. 

2.The government has been supplying free drugs since 2004, and 35000 have benefited up to now – though the size of the affected population is 150 times this number. 

3.The recent initiatives of networks and companies like AIDSCare Network, Emcure, Reliance-Cipla-CII, would lead to availability of much-needed drugs to a larger number of affected people. 

4.But how ironic it is that we should face a perennial shortage of drugs when India is one of the world’s largest suppliers of generic drugs to the developing world. 

A. JFIJ 

B. JIIJ 

C. IFIJ 

D. IFFJ 

E. JFII 

Answer: A. 

Explanation: 

The first statement is an opinion since it says “Given the poor quality…” and “…should be switching”. The second statement is clearly a fact. The fourth statement is a judgement since the words “…ironic it is…” indicate the opinion of the author and not something that can be verified by facts. The answer is option A 

Q. 68 Facts, which deal with pieces of information that one has heard, seen or read, and which are open to discovery or verification (the answer option indicates such a statement with an `F’). 

Inferences,which are conclusions drawn about the unknown, on the basis of the known (the answer option indicates such a statement with an `I’). 

Judgements which are opinions that imply approval or disapproval of persons, objects, situations and occurrences in the past, the present or the future (the answer option indicates such a statement with a J) 

Select the answer option that best describes the set of four statements. 

1.According to all statistical indications, the Sarva Shiksha Abhiyan has managed to keep pace with its ambitious goals. 

2.The Mid-day Meal Scheme has been a significant incentive for the poor to send their little ones to school, thus establishing the vital link between healthy bodies and healthy minds. 

3.Only about 13 million children in the age group of 6 to 14 years are out of school. 

4.The goal of universalisation of elementary education has to be a prerequisite for the evolution and development of our country. 

A. IIFJ 

B. JIIJ 

C. IJFJ 

D. IJFI 

E. JIFT 

Answer: C. 

Explanation: 

Statement 1 is an inference, because it is drawing a conclusion (Sarva Shiksha Abhiyan has managed to keep pace with its ambitious goals) based on a fact (all statistical indications). 

Statement 2 is a judgement since “the vital link between healthy bodies and healthy minds” is not something that can be investigated for data. Statement 3 is clearly a fact. Statement 4 is an opinion, and hence it is a judgement. 

Q. 69 Facts, which deal with pieces of information that one has heard, seen or read, and which are open to discovery or verification (the answer option indicates such a statement with an `F’). 

Inferences,which are conclusions drawn about the unknown, on the basis of the known (the answer option indicates such a statement with an `I’). 

Judgements which are opinions that imply approval or disapproval of persons, objects, situations and occurrences in the past, the present or the future (the answer option indicates such a statement with a J) 

Select the answer option that best describes the set of four statements. 

1.We should not be hopelessly addicted to an erroneous belief that corruption in India is caused by the crookedness of Indians. 

2.The truth is that we have more red tape – we take eighty-nine days to start a small business, Australians take two. 

3.Red tape leads to corruption and distorts a people’s character. 

4.Every red tape procedure is a point of contact with an official, and such contacts have the potential to become opportunities for money to change hands. 

A. JFIF 

B. JFJJ 

C. JIJF 

D. IFJF 

E. JFJI 

Answer: E. 

Explanation: 

Statement 1 expresses the personal opinion of the author and hence it is a judgement. Statement 2 is a fact. Statement 3 is again an opinion of the author and hence is a judgement. Statement 4 is an inference because it draws a conclusion (“has the potential…”) based on a fact (“Every red tape procedure is a point of contact with an official…”). Option E. is the answer. 

Q. 70 Facts, which deal with pieces of information that one has heard, seen or read, and which are open to discovery or verification (the answer option indicates such a statement with an `F’). 

Inferences,which are conclusions drawn about the unknown, on the basis of the known (the answer option indicates such a statement with an `I’). 

Judgements which are opinions that imply approval or disapproval of persons, objects, situations and occurrences in the past, the present or the future (the answer option indicates such a statement with a J) 

Select the answer option that best describes the set of four statements. 

1.Inequitable distribution of all kinds of resources is certainly one of the strongest and most sinister sources of conflict. 

2.Even without war, we know that conflicts continue to trouble us – they only change in character. 

3.Extensive disarmament is the only insurance for our future; imagine the amount of resources that can be released and redeployed. 

4.The economies of the industrialized western world derive 20% of their income from the sale of all kinds of arms. 

A. IJJI 

B. JIJF 

C. IIJF 

D. JIIF 

E. IJIF 

Answer: B. 

Explanation: 

Statements 1 and 3 are judgements because they are both opinions of the author. This eliminates all options except B. Hence B is the right answer. 

Q. 71 From the given options, choose the sentence that completes the paragraph in the most appropriate way. 

I am sometimes attacked for imposing `rules’. Nothing could be further from the truth: I hate rules. All I do is report on how consumers react to different stimuli. I may say to a copywriter, “Research shows that commercials with celebrities are below average in persuading people to buy products. Are you sure you want to use a celebrity?” Call that a rule? Or I may say to an art director, “Research suggests that if you set the copy in black type on a white background, more people will read it than if you set it in white type on a black background.” 

A. Guidance based on applied research can hardly qualify as `rules’. 

B. Thus, all my so called `rules’ are rooted in applied research. . 

C. A. suggestion perhaps, but scarcely a rule. 

D. Such principles are unavoidable if one wants to be systematic about consumer behaviour. 

E. Fundamentally it is about consumer behaviour – not about celebrities or type settings. 

Answer: C. 

Explanation: 

The last sentence should talk about the statement that the author made to the art director. According to the author, the statement is not a rule. So, option a) and c) are applicable. Between these two, option c) is better because it specifically talks about this statement and not about “rules” in general. So, c) is the best concluding sentence. 

Q. 72 From the given options, choose the sentence that completes the paragraph in the most appropriate way. 

Relations between the factory and the dealer are distant and usually strained as the factory tries to force cars on the dealers to smooth out production. Relations between the dealer and the customer are equally strained because dealers continuously adjust prices – make deals – to adjust demand with supply while maximizing profits. This becomes a system marked by ‘a lack of long-term commitment’ on either side, which maximizes feelings of mistrust. In order to maximize their bargaining positions, everyone holds back information – the dealer about the product and the consumer about his true desires. 

A. As a result, `deal making’ becomes rampant, without concern for customer satisfaction. 

B. As a result, inefficiencies creep into the supply chain. 

C. As a result, everyone treats the other as an adversary, rather than as an ally. 

D. As a result, fundamental innovations are becoming scarce in the automobile industry. 

E. As a result, everyone loses in the long run. 

Answer: E. 

Explanation: 

Option a) talks only about one part of the chain consisting of factory, dealer and customer. Option b) is far-fetched. Options c) and d) are out of scope. Option e) completes the para in the best possible way. 

Q. 73 From the given options, choose the sentence that completes the paragraph in the most appropriate way. 

In the evolving world order, the comparative advantage of the United States lies in its military force: Diplomacy and international law have always been regarded as annoying encumbrances, unless they can be used to advantage against an enemy. Every active player in world affairs professes to seek only peace and to prefer negotiation to violence and coercion. 

A. However, diplomacy has often been used as a mask by nations which intended to use force. 

B. However, when the veil is lifted, we commonly see that diplomacy is understood as a disguise for the rule of force. 

C. However, history has shown that many of these nations do not practice what they profess. 

D. However, history tells us that peace is professed by those who intend to use violence. 

E. However, when unmasked, such nations reveal a penchant for the use of force. 

Answer: B. 

Explanation: 

The last sentence in the paragraph should counter the statement made by the nations as well as talk about diplomacy, to bring the para to a meaningful conclusion. This is best captured in option b), which says, when the veil is lifted, diplomacy is often used as a disguise for the rule of force. 

Q. 74 From the given options, choose the sentence that completes the paragraph in the most appropriate way. 

Age has a curvilinear relationship with the exploitation of opportunity. Initially, age will increase the likelihood that a person will exploit an entrepreneurial opportunity because people gather much of the knowledge necessary to exploit opportunities over the course of their lives, and because age provides credibility in transmitting that information to others. However, as people become older, their willingness to bear risks declines, their opportunity costs rise, and they become less receptive to new information. 

A. As a result, people transmit more information rather than experiment with new ideas as they reach an advanced age. 

B. As a result, people are reluctant to experiment with new ideas as they reach an advanced age. 

C. As a result, only people with lower opportunity costs exploit opportunity when they reach an advanced age. 

D. As a result, people become reluctant to exploit entrepreneurial opportunities when they reach an advanced age. 

E. As a result, people depend on credibility rather than on novelty as they reach an advanced age. 

Answer: D. 

Explanation: 

The para talks about the relationship between age and entrepreneurial spirit. When people are young, they tend to believe that age brings in more credibility in transmitting information to others. However, as people age, they become less receptive to information and their opportunity costs rise. Hence, it is very likely that they become reluctant to exploit entrepreneurial opportunities as they age. Statement D, therefore, is the best concluding statement. 

Q. 75 From the given options, choose the sentence that completes the paragraph in the most appropriate way. 

We can usefully think of theoretical models as maps, which help us navigate unfamiliar territory. The most accurate map that it is possible to construct would be of no practical use whatsoever, for it would be an exact replica, on exactly the same scale, of the place where we were. Good maps pull out the most important features and throw away a huge amount of much less valuable information. Of course, maps can be bad as well as good – witness the attempts by medieval Europe to produce a map of the world. In the same way, a bad theory, no matter how impressive it may seem in principle, does little or nothing to help us understand a problem. 

A. But good theories, just like good maps, are invaluable, even if they are simplified. 

B. But good theories, just like good maps, will never represent unfamiliar concepts in detail. 

C. But good theories, just like good maps, need to balance detail and feasibility of representation. 

D. But good theories, just like good maps, are accurate only at a certain level of abstraction. 

E. But good theories, just like good maps, are useful in the hands of a user who knows their limitations. 

Answer: A. 

Explanation: 

The para talks about good maps throwing away a large amount of non-essential information and retaining more important information. They are also simplified as a result. So, the last sentence should be about good theories being simplified and more valuable. Option a) captures the essence in the best way. 

CAT Previous Year Paper 2005

CAT 2005

Section

Questions

Marks

Quantitative Aptitude

30 Questions (1 – 30)

50

Reading Comprehension

12 Questions

20

Verbal Ability

18 Questions

30

Data Interpretation

11 Questions

15

Logical Reasoning

19 Questions

35

Q. 1 If x = (16^3+17^3+18^3+19^3), then x divided by 70 leaves a remainder of

A. 0

B. 1

C. 69

D. 35

 

Q. 2 A chemical plant has four tanks (A, B, C and D), each containing 1000 litres of a chemical. The chemical is being pumped from one tank to another as follows.Question:Q. 2 A chemical plant has four tanks (A, B, C and D), each containing 1000 litres of a chemical. The chemical is being pumped from one tank to another as follows.

From A to B @ 20 litres/minute

From C to A @ 90 litres/minute

From A to D @ 10 litres/minute

From C to D @ 50 litres/minute

From B to C @ 100 litres/minute

From D to B @ 110 litres/minute

Which tank gets emptied first, and how long does it take (in minutes) to get empty after pumping starts?

A. A, 16.66

B. C, 20

C. D, 20

D. D, 25

 

Q. 3 Two identical circles intersect so that their centers, and the points at which they intersect, form a square of side 1 cm. The area in sq. cm of the portion that is common to the two circles is

A. π/4

B. (π/2) – 1

C. π/5

D. √2 – 1

 

Q. 4 A jogging park has two identical circular tracks touching each other, and a rectangular track enclosing the two circles. The edges of the rectangles are tangential to the circles. Two friends, A and B, start jogging simultaneously form the point where one of the circular tracks touches the smaller side of the rectangular track. A jogs along the rectangular track, while B jogs along the two circular tracks in a figure of eight. Approximately, how much faster than A does B have to run, so that they take the same time to return to their starting point?

A. 3.88%

B. 4.22%

C. 4.44%

D. 4.72%

 

Q. 5 In a chess competition involving some boys and girls of a school, every student had to play exactly one game with every other student. It was found that in 45 games both the players were girls, and in 190 games both were boys. The number of games in which one player was a boy and the other was a girl is

A. 200

B. 216

C. 235

D. 256

 

Questions: 6 – 7

Ram and Shyam run a race between points A and B, 5 km apart, Ram starts at 9 a.m from A at a speed of 5 km/hr, reaches B, and returns to A at the same speed, Shyam starts at 9:45 a.m. from A at a speed of 10 km/hr, reaches B and comes back to A at  the same speed.

Q. 6 At what time do Ram and Shyam first meet each other?

A. 10 a.m

B. 10.10 a.m

C. 10.20 a.m

D. 10.30 a.m

 

Q. 7 At what time does Shyam over take Ram?

A. 10.20 a.m

B. 10.30 a.m

C. 10.40 a.m

D. 10.50 a.m

 

Q. 8 If R = (30^65 – 29^65) / 30^64 + 29^64, then

A. 0 < R ≤ 0.1

B. 0.1 < R ≤ 0.5

C. 0.5 < R ≤ 1.0

D. R > 1.0

 

Q. 9 What is the distance in cm between two parallel chords of lengths 32 cm and 24 cm in a circle of radius 20 cm?

A. 1 or 7

B. 2 or 14

C. 3 or 21

D. 4 or 28

 

Q. 10 For which value of k does the following pair of equations yield a unique solution of x such that the solution is positive?

x^2 – y^2 = 0

(x – k)^2 + y^2 = 1

A. 2

B. 0

C. √2

D. -√2

 

Q. 11 Let n! = 1 x 2 x 3 x ……………. x n for integer n ≥ 1. If p = 1! + (2 x 2!) + (3 x 3!) + ……… + (10 x 10!), then p + 2 when divided by 11! Leaves a remainder of

A. 10

B. 0

C. 7

D. 1

 

Q. 12 Consider a triangle drawn on the X-Y plane with its three vertices of (41, 0), (0, 41) and (0, 0), each vertex being represented by its (X, Y) coordinates. The number of points with integer coordinates inside the triangle (excluding all the points on the boundary) is

A. 780

B. 800

C. 820

D. 741

 

Q. 13 The digits of a three-digit number A are written in the reverse order to form another threedigit number B. If B > A and B-A is perfectly divisible by 7, then which of the following is necessarily true?

A. 100 < A < 299

B. 106 < A < 305

C. 112 < A < 311

D. 118 < A < 317

 

Q. 14 If a1 = 1 and a(n+1) – 3a(n) + 2 = 4(n) for every positive integer n, then a100 equals

A. 3^99 – 200

B. 3^99 + 200

C. 3^100 – 200

D. 3^100 + 200

 

Q. 15 Let S be the set of five-digit numbers formed by digits 1, 2, 3, 4 and 5, using each digit exactly once such that exactly two odd position are occupied by odd digits. What is the sum of the digits in the rightmost position of the numbers in S?

A. 228

B. 216

C. 294

D. 192

 

Q. 16 The rightmost non-zero digits of the number 30^2720 is

A. 1

B. 3

C. 7

D. 9

 

Q. 17 Four points A, B, C and D lie on a straight line in the X-Y plane, such that AB = BC = CD, and the length of AB is 1 metre. An ant at A wants to reach a sugar particle at D. But there are insect repellents kept at points B and C. the ant would not go within one metre of any insect repellent. The minimum distance in metres the ant must traverse to reach the sugar particle is

A. 3√2

B. 1 + π

C. 4π/3

D. 5

 

Q. 18 If x ≥ y and y > 1, then the value of the expression logx (x/y) + logy (y/x) can never be

A. -1

B. -0.5

C. 0

D. 1

 

Q. 19 For a positive integer n, let p(n) denote the product of the digits of n and s(n) denote the sum of the digits of n. The number of integers between 10 and 1000 for which p(n) + s(n) = n is

A. 81

B. 16

C. 18

D. 9

 

Q. 20 Rectangular tiles each of size 70 cm by 30 cm must be laid horizontally on a rectangular floor of size 110 cm by 130 cm, such that the tiles do not overlap. A tile can be placed in any orientation so long as its edges are parallel to the edges of the floor. No tile should overshoot any edge of the floor. The maximum number of tiles that can be accommodated on the floor is

A. 4

B. 5

C. 6

D. 7

 

Q. 21 In the X-Y plane, the area of the region bounded by the graph |x + y| + |x – y| = 4 is

A. 8

B. 12

C. 16

D. 20

 

Q. 22 In the following figure (1), the diameter of the circle is 3 cm. AB and MN are two diameters such that MN is perpendicular to AB. In addition, CG is perpendicular to AB such that AE : EB = 1 : 2, and DF is perpendicular to MN such that NL : LM = 1 : 2. The length of DH in cm is

A. 2√2 – 1

B. (2√2 – 1)/2

C. (3√2 – 1)/2

D. (2√2 – 1)/3

 

Q. 23 Consider the triangle ABC shown in the figure (1) where BC = 12 cm, DB = 9 cm, CD = 6 cm and ∠BCD = ∠BAC.

What is the ratio of the perimeter of ΔADC to that to the ΔBDC?

A. 7/9

B. 8/9

C. 6/9

D. 5/9

 

Q. 24 P, Q, S and R are points on the circumference of a circle of radius r, such that PQR is an equilateral triangle and PS is a diameter of the circle. What is the perimeter of the quadrilateral PQSR?

A. 2r (1 + √3)

B. 2r (2 + √3)

C. r (1 + √5)

D. 2r + √3

 

Q. 25 Let S be a set of positive integers such that every element n of S satisfies the conditions. i. 1000 ≤ n ≤ 1200

ii. Every digit in n is odd

Then how many elements of S are divisible by 3?

A. 9

B. 10

C. 11

D. 12

 

Q. 26 Let x = √4 + √4 – √4 + √4 – ……… to infinity. Then x equals

A. 3

B. (√13 – 1)/2

C. (√13 + 1)/2

D. √13

 

Q. 27 Let g(x) be a function such that g(x + 1) + g(x – 1) = g(x) for every real x. Then for what value of p is the relation g(x + p) = g(x) necessarily true for every real x?

A. 5

B. 3

C. 2

D. 6

 

Q. 28 A telecom service provider engages male and female operators for answering 1000 calls per day. A male operator can handle 40 calls per day whereas a female operator can handle 50 calls per day. The male and the female operators get a fixed wage of Rs. 250 and Rs. 300 per day respectively. In addition, a male operator gets Rs. 15 per call he answers and female operator gets Rs. 10 per call she answers. To minimize the total cost, how many male operators should the service provider employ assuming he has to employ more than 7 of the 12 female operators available for the job?

A. 15

B. 14

C. 12

D. 10

 

Q. 29 Three Englishmen and three Frenchmen work for the same company. Each of them knows a secret not known to others. They need to exchange these secrets over person-to-person phone calls so that eventually each person knows all six secrets. None of the Frenchmen knows English, and only one Englishman knows French. What is the minimum number of phone calls needed for the above purpose?

A. 5

B. 10

C. 9

D. 15

 

Q. 30  A rectangular floor is fully covered with square tiles of identical size. The tiles on the edges are white and the tiles in the interior are red. The number of white tiles is the same as the number of red tiles. A possible value of the number of tiles along one edge of the floor is

A. 10

B. 12

C. 14

D. 16

 

Questions: 31 – 34

A game of strategy, as currently conceived in game theory, is a situation in which two or more “players” make choices among available alternatives (moves). The totality of choices determines the outcomes of the game, and it is assumed that the rank order of preferences for the outcomes is different for different players. Thus the “interests” of the players are generally in conflict. Whether these interests are diametrically opposed or only partially opposed depends on the type of game. Psychologically, most interesting situations arise when the interests of the players are partly coincident and partly opposed, because then one can postulate not only a conflict among the players but also inner conflicts within the players. Each is torn between a tendency to cooperate, so as to promote the common interests, and a tendency to compete, so as to enhance his own individual interests.

Internal conflicts are always psychologically interesting. What we vaguely call “interesting” psychology is n very great measure the psychology of inner conflict. Inner conflict is also held to be an important component of serious literature as distinguished from less serious genres. The classical tragedy, as well as the serious novel reveals the inner conflict of central figures. The superficial adventure story on the other hand, depicts only external conflict; that is, the threats to the person with whom the reader (or viewer) identifies stem in these stories exclusively from external obstacles and from the adversaries who create them. On the most primitive level this sort of external conflict is psychologically empty. In the fisticuffs between the protagonists of good and evil, no psychological problems are involved or, at any rate, none are depicted in juvenile representations of conflict.

The detective story, the “adult” analogue of a juvenile adventure tale, has at times been described as a glorification of intellectualized conflict. However, a great deal of the interest in the plots of these stories is sustained by withholding the unraveling of a solution to a problem. The effort of solving the problem is in itself not a conflict if the adversary (the unknown criminal) remains passive, like Nature, whose secrets the scientist supposedly unravels by deduction. If the adversary actively puts obstacles in the detective’s path toward the solution, there is genuine conflict. But the conflict is psychologically interesting only to the extent that it contains irrational components such as a tactical error on the criminal’s part or the detective’s insight into some psychological quirk of the criminal or something of this sort. Conflict conducted in a perfectly rational manner is psychologically no more interesting than a standard Western. For example, Tic-tac-toe, played perfectly by both players, is completely devoid of psychological interest. Chess may be psychologically interesting but only to the extent that it is played not quite rationally. Played completely rationally, chess would not be different from Tic-tac-toe. In short, a pure conflict of interest (what is called a zero-sum game) although it offers a wealth of interesting conceptual problems, is not interesting psychologically, except to the extent that its conduct departs from rational norms.

 

Q. 31 According to the passage, internal conflicts are psychologically more interesting than external conflicts because

A. internal conflicts, rather than external conflicts, form an important component of serious literature as distinguished from less serious genres.

B. only juveniles or very few “adults” actually experience external conflict, while internal conflict is more widely prevalent in society.

C. in situations of internal conflict, individuals experience a dilemma in resolving their own preferences for different outcomes.

D. there are no threats to the reader (or viewer) in case of external conflicts.

 

Q. 32 Which, according to the author, would qualify as interesting psychology?

A. A statistician’s dilemma over choosing the best method to solve an optimization problem.

B. A chess player’s predicament over adopting a defensive strategy against an aggressive opponent.

C. A mountaineer’s choice of the best path to Mt. Everest from the base camp.

D. A finance manager’s quandary over the best way of raising money from the market.

 

Q. 33 According to the passage, which of the following options about the application of game theory to a conflct-of-interest situation is true?

A. Assuming that the rank order of preferences for options is different for different players.

B. Accepting that the interests of different players are often in conflict.

C. Not assuming that the interests are in complete disagreement.

D. All of the above.

 

Q. 34 The problem solving process of a scientist is different from that of a detective because

A. scientists study inanimate objects, while detectives deal with living criminals or law offenders.

B. scientists study known objects, while detectives have to deal with unknown criminals or law offenders.

C. scientists study phenomena that are not actively altered, while detectives deal with phenomena that have been deliberately influenced to mislead.

D. scientists study psychologically interesting phenomena, while detectives deal with “adult” analogues of juvenile adventure tales.

 

Q. 35 The sentences given in the question, when properly sequenced, form a coherent

paragraph. Each sentence is labeled with a letter. Choose the most logical order of sentences from among the given choices to construct a coherent paragraph.

A. Similarly, turning to caste, even though being lower caste is undoubtedly a separate

cause of disparity, its impact is all the greater when the lower-caste families also happen to be poor.

B. Belonging to a privileged class can help a woman to overcome many barriers that obstruct women from less thriving classes.

C. It is the interactive presence of these two kinds of deprivation – being low class and being female – that massively impoverishes women from the less privileged classes.

D. A congruence of class deprivation and gender discrimination can blight the lives of poorer women very severely.

E. Gender is certainly a contributor to societal inequality, but it does not act independently of class.

A. EABDC

B. EBDCA

C. DAEBC

D. BECDA

 

Q. 36 The sentences given in the question, when properly sequenced, form a coherent

paragraph. Each sentence is labeled with a letter. Choose the most logical order of sentences from among the given choices to construct a coherent paragraph.

A. What identity is thus ‘defined by contrast’, divergence with the West becomes central

B. Indian religious literature such as the Bhagavad Gita or the Tantric texts, which are identified as differing from secular writings seen as ‘western’, elicits much greater interest in the West than do other Indian writings, including India’s long history of heterodoxy.

C. There is a similar neglect of Indian writing on non-religious subjects, from mathematics, epistemology and natural science to natural science to economics and linguistic.

D. Through selective emphasis that point up differences with the West, other civilizations can, in this way, be redefined in alien terms, which can be exotic and charming, or else bizarre and terrifying, or simply strange and engaging.

E. The exception is the Kamasutra in which western readers have managed to cultivate an interest

A. BDACE

B. DEABC

C. BDECA

D. BCEDA

 

Q. 37 The sentences given in each question, when properly sequenced, form a coherent paragraph. Each sentence is labeled with a letter. Choose the most logical order of sentences from among the given choices to construct a coherent paragraph.

A. This is now orthodoxy to which I subscribe – up to a point.

B. It emerged from the mathematics of chance and statistics.

C. Therefore the risk is measurable and manageable

D. The fundamental concept: Prices are not predictable, but the mathematical laws of chance can describe their fluctuations.

E. This is how what business schools now call modern finance was born.

A. ADCBE

B. EBDCA

C. ABDCE

D. DCBEA

 

Q. 38 In the question, the word at the top of the table is used in four different ways, numbered 1 to 4. Choose the option in which the usage of the word is incorrect or inappropriate. Near

A. I got there just after you left – a near miss!

B. She and her near friend left early

C. The war led to a near doubling of oil prices.

D. They came near to tears seeing the plight of the victims

 

Q. 39 In the question, the word at the top of the table is used in four different ways, numbered 1 to 4. Choose the option in which the usage of the word is incorrect or inappropriate. Hand

A. I have my hand full, I cannot do it today.

B. The minister visited the jail to see the breach at first hand.

C. The situation is getting out of hand here!

D. When the roof of my house was blown away, he was willing to lend me a hand

 

Q. 40  In the question, the word at the top of the table is used in four different ways, numbered 1 to 4. Choose the option in which the usage of the word is incorrect or inappropriate. For

A. He has a great eye for detail.

B. We are waiting for the day.

C. I can’t bear for her to be angry

D. It couldn’t be done for ever.

 

Questions: 41 – 44

Crinoline and croquet are out. As yet, no political activists have thrown themselves in front of the royal horse on Derby Day. Even so, some historians can spot the parallels. It is a time of rapid technological change. It is a period when the dominance of the world’s superpower is coming under threat. It is an epoch when prosperity masks underlying economic strain. And, crucially, it is a time when policy-makers are confident that all is for the best in the best of all possible worlds. Welcome to the Edwardian Summer of the second age of globalisation.

Spare a moment to take stock of what’s been happening in the past few months. Let’s start with the oil price, which has rocketed to more than $65 a barrel, more than double its level 18 months ago. The accepted wisdom is that we shouldn’t worry our little heads about that, because the incentives are there for business to build new production and refining capacity, which will effortlessly bring demand and supply back into balance and bring crude prices back to $25 a barrel. As Tommy Copper used to say, ‘Just like that’. Then there is the result of the French referendum on the European Constitution, seen as thick-headed luddites railing vainly against the modern world. What the French needed to realize, the argument went, was that there was no alternative to the reforms that would make the country more flexible, more competitive, more dynamic. Just the sort of reforms that allowed Gate Gourmet to sack hundreds of its staff at Heathrow after the sort of ultimatum that used to be handed out by Victorian mill owners. An alternative way of looking at the French “non” is that our neighbours translate “flexibility” as “you’re fired”.

Finally, take a squint at the United States. Just like Britian a century ago, a period of unquestioned superiority is drawing to a close. China is still a long way from matching America’s wealth, but it is growing at a stupendous rate and economic strength brings geopolitical clout. Already, there is evidence of a new scramble for Africa as Washington and Beijing compete for oil stocks. Moreover, beneath the surface of the US economy, all is not well. Growth looks healthy enough, but the competition from China and elsewhere has meant the world’s biggest economy now imports far more than it exports. The US is living beyond its means, but in this time of studied complacency a current account deficit worth 6 perfect of gross domestic product is seen as a sign of strength, not weakness. In this new Edwardian summer, comfort is taken from the fact that dearer oil has not had the savage inflationary consequences of 1973-1974, when a fourfold increase in the cost of crude brought an abrupt end to a postwar boom that had gone on uninterrupted for a quarter of a century. True, the cost of living has been affected by higher transport costs, but we are talking of inflation at 2.3 per cent and not 27 per cent. Yet the idea that higher oil prices are of little consequence is fanciful. If people are paying more to fill up their cars it leaves them with less to spend on everything else, but there is a reluctance to consume less. In the 1970s unions were strong and able to negotiate large, compensatory pay deals that served to intensify inflationary pressure. In 2005, that avenue is pretty much closed off, but the abolition of all the controls on credit that existed in the 1970s means that households are invited to borrow more rather than consume less. The knock-on effects of higher oil prices are thus felt in different ways – through high levels of indebtedness, in inflated asset prices, and in balance of payments deficits.

There are those who point out, rightly, that modern industrial capitalism has proved mightily resilient these past 250 years, and that a sign of the enduring strength of the system has been the way it apparently shrugged off everything – a stock market crash, 9/11, rising oil prices – that have been thrown at it in the half decade since the millennium. Even so, there are at least three reasons for concern. First, we have been here before. In terms of political economy, the first era of globalisation mirrored our own. There was a belief in unfettered capital flows, in free migration. Eventually, though, there was a backlash, manifested in a struggle between free traders and protectionists, and in rising labour militancy. Second, the world is traditionally as its most fragile at times when the global balance of power is in flux. By the end of the nineteenth century, Britain’s role as the hegemonic power was being challenged by the rise of the United States, Germany, and Japan while the Ottoman and Hapsburg empires were clearly in rapid decline. Looking ahead from 2005, it is clear that over the next two or three decades, both China and India – which together account for half the world’s population – will flex their muscles.

Finally, there is the question of what rising oil prices tell us. The emergence of China and India means global demand for crude is likely to remain high a t a time when experts say production is about to top out. If supply constraints start to bite, any decline in the prices are likely to be short-term cyclical affairs punctuating a long upward trend.

 

Q. 41 By the expression ‘Edwardian Summer’, the author refers

A. unparalleled luxury and opulence.

B. a sense of complacency among people because of all-round prosperity.

C. a culmination of all-round economic prosperity.

D. an imminent danger lurking behind economic prosperity.

 

Q. 42 What, according to the author, has resulted in a widespread belief in the resilence of modern capitalism?

A. Growth in the economies of Western countries despite shocks in the form of increase in levels of indebtedness and inflated asset prices.

B. Increase in the prosperity of Western countries and China despite rising oil prices.

C. Continued growth of Western economies despite a rise in terrorism, an increase in oil prices and other similar shocks.

D. The success of continued reforms aimed at making Western economies more dynamic, competitive and efficient.

 

Q. 43 Which of the following best represents the key argument made by the author?

A. The rise in oil prices, the flux in the global balance of power and historical precedents should make us question our belief that the global economic prosperity would continue.

B. The belief that modern industrial capitalism is highly resilient and capable of overcoming shocks will be belied soon.

C. Widespread prosperity leads to neglect of early signs of underlying economic weakness, manifested in higher oil prices and a flux in the global balance of power.

D. A crisis is imminent in the West given the growth of countries like China and India and the increases in oil prices.

 

Q. 44 What can be inferred about the author’s view when he states ‘As Tommy Cooper used to say “just like that” ?

A. Industry has incentives to build new production and refining capacity and therefore oil prices would reduce.

B. There would be a correction in the price levels of oil once new production capacity is added.

C. The decline in oil prices is likely to be short-term in nature.

D. It is not necessary that oil prices would go down to earlier levels.

 

Questions: 45 – 48

While complex in the extreme, Derrida’s work has proven to be a particularly influential approach to the analysis of the ways in which language structures our understanding of ourselves and the world we inhabit an approach he termed deconstruction. In its simplest formulation, deconstruction can be taken to refer to a methodological strategy which seeks to uncover layers of hidden meaning in a text that have been denied or suppressed. The term ‘text’, in this respect, does not refer simply to a written form of communication, however. Rather, texts are something we all produce and reproduce constantly in our every day social relations, be they spoken, written or embedded in the construction of material artifacts. At the heart of Derrida’s deconstructive approach is his critique of what he perceives to be the totalitarian impulse of the Enlightenment pursuit to bring all that exists in the world under the domain of representative language, a pursuit he refers to as logocentrism. Logocentrism is the search for a rational language that is able to know and represent the world and all its aspects perfectly and accurately. Its totalitarian dimension, for Derrida at least, lies primarily in its tendency to marginalize or dismiss all that does not neatly comply with its particular linguistic representations, a tendency that, throughout history, has all too frequently been manifested in the form of authoritarian institutions. Thus logocentrism has, in its search for the truth of absolute representation, subsumed difference and oppressed that which it designates as its alien ‘other’. For Derrida, western civilization has been built upon such a systematic assault on alien cultures and ways of life, typically in the name of reason and progress. In response to logocentrism, deconstruction posits the idea that the mechanism by which this process of marginalization and the ordering of truth occurs is through establishing systems of binary opposition. Oppositional linguistic dualisms, such as rational/irrational, culture/nature and good/bad are not, however, construed as equal partners as they are in, say, the semiological structuralism of Saussure. Rather, they exist, for Derrida, in a series of hierarchical relationships with the first term normally occupying a superior position. Derrida defines the relationship between such oppositional terms using the neologism difference. This refers to the realization that in any statement, oppositional terms differ from each other (for instance, the difference between rationality and irrationality is constructed through oppositional usage), and at the same time, a hierarchical relationship is maintained by the deference of one term to the other (in the positing of rationality over irrationality, for instance). It is this latter point which is perhaps the key to understanding Derrida’s approach to deconstruction. For the fact at any given time one term must defer to its oppositional ‘other’, means that the two terms are constantly in a state of interdependence. The presence of one is dependent upon the absence or ‘absent-presence’ of the ‘other’, such as in the case of good and evil, whereby to understand the nature of one, we must constantly relate it to the absent term in order to grasp its meaning. That is, to do good, we must understand that our act is not evil, for without that comparison the term becomes meaningless. Put simply, deconstruction represents an attempt to demonstrate the absent-presence of this oppositional ‘other’, to show that what we say or write is in itself not expressive simply of what is present, but also of what is absent. Thus, deconstruction seeks to reveal the interdependence of apparently dichotomous terms and their meanings relative to their textual context; that is, within the linguistic power relations which structure dichotomous terms hierarchically. In Derrida’s own words, a deconstructive reading “must always aim at a certain relationship, unperceived by the writer, between what he commands and what he does not command of the patterns of a language that he uses…. [It] attempts to make the not-seen accessible to sight.’ Meaning, then, is never fixed or stable, whatever the intention of the author of a text. For Derrida, language is a system of relations that are dynamic, in that all meanings we ascribe to the world are dependent not only on what we believe to be present but also on what is absent. Thus, any act of interpretation must refer not only to what the author of a text intends, but also to what is absent from his or her intention. This insight leads, once again, to Derrida’s further rejection of the idea of the definitive authority of the intentional agent or subject. The subject is decentred; it is conceived as the outcome of relations of differance. As author of its own biography, the subject thus becomes the ideological fiction of modernity and its logocentric philosophy, one that depends upon the formation of hierarchical dualisms, which repress and deny the presence of the absent ‘other’. No meaning can, therefore, even be definitive, but is merely an outcome of a particular interpretation.

 

Q. 45 According to the passage, Derrida believes that:

A. Reality can be construed only through the use of rational analysis

B. Language limits our construction of reality.

C. A universal language will facilitate a common understanding of reality.

D. We need to uncover the hidden meaning in a system of relations expressed by language

 

Q. 46 To Derrida, ‘logocentrism’ does not imply:

A. A totalitarian impulse

B. A domain of representative language.

C. Interdependence of the meanings of dichotomous terms.

D. A strategy that seeks to suppress hidden meanings in a text.

 

Q. 47 According to the passage, Derrida believes that the system of binary opposition

A. represents a prioritization or hierarchy.

B. reconciles contradictions and dualities.

C. weakens the process of marginalization and ordering of truth.

D. deconstructs reality.

 

Q. 48 Derrida rejects the idea of ‘definitive authority of the subject’ because

A. interpretation of the text may not make the unseen visible.

B. the meaning of the text is based on binary opposites.

C. the implicit power relationship is often ignored.

D. any act of interpretation must refer to what the author intends.

 

Questions: 49 – 52

Each of the following questions has a paragraph from which the last sentence has been deleted. From the given options, choose that one that completes the paragraph in the most appropriate way.

 

Q. 49 The audiences for crosswords and sudoku, understandably, overlap greatly, but there are differences, too. A crossword attracts a more literary person, while sudoku appeals to a keenly logical mind. Some crossword enthusiasts turn up their noses at sudoku because they feel it lacks depth. A good crossword requires vocabulary, knowledge, mental flexibility and sometimes even a sense of humor to complete. It touches numerous areas of life and provides an “Aha!” or two along the way.__________________

A. Sudoku, on the other hand, is just a logical exercise, each one similar to the last.

B. Sudoku, incidentally, is growing faster in popularity than crosswords, even among the literati.

C. Sudoku, on the other hand, can be attempted and enjoyed even by children.

D. Sudoku, however, is not exciting in any sense of the term.

 

Q. 50 Most firms consider expert individuals to be too elitist, temperamental, egocentric, and difficult to work with. Force such people to collaborate on a high-stakes project and they just might come to fisticuffs. Even the very notion of managing such a group seems unimaginable. So most organizations fall into default mode, setting up project teams of people who get along nicely. _______________

A. The result, however, is disastrous

B. The result is mediocrity.

C. The result is creation of experts who then become elitist.

D. Naturally, they drive innovations.

 

Q. 51 Federer’s fifth grand slam win prompted a reporter to ask whether he was the best ever. Federer is certainly not lacking in confidence, but he wasn’t about to proclaim himself the best ever. “The best player of this generation, yes”, he said. “But nowhere close to ever. Just look at the records that some guys have. I’m a minnow.”

A. His win against Agassi, a genius from the previous generation, contradicts that.

B. Sampras, the king of an earlier generation, was as humble.

C. He is more than a minnow to his contemporaries.

D. The difference between ‘the best of this generation’ and ‘the best ever’ is a matter of perception.

 

Q. 52 Thus the end of knowledge and the closing of the frontier that it symbolizes is not a looming crisisat all, but merely one of many embarrassing fits of hubris in civilization’s long industry. In the end, it will pass away and be forgotten. Ours is not the first generation to struggle to understand the organizational laws of the frontier, deceive itself that it has succeeded, and go to its grave having failed.

A. One would be wise to be humble.

B. But we might be the first generation to actually reach the frontier.

C. But we might be the first generation to deal with the crisis.

D. However, this time the success is not illusory.

 

Questions: 53 – 56

Each question consists of four sentences on a topic. Some sentences are grammatically incorrect or inappropriate. Select the option that indicates the grammatically correct and appropriate sentence(s).

 

Q. 53 A. When virtuoso teams begin their work, individuals are in and group consensus is out.

B. As project progresses, however, the individual stars harness themselves to the product of the group.

C. Sooner or later, the members break through their own egocentrism and become a plurality with single-minded focus on the goal.

D. In short, they morph into a powerful team with a shared identity.

A. A & C

B. A & D

C. B & D

D. A, C & D

 

Q. 54 A. Large reductions in the ozone layer, which sits about 15-30 km above the Earth, take place each winter over the polar regions, especially the Antarctic, as low temperatures allow the formationof stratospheric clouds that assist chemical reactions breaking down ozone.

B. Industrial chemicals containing chlorine and bromine have been blamed for thinning the layer because they attack the ozone molecules, making them to break apart.

C. Many an offending chemicals have now been banned.

D. It will still take several decades before these substances have disappeared from the atmosphere.

A. D

B. B & D

C. A & D

D. A & C

 

Q. 55 A. The balance of power will shift to the East as China and India evolve.

B. Rarely the economic ascent of two still relatively poor nations has been watched with such a mixture of awe, opportunism, and trepidation.

C. Postwar era witnessed economic miracles in Japan and South Korea, but neither was populous enough to power worldwide growth or change the game in a complete spectrum of industries.

D. China and India, by contrast, posses the weight and dynamism to transform the 21st century global economy.

A. A, B & C

B. A & D

C. C

D. C & D

 

Q. 56 A. People have good reason to care about the welfare of animals.

B. Ever since Enlightenment, their treatment has been seen as a measure of mankind’s humanity.

C. It is no coincidence that William Wilberforce and Sir Thomas Foxwell Buxton, two leaders of the movement to abolish the slave trade, helped found the Royal Society for the Prevention of Cruelty to Animals in 1820s.

D. An increasing number of people go further: mankind has a duty not to cause pain to animals that have the capacity to suffer.

A. A & D

B. B

C. A & C

D. C & D

 

Questions: 57 – 60

Each of the following questions has a paragraph with one quoted word that does not make sense. Choose the most appropriate replacement for that word from the options given below the paragraph.

 

Q. 57 Intelligent design derives from an early 19th – century explanation of the natural world given by an English clergyman, William Paley. Paley was the populariser of the famous watchmaker analogy. Proponents of intelligent design are “crupping” Paley’s argument with a new gloss from molecular biology.

A. destroying

B. testing

C. resurrecting

D. questioning

 

Q. 58 Women squat, heads covered, beside huge piles of limp fodder and “blunk” oil lamps, and just about all the cows in the three towns converge upon this spot. Sinners, supplicants and yes, even scallywags hand over a few coins for a crack at redemption and a handful of grass.

A. Shining

B. Bright

C. Sputtering

D. Effulgent

 

Q. 59 It is “klang” to a sensitive traveler who walks through this great town, when he sees the streets, the roads and cabin doors crowded with beggars, mostly women, followed by three, four, or six children, all in rags and importuning every passenger for alms.

A. Amusing

B. Irritating

C. Disgusting

D. Distressing

 

Q. 60 Or there is the most “fingummy” diplomatic note on record: when Philip of Macedon wrote to the Spartans that, if he came within their borders, he would leave not one stone of their city, they wrote back the one word – If

A. Witty

B. Rude

C. Simple

D. Terse

 

Questions: 61 – 64

A management institute was established on January 1, 2000 with 3, 4, 5, and 6 faculty members in the Marketing, Organisational Behaviour (OB), Finance, and Operations Management (OM) areas respectively, to start with. No faculty member retired or joined the institute in the first three months of the year 2000. In the next four years, the institute recruited one faculty member in each of the four areas. All these new faculty members, who joined the institute subsequently over the years, were 25 years old at the time of their joining the institute. All of them joined the institute on April 1. During these four years, one of the faculty members retired at the age of 60. The diagram shown in figure (1) gives the areawise average age (in terms of number of completed years) of faculty members as on April 1 of 2000, 2001, 2002, and 2003.

 

Q. 61 From which area did the faculty member retire?

A. Finance

B. Marketing

C. OB

D. OM

 

Q. 62 Professors Naresh and Devesh, two faculty members in the Marketing area, who have been with the Institute since its inception, share a birthday, which falls on 20th November. One was born in 1947 and the other one in 1950. On April 1 2005, what was the age of the third faculty member, who has been in the same area since inception?

A. 47

B. 50

C. 5

D. 52

 

Q. 63 In which year did the new faculty member join the Finance area?

A. 2000

B. 2001

C. 2002

D. 2003

 

Q. 64 What was the age of the new faculty member, who joined the OM area, as on April 1, 2003?

A. 25

B. 26

C. 27

D. 28

 

Questions: 65 – 67

The table given in figure (1) reports annual statistics related to rice production in select states of India for a particular year.

 

Q. 65 Which two states account for the highest productivity of rice (tons produced per hectare of rice cultivation)?

A. Haryana and Punjab

B. Punjab and Andhra Pradesh

C. Andhra Pradesh and Haryana

D. Uttar Pradesh and Haryana

 

Q. 66 How many states have a per capita production of rice (defined as total rice production divided by its population) greater than Gujarat?

A. 3

B. 4

C. 5

D. 6

 

Q. 67 An intensive rice producing state is defined as one whose annual rice production per million of population is at least 400,000 tons. How many states are intensive rice producing states?

A. 5

B. 6

C. 7

D. 8

 

Questions: 68 – 70

The table given in figure (1) reports the gender, designation and age-group of the employees in an organization. It also provides information on their commitment to projects coming up in the months of January (Jan), February (Feb), March (Mar) and April (Apr), as well as their interest in attending workshops on: Business Opportunities (BO), Communication Skills (CS), and E-Governance (EG). For each workshop, exactly four employees are to be sent, of which at least two should be Females and at least one should be Young. No employee can be sent to a workshop in which he she is not interested in. An employee cannot attend the workshop on.

1. Communication Skills, if he/she is committed to internal projects in the month of January.

2. Business Opportunities, if he/she is committed to internal projects in the month of February.

3. E-governance, if he/she is committed to internal projects in the month of March.

 

Q. 68 Assuming that Parul and Hari are attending the workshop on Communication Skills (CS), then which of the following employees can possibly attend the CS workshop?

A. Rahul and Yamini

B. Dinesh and Lavanya

C. Anshul and Yamini

D. Fatima and Zeena

 

Q. 69 How many Executives (Exe) cannot attend more than one workshop?

A. 2

B. 3

C. 15

D. 16

 

Q. 70 Which set of employees cannot attend any of the workshops?

A. Anshul, Charu, Eashwaran and Lavanya

B. Anshul, Bushkant, Gayatri, and Urvashi

C. Charu, Urvashi, Bushkant and Mandeep

D. Anshul, Gayatri, Eashwaran and Mandeep

 

Questions: 71 – 74

In the table given in figure (1) is the listing of players, seeded from highest (#1) to lowest (#32), who are due to play in an Association of Tennis Players (ATP) tournament for women. This tournament has four knockout rounds before the final, i.e., first round, second round, quarterfinals, and semi-finals. In the first round, the highest seeded player plays the lowest seeded player (seed #32) which is designated match No. 1 of first round; the 2nd seeded player plays the 31st seeded player which is designated match No. 2 of the first round, and so on. Thus, for instance, match No. 16 of first round is to be played between 16th seeded player and the 17th seeded player. In the second round, the winner of match No. 1 of first round plays the winner of match No. 16 of first round and is designated match No. 1 of second round. Similarly, the winner of match No. 2 of first round plays the winner of match No. 15 of first round, and is designated match No. 2 of second round. Thus, for instance, match No. 8 of the second round is to be played between the winner of match No. 8 of first round and the winner of match No. 9 of first round. The same pattern is followed for later rounds as well.

 

Q. 71 If there are no upsets (a lower seeded player beating a higher seeded player) in the first round, and only match Nos. 6, 7, and 8 of the second round result in upsets, then who would meet Lindsay Davenport in quarter finals, in case Davenport reaches quarter finals?

A. Justine Henin

B. Nadia Petrova

C. Patty Schnyder

D. Venus Williams

 

Q. 72 If Elena Dementieva and Serena Williams lose in the second round, while Justine Henin and Nadia Petrova make it to the semi-finals, then who would play Maria Sharapova in the quarterfinals, in the event Sharapova reaches quarterfinals?

A. Dinara Safina

B. Justine Henin

C. Nadia Petrova

D. Patty Schnyder

 

Q. 73 If, in the first round, all even numbered matches (and none of the odd numbered ones) result in upsets, and there are no upsets in the second round, then who could be the lowest seeded player facing Maria Sharapova in semi-finals?

A. Anastasia Myskina

B. Flavia Pennetta

C. Nadia Petrova

D. Svetlana Kuznetsova

 

Q. 74 If the top eight seeds make it to the quarterfinals, then who, amongst the players listed below, would definitely not play against Maria Sharapova in the final, in case Sharapova reaches the final?

A. Amelie Mauresmo

B. Elena Dementieva

C. Kim Clijsters

D. Lindsay Davenport

 

Questions: 75 – 78

Venkat, a stockbroker, invested a part of his money in the stock of four companies — A, B, C and D. Each of these companies belonged to different industries, viz., Cement, Information Technology (IT), Auto, and Steel, in no particular order. At the time of investment, the price of each stock was Rs. 100. Venkat purchased only one stock of each of these companies. He was expecting returns of 20%, 10%, 30% and 40% from the stock of companies A, B, C and D, respectively. Returns are defined as the change in the value of the stock after one year, expressed as a percentage of the initial value. During the year, two of these companies announced extraordinarily good results. One of these two companies belonged to the Cement or the IT industry, while the other one belonged to either the Steel or the Auto industry. As a result, the returns on the stocks of these two companies were higher than the initially expected returns. For the company belonging to the Cement or the IT industry with extraordinarily good results, the returns were twice that of the initially expected returns. For the company belonging to the Steel or the Auto industry, the returns on announcement of extraordinarily good results were only one and a half times that of the initially expected returns. For the remaining two companies which did not announce extraordinarily good results, the returns realized during the year were the same as initially expected.

 

Q. 75 What is the minimum average return Venkat would have earned during the year?

A. 30%

B. 31(1/4)%

C. 32(1/2)%

D. Cannot be determined

 

Q. 76 If Venkat earned a 35% return on average during the year, then which of these statements would necessarily be true?

I. Company A belonged either to Auto or to Steel Industry.

II. Company B did not announce extraordinarily good results.

III. Company A announced extraordinarily good results.

IV. Company D did not announce extraordinarily good results.

A. I and II only

B. II and III only

C. III and IV only

D. II and IV only

 

Q. 77 If Venkat earned a 38.75% return on average during the year, then which of these statement(s) would necessarily be true?

I. Company C belonged either to Auto or to Steel Industry.

II. Company D belonged either to Auto or to Steel Industry.

III. Company A announced extraordinarily good results.

IV. Company B did not announce extraordinarily good results.

A. I and II only

B. II and III only

C. I and IV only

D. II and IV only

 

Q. 78 If Company C belonged to the Cement or the IT industry and did announce extraordinarily good results, then which of these statement(s) would necessarily be true?

I. Venkat earned not more than 36.25% return on average.

II. Venkat earned not less than 33.75% return on average.

III. If Venkat earned 33.75% return on average, Company A announced extraordinarily good results.

IV. If Venkat earned 33.75% return on average, Company B belonged either to Auto or to Steel industry.

A. I and II only

B. II and IV only

C. II and III only

D. III and IV only

 

Questions: 79 – 82

The year is 2089. Beijing, London, New York, and Paris are in contention to host the 2096 Olympics. The eventual winner is determined through several rounds of voting by members of the IOC with each member representing a different city. All the four cities in contention are also represented in IOC. (a) In any round of voting, the city receiving the lowest number of votes in that round gets eliminated. The survivor after the last round of voting gets to host the event.

(b) A member is allowed to cast votes for at most two different cities in all rounds of voting combined. (Hence, a member becomes ineligible to cast a vote in a given round if both the cities (s)he voted for in earlier rounds are out of contention in that round of voting.) (c) A member is also ineligible to cast a vote in a round if the city (s)he represents is in contention in that round of voting.

(d) As long as the member is eligible, (s)he must vote and vote for only one candidate city in any round. The incomplete table given in figure (1) shows the information on cities that received the maximum and minimum votes in different rounds, the number of votes cast in their favour, and the total votes that were cast in those rounds.

It is also known that:

1. All those who voted for London and Paris in round, 1 continued to vote for the same cities in subsequent rounds as long as these cities were in contention. 75% of those who voted for Beijing in round 1, voted for Beijing in round 2 as well.

2. Those who voted for New York in round 1, voted either for Beijing or Paris in round 2.

3. The difference in votes cast for the two contending cities in the last round was 1.

4. 50% of those who voted for Beijing in round 1, voted for Paris in round 3.

 

Q. 79 What percentage of members from among those who voted for New York in round 1, voted for Beijing in round 2?

A. 33.33

B. 50

C. 66.67

D. 75

 

Q. 80 What is the number of votes cast for Paris in round 1?

A. 16

B. 18

C. 22

D. 24

 

Q. 81 What percentage of members from among those who voted for Beijing in round 2 and were eligible to vote in round 3, voted for London?

A. 33.33

B. 38.10

C. 50

D. 66.67

 

Q. 82 Which of the following statements must be true?

I. IOC member from New York must have voted for Paris in round 2.

II. IOC member from Beijing voted for London in round 3.

A. Only I

B. Only II

C. Both I and II

D. Neither I nor II

 

Questions: 83 – 86

The table given in figure (1) presents the revenue (in million rupees) of four firms in three states. These firms, Honest Ltd., Aggressive Ltd., Truthful Ltd. and Profitable Ltd. are disguised in the table as A, B, C and D, in no particular order. Further, it is known that:

1. In the state of MP, Truthful Ltd. has the highest market share.

2. Aggressive Ltd.’s aggregate revenue differs from Honest Ltd.’s by Rs. 5 million.

 

Q. 83 What can be said regarding the following two statements?

Statement 1: Profitable Ltd. has the lowest share in MP market.

Statement 2: Honest Ltd.’s total revenue is more than Profitable Ltd.

A. If Statement 1 is true then Statement 2 is necessarily true.

B. If Statement 1 is true then Statement 2 is necessarily false.

C. Both Statement 1 and Statement 2 are true.

D. Neither Statement 1 nor Statement 2 is true.

 

Q. 84 What can be said regarding the following two statements?

Statement 1: Aggressive Ltd.’s lowest revenues are from MP.

Statement 2: Honest Ltd.’s lowest revenues are from Bihar.

A. If Statement 2 is true then Statement 1 is necessarily false.

B. If Statement 1 is false then Statement 2 is necessarily true.

C. If Statement 1 is true then Statement 2 is necessarily true.

D. None of the above.

 

Q. 85 What can be said regarding the following two statements?

Statement 1: Honest Ltd. has the highest share in the UP market.

Statement 2: Aggressive Ltd. has the highest share in the Bihar market.

A. Both statements could be true.

B. At least one of the statements must be true.

C. At most one of the statements is true.

D. None of the above

 

Q. 86 If Profitable Ltd.’s lowest revenue is from UP, then which of the following is true?

A. Truthful Ltd.’s lowest revenues are from MP

B. Truthful Ltd.’s lowest revenues are from Bihar.

C. Truthful Ltd.’s lowest revenues are from UP.

D. No definite conclusion is possible.

 

Questions: 87 – 90

Help Distress (HD) is an NGO involved in providing assistance to people suffering from natural disasters. Currently, it has 37 volunteers. They are involved in three projects: Tsunami Relief (TR) in Tamil Nadu, Flood Relief (FR in Maharashtra, and Earthquake Relief (ER) in Gujarat. Each volunteer working with Help Distress has to be involved in at least one relief work project.

1. A maximum number of volunteers are involved in the FR project. Among them, the number of volunteers involved in FR project alone is equal to the volunteers having additional involvement in the ER project.

2. The number of volunteers involved in the ER project alone is double the number of volunteers involved in all the three projects.

3. 17 volunteers are involved in the TR project.

4. The number of volunteers involved in the TR project alone is one less than the number of volunteers involved in ER project alone.

5. Ten volunteers involved in the TR project are also involved in at least one more project.

 

Q. 87 Based on the information given above, the minimum number of volunteers involved in both FR and TR projects, but not in the ER project is

A. 1

B. 3

C. 4

D. 5

 

Q. 88 Which of the following additional information would enable to find the exact number of volunteers involved in various projects?

A. Twenty volunteers are involved in FR.

B. Four volunteers are involved in all the three projects.

C. Twenty three volunteers are involved in exactly one project.

D. No need for any additional information.

 

Q. 89 After some time, the volunteers who were involved in all the three projects were asked to withdraw from one project. As a result, one of the volunteers opted out of the TR project, and one opted out of the ER project, while the remaining ones involved in all the three projects opted out of the FR project. Which of the following statements, then, necessarily follows?

A. The lowest number of volunteers is now in TR project

B. More volunteers are now in FR project as compared to ER project.

C. More volunteers are now in TR project as compared to ER project.

D. None of the above

 

Q. 90 After the withdrawal of volunteers, as indicated in Question 89, some new volunteers joined the NGO. Each one of them was allotted only one project in a manner such that, the number of volunteers working in one project alone for each of the three projects became identical. At that point, it was also found that the number of volunteers involved in FR and ER projects was the same as the number of volunteers involved in TR and ER projects. Which of the projects now has the highest number of volunteers?

A. ER

B. FR

C. TR

D. Cannot be determined

 

 

Answer Sheet
Question 1 2 3 4 5 6 7 8 9 10
Answer A C B D A B B D D C
Question 11 12 13 14 15 16 17 18 19 20
Answer D A B C B A B D D C
Question 21 22 23 24 25 26 27 28 29 30
Answer C B A A A C D D C B
Question 31 32 33 34 35 36 37 38 39 40
Answer C B D C B D B B A D
Question 41 42 43 44 45 46 47 48 49 50
Answer B C A D D C A A A B
Question 51 52 53 54 55 56 57 58 59 60
Answer C A B C B A C C D D
Question 61 62 63 64 65 66 67 68 69 70
Answer A D C C A B D A B B
Question 71 72 73 74 75 76 77 78 79 80
Answer D C A C A B C B D D
Question 81 82 83 84 85 86 87 88 89 90
Answer D A B C C C C A B A

CAT Previous Year Paper 2004

CAT 2004

Quant 

Instructions 

Directions for the following three questions: Answer the questions on the basis of the information given below

In the adjoining figure I and II are circles with centres P and Q respectively, The two circles touch each other and have common tangent that touches them at points R and S respectively. This common tangent meets the line joining P and Q at O. The diameters of I and II are in the ratio 4: 3. It is also known that the length of PO is 28 cm. 

Q. 1 What is the ratio of the length of PQ to that of QO? 

A. 1 : 4 

B. 1 : 3 

C. 3 : 8 

D. 3 : 4 

Answer: B. 

Explanation: 

Triangles PRO and QSO are similar. 

PR/QS = 4/3. 

Therefore, PO/QO = 4/3 => 

Let the radius of 2 circles be 4R and 3R respectively. 

28/(28-7R) = 4/3 

4-R = 3 

R = 1 

Required ratio of PQ/QO = (4R+3R)/(28-7R) = 7R/(28-7R) = 7/21 = 1:3 

Q. 2 What is the radius of the circle II? 

A. 2 cm 

B. 3 cm 

C. 4 cm 

D. 5 cm 

Answer: B. 

Explanation: 

Let the radius of circles I and II be 4R and 3R respectively. Triangles PRO and QSO are similar. 

PR/QS = 4/3. 

Therefore, PO/QO = PR/QS 

=> PO/QO = 4/3 

=> 28/(28-7R) = 4/3 

=> 4-R = 3 

=> R = 1 

Radius of smaller circle = 3R = 3 

Q. 3 The length of SO is 

A. 8√3cm 

B. 10√3cm 

C. 12√3cm 

D. 14√3 cm 

Answer: C. 

Explanation: 

Triangles PRO and QSO are similar. 

PR/QS = 4/3. 

Therefore, PO/QO = 4/3 => 

Let the radius of 2 circles be 4R and 3R respectively. 28/(28-7R) = 4/3 

4-R = 3 

R = 1 

Therefore, SO =√(21 2 − 3 2 ) =√432 =12√3 cm 

 

Instructions 

Directions for the following two questions: 

Answer the questions on the basis of the information given below.

 f1(x) = x if 0 ≤ x ≤ 1 f1(x) = 1  if x >= 0 otherwise 

f2(x) = f1(−x) for all x

f3(x) = −f2(x) for all x

f4(x) = f3(−x) for all x

 

Q. 4 How many of the following products are necessarily zero for every x: 

f1(x)f2(x),  f2(x)f3(x),  f2(x)f4(x

A.

B.

C.

D.

Answer: C. 

Explanation: 

Checking for different values of x . Suppose x= -0.5 we get

 f1(x)f2(x) = 0 ∗ 0.5 = 0 

f2(x)f4(x) = 0.5 ∗ 0 = 0 . 

But f2(x)f3(x) is not equal to zero. 

Hence two functions are necessarily equal to zero and two products given above are equal to zero. 

Q. 5 Which of the following is necessarily true? 

A. f4(x) = f1(x) for all x 

B. f1(x) = −f3(−x) for all x 

C. f2(−x) = f4(x) for all x 

D. f1(x) + f3(x) = 0  for all x 

Answer: B. 

Explanation: 

Relation between f3 and f1 would be 

f3(x) = −f1(−x). 

Put x= –x we get 

f3(−x) = −f1(x) so multiply by -1 we get 

f3(−x) = f1(x). 

Instructions 

Directions for the following two questions: Answer the questions on the basis of the information given below. 

In an examination, there are 100 questions divided into three groups A, B. and C. such that each group contains at least one question. Each question in group A. carries 1 mark, each question in group B. carries 2 marks and each question in group C. carries 3 marks. It is known that the questions in group A. together carry at least 60% of the total marks. 

Q. 6 If group B contains 23 questions, then how many questions are there in group C? 

A.

B.

C.

D. Cannot be determined 

Answer: A. 

Explanation: 

Group B contains 23 questions => Marks of group B = 46 

Let the number of questions in A be x and in C be 77-x

Marks of group A = x 

So, x/(x+46+3*77-3x) >= 60% 

=> 5x >= 3(277-2x

=> 11x >= 831 

=> x >= 75.54 

=> x = 76 (min) 

So, the possible number of questions in group C = 1. 

Q. 7 If group C contains 8 questions and group B carries at least 20% of the total marks, which of the following best describes the number of questions in group B? 

A. 11 or 12 

B. 12 or 13 

C. 13 or 14 

D. 14 or 15 

Answer: C. 

Explanation: 

Let the number of questions in group B be x

So, number of questions in group A = 92-x 

Marks of group B = 2x 

2x/(92-x+2x+24) >= 20% 

=> 10x >= 116+x 

=> 9x >= 116 

=> x >= 12.88 

From the options, x can be 13 or 14 

 

Instructions 

For the following questions answer them individually 

Q. 8 Two boats, traveling at 5 and 10 kms per hour, head directly towards each other. They begin at a distance of 20 kms from each other. How far apart are they (in kms) one minute before they collide? 

A. 1/12 

B. 1/6 

C. 1/4 

D. 1/3 

Answer: C. 

Explanation: 

The relative speed is 15 km/hr = 15 km/60 min = 0.25 km/min = 250 m/min. 

Therefore, one minute before they collide, they are at a distance of 250m. 

Q. 9 A rectangular sheet of paper, when halved by folding it at the midpoint of its longer side, results in a rectangle, whose longer and shorter sides are in the same proportion as the longer and shorter sides of the original rectangle. If the shorter side of the original rectangle is 2, what is the area of the smaller rectangle? 

A.  4√2

B. 2√2

C. √2

D. None of the above 

Answer: B. 

Explanation: 

Let the longer side of the original rectangle be x. Therefore, breadth of the modified rectangle = x/2. 

=> x:2 = 2:(x/2) 

=>x2= 8 

Area of smaller rectangle = (x/2)*2 = x = 2√2

Q. 10 If the sum of the first 11 terms of an arithmetic progression equals that of the first 19 terms, then what is the sum of the first 30 terms? 

A.

B. -1 

C.

D. Not unique 

Answer: A. 

Explanation: 

Sum of the first 11 terms = 11/2 ( 2a+10d) 

Sum of the first 19 terms = 19/2 (2a+18d) 

=> 22a+110d = 38a+342d => 16a = -232d 

=> 2a = -232/8 d = -29d 

Sum of the first 30 terms = 15(2a+29d) = 0 

Q. 11 If a man cycles at 10 km/hr, then he arrives at a certain place at 1 p.m. If he cycles at 15 km/hr, he will arrive at the same place at 11 a.m. At what sped must he cycle to get there at noon? 

A. 11 km/hr 

B. 12 km/hr 

C. 13 km/hr 

D. 14 km/hr 

Answer: B. 

Explanation: 

Let the distance to be travelled be D 

In the first case, D/10 = t 

In the second case, D/15 = t-2 

=> D/15 = D/10 – 2 

=> 2D = 3D – 60 

=> D = 60 km and T = 6 hours 

Therefore, to get there at noon, he has to travel at 60/5 = 12 km/hr 

Q. 12 On January 1, 2004 two new societies S1 and S2 are formed, each n numbers. On the first day of each subsequent month, S1 adds b members while S2 multiples its current numbers by a constant factor r. Both the societies have the same number of members on July 2, 2004. If b = 10.5n, what is the value of r? 

A. 2.0 

B. 1.9 

C. 1.8 

D. 1.7 

Answer: A. 

Explanation: 

According to given condition we have , 

n+6b =nr6  and b=10.5n , 

63n+n = nr6 

r6 = 64 

r = 2 

Q. 13 If f(x) = x3 − 4x + p, and f(0) and f(1) are of opposite signs, then which of the following is necessarily true 

A. -1 < p < 2 

B. 0 < p < 3 

C. -2 < p < 1 

D. -3 < p < 0 

Answer: B. 

Explanation: 

f(1) = 1-4+p = p-3 

f(0) = p 

Since they are of opposite signs, p(p-3) < 0 

=> 0 < p < 3 

Q. 14 Suppose n is an integer such that the sum of digits on n is 2, and 1010 < n < 1011. The number of different values of n is 

A. 11 

B. 10 

C.

D.

Answer: A. 

Explanation: 

The sum of digits should be 2. The possibilities are 1000000001, 1000000010, 10000000100,..these 10 cases . Also additional 1 case where 20000000000. 

Hence option A . 

Q. 15 A milkman mixes 20 litres of water with 80 litres of milk. After selling one-fourth of this mixture, he adds water to replenish the quantity that he had sold. What is the current proportion of water to milk? 

A. 2 : 3 

B. 1 : 2 

C. 1 : 3 

D. 3 : 4 

Answer: A. 

Explanation: 

After selling 1/4th of the mixture, the remaining quantity of water is 15 liters and milk is 60 liters. So the milkman would add 25 liters of water to the mixture. The total amount of water now is 40 liters and milk is 60 liters. Therefore, the required ratio is 2:3. 

Q.16 Let  y = 1/(2+ 1/(3 +1/(2 + 1/(3+…)))). Then y equal?

A. {(11)+3}/2

B. {(11)-3}/2

C. {(15)+3}/2

D. {(15)-3}/2

Answer: D. 

Explanation: 

y = 1/(2+ 1/(3 +1/(2 + 1/(3+…))))

which is equal to y=1/(2+ 1/(3+y)) solving we get

y = (3+y) /(7+2y) we get 

2y2 + 6y − 3 = 0 . 

Solution of this equation is {(15)-3}/2. 

Q. 17 N persons stand on the circumference of a circle at distinct points. Each possible pair of persons, not standing next to each other, sings a two-minute song one pair after the other. If the total time taken for singing is 28 minutes, what is N?  

A.

B.

C.

D. None of the above 

Answer: B. 

Explanation: 

Total number of pairs is NC2. Number of pairs standing next to each other = N. Therefore, number of pairs in question

= NC2 – N 

= 28/2 = 14. 

If N = 7,

7C2 – 7 = 21 – 7 = 14.

N = 7. 

Q. 18 A father and his son are waiting at a bus stop in the evening. There is a lamp post behind them. The lamp post, the father and his son stand on the same straight line. The father observes that the shadows of his head and his son’s head are incident at the same point on the ground. If the heights of the lamp post, the father and his son are 6 metres, 1.8 metres and 0.9 metres respectively, and the father is standing 2.1 metres away from the post then how far (in metres) is son standing form his father? 

A. 0.9 

B. 0.75 

C. 0.6 

D. 0.45 

Answer: D. 

Explanation: 

The three triangles are similar. 

Let the distance of the tip of the shadow from the child be y. Let the child be standing at distance x from the father. So, 6/(2.1+x+y) = 1.8/(x+y) = 0.9/y 

=> 2y = x+y => x = y 

=> 6/(2.1 + 2x) = 0.9/x 

=> 6x = 0.9*(2.1+2x) 

=> 6x = 1.89 + 1.8x 

=> 4.2x = 1.89 

=> x = 1.89/4.2 = 0.45 

Q. 19 Let f(x) = ax2 bx, where a and b are constants. Then at x = 0, f(x) is

A. maximized whenever a > 0, b > 0 

B. maximized whenever a > 0, b < 0 

C. minimized whenever a > 0, b > 0 

D. minimized whenever a > 0, b< 0 

Answer: D. 

Explanation: 

f(x) = ax2 bx∣ . When x = 0, f(x) = 0 

When a > 0  and b < 0, 

For x > 0, f(x) = ax2 bx, will be greater than 0 as ax2 > 0 and bx < 0 as b is negative and x is positive. 

For x < 0, f(x) = ax2 + bx will again be greater than 0 as ax2 > 0 and bx > 0 as both b and x are negative. 

Therefore, the function f(x) is positive when x < 0 and x > 0 when but becomes 0 when x = 0. 

Therefore, for a > 0 and b < 0, f(x) will attain its minimum value at x = 0. 

Q. 20 Each family in a locality has at most two adults, and no family has fewer than 3 children. 

Considering all the families together, there are adults than boys, more boys than girls, and more girls than families. 

Then the minimum possible number of families in the locality is 

A.

B.

C.

D.

Answer: D. 

Explanation: 

Consider 3 family. Let 1st one have 2 A , 1B and 2 G , 2nd one have 2 A , 2B and 1 G, 3rd one have 2 A , 2B and 1 G . SO total A-6 , B – 5 , g – 4 , F – 3 . 

Hence minimum is 3 . 

Q. 21 Let C be a circle with centre P0 and AB be a diameter of C. Suppose P1 is the mid point of the line segment P0B, P2, is the mid point of the line segment P1B and so on. Let C1, C2, C3, …  be circles with diameters  P0P1, P1P2, P2P3 respectively. Suppose the circles  C1, C2, C3, … are all shaded. The ratio of the C area of the unshaded portion of to that of the original circle is 

A. 8 : 9 

B. 9 : 10 

C. 10 : 11 

D. 11 : 12 

Answer: D. 

Explanation: 

Radius of the circles C1, C2, C3, … would be in GP with (R/4),(R/8),(R/16) and so on. Radii of circles are in the ratio 1:4. 

Ratio of unshaded region to the ratio of original circle = 1- [Ratio of shaded region / Ratio of original circle]

=>1- [πr2 /16+πr2 /64+….. / πr2]

=>1- [1/16 / 1-¼ ]

=> 1-1/12

=>11/12

=> 11:12

Q. 22 Consider the sequence of numbers a1, a2, a3 ……. to infinity where a1 = 81.33 and a2 = −19 andaj = aj−1 aj−2 for  j > 3. What is the sum of the first 6002 terms of this sequence? 

A. -100.33 

B. -30.00 

C. 62.33 

D. 119.33 

Answer: C. 

Explanation: 

According to given conditions the terms are 81.33, -19, -100.33, -81.33, 19, 100.33, 81.33,-19,.. Hence the series repeats after every 6 terms . Also summation of these 6 terms is 0. Hence summation is 60002 terms will we sum of first 2 terms which is 62.33. 

Q. 23 A sprinter starts running on a circular path of radius r metres. Her average speed (in metres/minute) is πr during the first 30 seconds, πr/2 during next one minute, πr/4 during next 2 minutes, πr/8 during next 4 minutes, and so on. What is the ratio of the time taken for the nth round to that for the previous round? 

A.

B.

C. 16 

D. 32 

Answer: C. 

Explanation: 

Let radius be 1 units and p = 3.14 or π. So circumference is 2 * π

According to given condition distance covered in first 1/2 mins = π/2 km, distance covered in next 1 min = π/2 km, 

distance covered in next 2 mins =π/2 km and finally distance covered in next 4 minutes = π/2km. 

Time taken to cover first round = 1/2 + 1 + 2 + 4 = 7.5 minutes.  

Now time taken to cover π/2 is in GP. 

For the second round the time taken is = 8+16+32+64 = 120 

Ratio = 120/7.5 = 16 

Q. 24 Let u = (log2 x)2 − 6log2 x + 12  where x is a real number. Then the equation xu = 256, has 

A. no solution for x 

B. exactly one solution for x 

C. exactly two distinct solutions for x 

D. exactly three distinct solutions for x 

Answer: B. 

Explanation: 

xu = 256 

Taking log to the base 2 on both the sides, 

u ∗ log2 x = log2 256 

=>[(log2 x)2 − 6log2 x + 12] ∗ log2 x = 8 

=>(log2x)3 − 6(log2x)2 + 12log2x = 8 

Let log2x =

t3 − 6t2 + 12t − 8 = 0 

(t − 2)3 = 0 

Therefore, log2x = 2 

=> x = 4 is the only solution 

Hence, option B. is the correct answer.

Q. 25 If the lengths of diagonals DF, AG and CE of the cube shown in the adjoining figure are equal to the three sides of a triangle, then the radius of the circle circumscribing that triangle will be?

A. equal to the side of the cube 

B. √3 times the side of the cube 

C. 1/√3 times the side of the cube 

D. impossible to find from the given information 

Answer: A. 

Explanation: 

Consider side of the cube as x. 

So diagonal will be of length √3* x. 

Now if diagonals are side of equilateral triangle we get area = 3*√3 ∗ x2 /4 . 

Also in a triangle 

4 * Area * R = Product of sides 

4* 3*√3 ∗ x2 /4 * R = .3* √3 ∗ x3

R = x 

Q. 26 In the adjoining figure, the lines represent one-way roads allowing travel only northwards or only westwards. Along how many distinct routes can a car reach point B from point A? 

A. 15 

B. 35 

C. 120 

D. 336 

Answer: B. 

Explanation: 

The person has to take 3 steps north and 4 steps west, in whatever way he travels. 

Total steps = 7, 3 north and 4 west. 

Number of ways = 7!/(4!3!) = 35 

Q. 27 On a semicircle with diameter AD, chord BC is parallel to the diameter. Further; AD and BC are separated by 2cm, while AD has length 8. What is the length of BC? 

A. 2√3

B. 4√3

C. 8.6 

D. None of the above 

Answer: B. 

Explanation: 

In the triangle BEO, BE =√42 − 22 = 2√3

=> BC = 4√3

Q. 28 A circle with radius 2 is placed against a right angle. Another smaller circle is also placed as shown in the adjoining figure. What is the radius of the smaller circle? 

A. 3 − 2√2

B. 4 − 2√2

C. 7 − 4√2

D. 6 − 4√2

Answer: D. 

Explanation: 

Using pythagoras we can find diagonal of small square shown in fig. = 2*√2 . Now this is equal to 2 + r + √2*r. 

Equating we get r = 6 − 4√2.

Q. 29  In the adjoining figure, chord ED is parallel to the diameter AC of the circle. If angle CBE = 65°, then what is the value of angle DEC? 

A. 35 

B. 55 

C. 45 

D. 25 

Answer: D. 

Explanation: 

If EBC = 65 then EOC = 130 then OEC = OCE = 25 NOw since OC and ED are parallel we have OCE = OED = 25 Hence option D 

Q. 30 The remainder, when (1523 + 23 ) is divided by 19, is 

A.

B. 15 

C.

D. 18 

Answer: C. 

Explanation: 

The remainder when 1523 is divided by 19 equals (−4)23

The remainder when 2323 is divided by 19 equals  423

So, the sum of the two equals (−4)23 + (4)23 = 0

Q. 31 A new flag is to be designed with six vertical stripes using some or all of the colours yellow, green, blue and red. Then, the number of ways this can be done such that no two adjacent stripes have the same colour is 

A. 12 × 81 

B. 16 × 192 

C. 20 × 125 

D. 24 × 216 

Answer: A. 

Explanation: 

The number of ways of selecting a colour for the first stripe is 4. The number of ways of selecting a colour for the second stripe is 3. Similarly, the number of ways of selecting colours for the third, fourth, fifth and sixth stripes are 3, 3, 3 and 3 respectively. 

The total number of ways of selecting the colours is, therefore, 4*3*3*3*3*3 = 12*81. 

Q. 32 If a/b+c =b/a+c =c/b+a = r, then r cannot take any value except 

A. 1/2 

B. -1 

C. 1/2 or -1 

D. -1/2 or -1 

Answer: C. 

Explanation: 

a = r(b+c) 

b = r(a+c) 

c = r(a+b) 

On adding all the equations, 

a+b+c = 2r(a+b+c) 

If r = 1/2, a+b+c = a+b+c (valid) 

If r = -1, a+b+c = -2(a+b+c) => a+b+c = 0 => b+c = -a and a/(b+c) = a/(-a) = -1 (valid) 

So, r can take the values 1/2 or -1 

Q. 33 Karan and Arjun run a 100-meter race, where Karan beats Arjun 10 metres. To do a favour to Arjun, Karan starts 10 metres behind the starting line in a second 100 metre race. They both run at their earlier speeds. Which of the following is true in connection with the second race? 

A. Karan and Arjun reach the finishing line simultaneously. 

B. Arjun beats Karan by 1 metre 

C. Arjun beats Karan by 11 metres. 

D. Karan beats Arjun by 1 metre. 

Answer: D. 

Explanation: 

The speeds of Karan and Arjun are in the ratio 10:9. Let the speeds be 10s and 9s. 

Time taken by Karan to cover 110 m = 110/10s = 11/s 

Time taken by Arjun to cover 100 m = 100/9s = 11.11/s 

Therefore, Karan reaches the finish line before Arjun. From the options, the only possible answer is d).

Q. 34 In NutsAndBolts factory, one machine produces only nuts at the rate of 100 nuts per minute and needs to be cleaned for 5 minutes after production of every 1000 nuts. 

Another machine produces only bolts at the rate of 75 bolts per minute and needs to be cleaned for 10 minutes after production of every 1500 bolts. If both the machines start production at the same time, what is the minimum duration required for producing 9000 pairs of nuts and bolts? 

A. 130 minutes 

B. 135 minutes 

C. 170 minutes 

D. 180 minutes 

Answer: C. 

Explanation: 

Machine A takes 15 min to produce 1000 nuts with clean time. machine b takes 30 min to make 1500 nuts with clean time . So B is slower. So with B 900 nuts will be made in 180 mins but at last round cleaning time of 10 min no need to count hence 170 mins 

Q. 35 The total number of integers pairs (x, y) satisfying the equation x + y = xy is 

A.

B.

C.

D. None of the above 

Answer: C. 

Explanation: 

xy = x + y 

=> xy – x – y = 0 

=> xy – x – y + 1 = 1 

=> (x – 1) (y – 1) = 1 

=> Both x – 1 and y – 1 have to be equal to 1 or -1. 

So, values taken by (x,y) are (2,2) and (0,0). 

=> 2 solutions 

Data Interpretation 

Instructions 

Directions for the following four questions: Answer the questions on the basis of the information given below. 

The Dean’s office recently scanned student results into the central computer system. When their character reading software cannot read something, it leaves the space blank. The scanner output read as follows: 

In the grading system, A, B, C, D, and F grades fetch 6, 4, 3, 2, and 0 grade points respectively. The Grade Point Average (GPA) is the arithmetic mean of the grade points obtained in the five subjects. For example Nisha’s GPA. is (6 + 2 + 4 + 6 + 0) / 5 = 3.6. Some additional facts are also known about the students’ grades. These are 

(i) Vipul obtained the same grade in Marketing as Aparna obtained in Finance and Strategy. (ii) Fazal obtained the same grade in Strategy as Utkarsh did in Marketing. 

(iii) Tara received the same grade in exactly three courses. 

Q. 36 What grade did Preeti obtain in Statistics? 

A.

B.

C.

D.

Answer: A. 

Explanation: 

Let preeti obtain x and y in statistics and operation respectively. 

Now we know that GPA. = 3.2 . So (x+y+2+2) = 16 . 

So x+y = 12. for this only one value is possible x=y=6 . 

So A grade . 

Q. 37 In operations, Tara could have received the same grade as 

A. Ismet 

B. Hari 

C. Jagdeep 

D. Manab 

Answer: D. 

Explanation: 

Since tara received the same grade in exactly 3 subjects , only one possibility is getting B. in 3 subjects and F in other 2. So in operation her grade would be either B or F.Among options only manab has B in operations . hence option D 

Q. 38 In Strategy, Gowri’s grade point was higher than that obtained by 

A. Fazal 

B. Hari 

C. Nisha 

D. Rahul 

Answer: B. 

Explanation: 

Knowing the GPA. we can find gawrys grade in strategy as 

16 + x=19; x=3. So her grade was C. 

Now considering hari’s grade 12+x+y=14 ; so x+y=2. 

So he must have got D or F in strategy. Hence in Strategy, Gowri’s grade point was higher than that obtained by hari. 

Q. 39 What grade did Utkarsh obtain in Finance? 

A.

B.

C.

D.

Answer: C. 

Explanation: 

Considering utkarsh grades we get , x+y = 6, So possibilities of his missing grades are (4,2) or (6,0). Now according to given condition, his grades in marketing are same as faizal’s in strategy and faizal got grade B in strategy . hence utkarsh got 4 in marketing and 2 in finance i.e. grade D . Hence option C 

Instructions 

Directions for the following four questions: Answer the questions on the basis of the information given below.

The data points in the figure below represent monthly income and expenditure data of individual members of the Ahuja family, the Bose family, the Coomar family, and the Dubey family. 

The X axis represents Expenditure and the Y axis represents the Income of the individual members. For these questions, savings is defined as Savings = Income – Expenditure 

Q. 40 Which family has the lowest average income? 

A. Ahuja 

B. Bose 

C. Coomar 

D. Dubey 

Answer: C. 

Explanation: 

SInce the Y-axis gives the income, to find minimum average income see if any family has income in different months as below as possible in the graph. 

From the graph we can see that coomar has lowest average income. Hence option C. 

Q. 41 Which family has the highest average expenditure? 

A. Ahuja 

B. Bose 

C. Coomar 

D. Dubey 

Answer: D. 

Explanation: 

Since X axis gives expenditure we have to see if average is as right as possible. Average expenditure of dubey family is rightmost. Hence, option D. 

Q. 42 Which family has the lowest average savings? 

A. Ahuja 

B. Bose 

C. Coomar 

D. Dubey 

Answer: D. 

Explanation: 

From the graph we can clearly make out that for dubey family in both the months income is nearly equal to expenditure, thus hey have lowest savings. 

Q. 43 The highest amount of savings accrues to a member of which family? 

A. Ahuja 

B. Bose 

C. Coomar 

D. Dubey 

Answer: A. 

Explanation: 

To have highest amount of savings, income should be high and expenditure should be low. For ahuja family the gap is highest . Hence they have highest amount of savings 

Instructions 

Directions for the following four questions: Answer the questions on the basis of the information given below. 

Prof. Singh has been tracking the number of visitors to his homepage? His service provider has provided him with the following data on the country of origin of the visitors and the university they belong to. 

Q. 44 To which country does University 5 belong? 

A. India or Netherlands but not USA. 

B. India or USA. but not Netherlands 

C. Netherlands or USA. but not India 

D. India or USA. but not UK 

Answer: A. 

Explanation: 

According to given conditions there are following possibilities of university belonging from specific country .

We can clearly make out that University 5 belongs to India or Netherlands but not USA. Hence option A. 

Q. 45 University 1 can belong to 

A. UK 

B. Canada 

C. Netherlands 

D. USA. 

Answer: C. 

Explanation: 

According to given conditions there are following possibilities of university belonging from specific country . 

We can see that University 1 can belong to Netherlands out of the given options. 

Q. 46 Which among the listed countries can possibly host three of the eight listed universities? 

A. None 

B. Only UK 

C. Only India 

D. Both India and UK 

Answer: A. 

Explanation: 

According to given conditions there are following possibilities of university belonging from specific country . 

Thus we can see that no country can can possibly host three of the eight listed universities.

Q. 47 Visitors from how many universities from UK visited Prof. Singh’s homepage in the three days? 

A.

B.

C.

D.

Answer: B. 

Explanation: 

On day 3, there were 2 visitors from UK and 1 from USA. On the same day, the site was visited by 2 persons from University 4 and 1 from University 6. So University 4 is located in UK and University 6 is in USA. Similar reasoning for day 2 gives us the conclusion that University 3 is located in Netherlands and University 8 is in India. On day 1, the number of visitors from USA. is 1 and that from University 6 is 1. University 6 is in USA. (derived above), which implies no other university is in USA. The number of visitors from India on day 1 is 1. Also, no visitor from University 8, which is in India has visited the site on day 1. This implies that one of University 1 and University 5 is in India and the other in Netherlands. A. similar logic gives us that one of University 2 and University 6 is in UK and the other in Canada. 

We can see that university 4 and university 7 from UK visited prof. Singh’s homepage in the three days. 

Instructions 

Directions for the following four questions: Answer the questions on the basis of the information given below.

Purana and Naya are two brands of kitchen mixer-grinders available in the local market. Purana is an old brand that was introduced in 1990, while Naya was introduced in 1997. For both these brands, 20% of the mixer-grinders bought in a particular year are disposed off as junk exactly two years later. It is knows that 10 Purana mixer-grinders were disposed off in 1997. The following figures show the number of Purana and Naya mixer-grinders in operation from 1995 to 2000, as at the end of the year. 

Q. 48 How many Naya mixer-grinders were purchased in 1999? 

A. 44 

B. 50 

C. 55 

D. 64 

Answer: B. 

Explanation: 

There were 30 naya mixture in 97′ which will also work in 98′ ,thus we have 50 new mixtures in 98′ . Now in 99′ 20% of 30 i.e. 6 are thrown in junk . So in 99′ we have 24 + 50 = 74 naya mixtures of previous years. So naya mixture grinder purchased in 99′ is 124-74 = 50. Hence option b. 

Q. 49 How many Naya mixer-grinders were disposed off by the end of 2000? 

A. 10 

B. 16 

C. 22 

D. Cannot be determined from the data 

Answer: B. 

Explanation: 

Number of Naya mixer-grinders were disposed off by the end of 1999 are 6 and at the end of 2000 are 10. So in total 16 naya mixer disposed off. 

Q. 50 How many Purana mixer-grinders were disposed off in 2000? 

A.

B.

C.

D. Cannot be determined from the data 

Answer: D. 

Explanation: 

Since 10 purana were disposed off in 1997, it means 50 new have been bought in 1995, which implies that 70 were already in use. The breakup of 70 purana cannot be determined. So the number of purana disposed in 2000 cannot be calculated. 

Q. 51 How many Purana mixer-grinders were purchased in 1999? 

A. 20 

B. 23 

C. 50 

D. Cannot be determined from the data 

Answer: A. 

Explanation: 

Purana mixer-grinders were purchased in 1999 = 236 – 222 +Disposed in 1999 

= 236-222 + 20%(Purana mixer-grinders were purchased in 1997) 

= 236 – 222 + 20%(182 – 162 + 10) = 236- 222 + 6 = 20. 

Hence option D. 

Instructions 

Directions for the following four questions: Answer the questions on the basis of the information given below. 

A. study was conduced to ascertain the relative importance that employees in five different countries assigned to five different traits in their Chief Executive Officers. The traits were compassion (C), decisiveness (D), negotiation skills (N), public visibility (P), and vision (V). The level of dissimilarity between two countries is the maximum difference in the ranks allotted by the two countries to any of the five traits. The following table indicates the rank order of the five traits for each country. 

Q. 52 Which of the following pairs of countries are most dissimilar? 

A. China and Japan 

B. India and China 

C. Malaysia and Japan 

D. Thailand and Japan 

Answer: D. 

Explanation: 

The maximum difference of the ranks between all traits is largest for Japan and Thailand which is 4. Hence option D. 

Q. 53 Which of the following countries is least dissimilar to India? 

A. China 

B. Japan 

C. Malaysia 

D. Thailand 

Answer: A. 

Explanation: 

If we find maximum of difference in ranks for all traits between India and others we find that the difference is least for India and china with just 2 points. Hence china is least dissimilar to India. 

 

Q. 54 Which amongst the following countries is most dissimilar to India? 

A. China 

B. Japan 

C. Malaysia 

D. Thailand 

Answer: B. 

Explanation: 

Maximum difference in ranks between Japan and India is 4 which is highest when compared to other countries. hence option B. 

Q. 55 Three of the following four pairs of countries have identical levels of dissimilarity. Which pair is the odd one out? 

A. Malaysia and China 

B. China and Thailand 

C. Thailand and Japan 

D. Japan and Malaysia 

Answer: D. 

Explanation: 

The maximum difference in ranks in any of the parameters is termed to be dissimilarity. Difference between the ranks of Malaysia and China in Vision (V) = 5-1 =4 Difference between the ranks of China and Thailand in Vision (V) = 5-1 = 4 Difference between the ranks of Thailand and Japan in Decisiveness (D) = 5 -1 =4 The maximum rank difference between Japan and Malaysia is in Vision (V) and Negotiation skills (N). The maximum difference in rank is 4. Therefore, option D. is the odd one. 

Instructions 

Directions for the following four questions: Answer the questions on the basis of the information given below.

Coach John sat with the score cards of Indian players from the 3 games in a one-day cricket tournament where the same set of players played for India and all the major batsmen got out. John summarized the batting performance through three diagrams, one for each games. In each diagram, the three outer triangles communicate the number of runs scored by the three top scores from India, where K, R, S, V, and Y represent Kaif, Rahul, Saurav, Virender, and Yuvraj respectively. The middle triangle in each diagram denotes the percentage of total score that was scored by the top three Indian scorers in that game. No two players score the same number of runs in the same game. John also calculated two batting indices for each player based on his scores in the tournaments; the R-index of a batsman is the difference between his highest and lowest scores in the 3 games while the M-index is the middle number, if his scores are arranged in a non-increasing order. 

Q. 56 For how many Indian players is it possible to calculate the exact M-index? 

A.

B.

C.

D. More than 2 

Answer: C. 

Explanation: 

We can clearly see that m index of only saurav and rahul is possible.Hence option C. 

Q. 57 Among the players mentioned, who can have the lowest R-index from the tournament? 

A. Only Kaif, Rahul or Yuvraj 

B. Only Kaif or Rahul 

C. Only Kaif or Yuvraj 

D. Only Kaif 

Answer: A. 

Explanation: 

We can find out that R- index can be lowest of any one out of Kaif, Rahul or Yuvraj. 

Q. 58 How many players among those listed definitely scored less than Yuvraj in the tournament? 

A.

B.

C.

D. More than 2 

Answer: B. 

Explanation: 

If we calculate total possible runs scored by each player we find that 1 player which is rahul definately scores below yuvraj. 

Q. 59 Which of the players had the best M-index from the tournament? 

A. Rahul 

B. Saurav 

C. Virender 

D. Yuvraj 

Answer: B. 

Explanation: 

Saurav has the best M-index . 

Instructions 

Directions for the following four questions: Answer the questions on the basis of the information given below. 

Twenty one participants from four continents (Africa, Americas, Australasia, and Europe) attended a United Nations conference. Each participant was an expert in one of four fields, labour, health, population studies, and refugee relocation. The following five facts about the participants are given. 

(a) The number of labour experts in the camp was exactly half the number of experts in each of the three other categories. 

(b) Africa did not send any labour experts. Otherwise, every continent, including Africa, sent at least one expert for each category. 

(c) None of the continents sent more than three experts in any category. 

(d) If there had been one less Australasian expert, then the Americas would have had twice as many experts as each of the other continents. 

(e) Mike and Alfanso are leading experts of population studies who attended the conference. They are from Australasia. 

Q. 60 Which of the following combinations is NOT possible? 

A. 2 experts in population studies from the Americas and 2 health experts from Africa attended the conference. 

B. 2 experts in population studies from the Americas and 1 health expert from Africa attended the conference. 

C. 3 experts in refugee relocation from the Americas and 1 health expert from Africa attended the conference. 

D. Africa and America each had 1 expert in population studies attending the conference. 

Answer: D. 

Explanation: 

According to given conditions, the possible solutions are as given in the tables below: 

From the above tables we can see that there was no possible case in which Africa and America had 1 expert each in population studies. Thus, statement D. is false. 

Q. 61 If Ramos is the lone American expert in population studies, which of the following is NOT true about the numbers of experts in the conference from the four continents? 

A. There is one expert in health from Africa. 

B. There is one expert in refugee relocation from Africa. 

C. There are two experts in health from the Americas. 

D. There are three experts in refugee relocation from the Americas. 

Answer: C. 

Explanation: 

According to given conditions, the possible solutions are as given in the tables below: 

From the tables we can see that when there is only 1 population expert from America, there are 3 American Health Experts. Hence option C. is not true. 

Q. 62 Alex, an American expert in refugee relocation, was the first keynote speaker in the conference. What can be inferred about the number of American experts in refugee relocation in the conference, excluding Alex? 

i. At least one 

ii. At most two 

A. Only i and not ii 

B. Only ii and not I 

C. Both i and ii 

D. Neither i nor ii 

Answer: C. 

Explanation: 

According to given conditions, the possible solutions are as given in the tables below: 

If an American expert in refugee relocation, was the first keynote speaker in the conference. Then apart from him there is atleast 1 and atmost 2 american expert in refugee relocation. Hence option C. 

Q. 63 Which of the following numbers cannot be determined from the information given? 

A. Number of labour experts from the Americas. 

B. Number of health experts from Europe. 

C. Number of health experts from Australasia. 

D. Number of experts in refugee relocation from Africa. 

Answer: D. 

Explanation: 

According to given conditions, the possible solutions are as given in the tables below:

We can see that there are 2 possibilities for number of experts in refugee relocation from Africa.Hence it cannot be 

determined. 

Instructions 

Directions for the following four questions: Answer the questions on the basis of the information given below. The year was 2006. All six teams in Pool A. of World Cup hockey, play each other exactly once. 

Each win earns a team three points, a draw earns one point and a loss earns zero points. The two teams with the highest points qualify for the semifinals. In case of a tie, the team with the highest goal difference (Goal For . Goals Against) qualifies. 

In the opening match, Spain lost to Germany. 

After the second round (after each team played two matches), the pool table looked as shown below. 

In the third round, Spain played Pakistan, Argentina played Germany, and New Zealand played South Africa. All the third round matches were drawn. The following are some results from the fourth and fifth round matches 

(a) Spain won both the fourth and fifth round matches. 

(b) Both Argentina and Germany won their fifth round matches by 3 goals to 0. 

(c) Pakistan won both the fourth and fifth round matches by 1 goal to 0. 

Q. 64 Which one of the following statements is true about matches played in the first two rounds? 

A. Germany beat New Zealand by 1 goal to 0. 

B. Spain beat New Zealand by 4 goals to 0. 

C. Spain beat South Africa by 2 goals to 0. 

D. Germany beat South Africa by 2 goals to 1. 

Answer: D. 

Explanation: 

Germany and Argentina have 2 games each and Spain and Pakistan have won 1 game each. We also know that Germany beat Spain. 

So Germany won its second game against one among SA/ NZ. 

Similarly Argentina must have won one game against Pakistan and another against SA/ NZ. Argentina scored 2 goals and conceded 0 goals. So it must have won both the games 1-0. NZ conceded 6 goals. The only possibility is it lost 5-1 to Spain and lost 1-0 to Argentina. As Spain conceded 2 goals it must have lost its opening game to Germany by 1-0. 

Now, the results of all the games can be known. 

Following are the possible match result after 2 rounds : 

HENCE OPTION D. IS THE CORRECT ANSWER. 

Q. 65 Which one of the following statements is true about matches played in the first two rounds? 

A. Pakistan beat South Africa by 2 goals to 1. 

B. Argentina beat Pakistan by 1 goal to 0. 

C. Germany beat Pakistan by 2 goals to 1. 

D. Germany beat Spain by 2 goals to 1. 

Answer: B. 

Explanation: 

Following are the possible match result of 2 rounds : 

Hence option D. is the correct answer. 

Q. 66 If Pakistan qualified as one of the two teams from Pool A, which was the other team that qualified? 

A. Argentina 

B. Germany 

C. Spain 

D. Cannot be determined 

Answer: D. 

Explanation: 

Data is inconsistent as from the statements we can infer that Spain, Germany, Argentina and Pakistan won their 5th round matches. But only 3 teams can win any round.Hence the following question can’t be answered. 

Q. 67 Which team finished at the top of the pool after five rounds of matches? 

A. Argentina 

B. Germany 

C. Spain 

D. Cannot be determined 

Answer: D. 

Explanation: 

Data is inconsistent as from the statements we can infer that Spain, Germany, Argentina and Pakistan won their 5th round matches. But only 3 teams can win any round as there are a total of 6 teams.Hence the following question can’t be answered. 

Instructions 

For the following questions answer them individually 

Q. 68 Zakib spends 30% of his income on his children’s education, 20% on recreation and 10% on healthcare. The corresponding percentage for Supriyo are 40%, 25%, and 13%. Who spends more on children’s education? 

A. Zakib spends more on recreation than Supriyo. 

B. Supriyo spends more on healthcare than Zakib. 

A. The question can be answered by using one of the statements alone but not by using the other statement alone. 

B. The question can be answered by using either of the statements alone. 

C. The question can be answered by using both statements together but not by either statement alone. 

D. The question cannot be answered on the basis of the two statements. 

Answer: A. 

Explanation: 

We have to find out if 3Z > 4S => Z > 1.33S 

According to statement 1, Z/5 > S/4 => Z > 1.25S 

According to statement 2, 13S > 10Z => Z < 1.3S 

So, using statement 2 alone, we can answer the question 

Q. 69 Four candidates for an award obtain distinct scores in a test. Each of the four casts a vote to choose the winner of the award. The candidate who gets the largest number of votes wins the award. In case of a tie in the voting process, the candidate with the highest score wins the award. Who wins the award? 

A. The candidates with top three scores each vote for the top score amongst the other three. 

B. The candidate with the lowest score votes for the player with the second highest score. 

A. The question can be answered by using one of the statements alone but not by using the other statement alone. 

B. The question can be answered by using either of the statements alone. 

C. The question can be answered by using both statements together but not by either statement alone. 

D. The question cannot be answered on the basis of the two statements. 

Answer: A. 

Explanation: 

Let the persons be A,B,C and D in order of the scores that is, A scores the highest and D the least. 

Statement A: A votes for B , B and C votes for A 

Even if D votes for B,then also A wins as A has higher score as compared to B 

A alone is sufficient. 

Statement B: D votes for B 

Nothing can be inferred from this statement. 

Q. 70 In a class of 30 students, Rashmi secured the third rank among the girls, while her brother Kumar studying in the same class secured the sixth rank in the whole class. Between the two, who had a better overall rank? 

A. Kumar was among the top 25% of the boys merit list in the class in which 60% were boys. 

B. There were three boys among the top five rank holders, and three girls among the top ten rank holders. 

A. The question can be answered by using one of the statements alone but not by using the other statement alone. 

B. The question can be answered by using either of the statements alone. 

C. The question can be answered by using both statements together but not by either statement alone. 

D. The question cannot be answered on the basis of the two statements. 

Answer: A. 

Explanation: 

We know that Rashmi secured the third rank among the girls, while her brother Kumar studying in the same class secured the sixth rank in the whole class. 

Considering statement A. we have 18 B. and 12 G and out of 18 boys kumar is among top 4 . But in this case we can’t get the relative posiiton of rashmi and kumar. 

Consider statement B. We know that among top 5 , 3 are boys and 2 are girls .So rashmi is not among top5 and we know that Kumar’s rank is 6 . So Rashmi is definately below Kumar. Hence, the question can be answered by using one of the statements alone but not by using the other statement alone. 

Q. 71 Tarak is standing 2 steps to the left of a red mark and 3 steps to the right of a blue mark. He tosses a coin. If it comes up heads, he moves one step to the right; otherwise he moves one step to the left. He keeps doing this until he reaches one of the two marks, and then he stops. 

At which mark does he stop? 

A. He stops after 21 coin tosses. 

B. He obtains three more tails than heads. 

A. The question can be answered by using one of the statements alone but not by using the other statement alone. 

B. The question can be answered by using either of the statements alone. 

C. The question can be answered by using both statements together but not by either statement alone. 

D. The question cannot be answered on the basis of the two statements. 

Answer: B. 

Explanation: 

Consider Statement A: 

B_ _x_R 

If the person has to reach red, then he has to take 2 steps in the most simplest manner. Now suppose he first takes left step and then takes a right step. He has taken 2 steps and has to take further 2 steps in order to reach red. In this way, a person has to take even number of steps to reach red. In the similar manner, a person has to take three steps to reach blue in the simplest manner. It means he would have to take odd number of steps to reach blue and since 21 is an odd number, therefore the person would reach blue step. 

Consider Statement B, 

It is given that the person obtains 3 more tails as compared to heads. Take the case of 1 head and four tails. In this case, he would reach blue step.Now take the case of 2 heads and 5 tails. In this case 2 heads are neutralised by 2 tails. The ramaining portion is 3 tails which would help to reach blue steps. In this manner, no matter how many heads are, three more tails would make sure that the person reaches blue corner. 

Q. 72 Ravi spent less than Rs. 75 to buy one kilogram each of potato, onion, and gourd. Which one of the three vegetables bought was the costliest? 

A. 2 kg potato and 1 kg gourd cost less than 1 kg potato and 2 kg gourd. 

B. 1 kg potato and 2 kg onion together cost the same as 1 kg onion and 2 kg gourd. 

A. The question can be answered by using one of the statements alone but not by using the other statement alone.

B. The question can be answered by using either of the statements alone. 

C. The question can be answered by using both statements together but not by either statement alone. 

D. The question cannot be answered on the basis of the two statements. 

Answer: C. 

Explanation: 

Using statement 1 alone, 2P + G < P + 2G 

=> P < G 

Using statement 2 alone, P + 2O = O + 2G 

=> P + O = 2G 

So, using either statement alone, we cannot find the answer to the question. 

However, by using both the statements together, we can determine that P < G < O 

So, option c) is the answer. 

Q. 73 Nandini paid for an article using currency notes of denominations Re. 1, Rs. 2, Rs. 5, and Rs. 10 using at least one note of each denomination. The total number of five and ten rupee notes used was one more than the total number of one and two rupee notes used. What was the price of the article? 

A. Nandini used a total of 13 currency notes. 

B. The price of the article was a multiple of Rs. 10. 

A. The question can be answered by using one of the statements alone but not by using the other statement alone. 

B. The question can be answered by using either of the statements alone. 

C. The question can be answered by using both statements together but not by either statement alone. 

D. The question cannot be answered on the basis of the two statements. 

Answer: D. 

Explanation: 

Let the no. of 1,2,5,10 Rs notes be x,y,z,w. 

According to given condition we have z+w=x+y+1 . 

Now considering statement A. 

The possible values are 4+3=3+3+1 . 

But in this case the data is not sufficient to find price of article as the both 5 and 10 rs can have any of the 4 or 3 notes. 

Hence statement A. is not sufficient. 

Now considering statement B. , 

The price of the article was a multiple of Rs. 10. 

For this, more than 1 possibility of no. of notes of a particular value is there. Also 2 statements together can’t give the total price of the article. 

Verbal 

Instructions 

Directions for the following ten questions: Fill up the blanks, numbered [1], [2] up to [10], in the two passages below with most appropriate word from the options given for each blank. 

First Passage: 

“Between the year 1946 and the year 1955, I did not file any income tax returns.” With that [1] statement, Ramesh embarked on an account of his encounter with the income tax department. “I originally owned Rs. 20,000 in unpaid taxes. With [2] and [3], the 20,000 became 60,000. The Income tax Department then went into action, and I learned first hand just how much power the Tax Department wields. Royalties and trust funds can be [4]; automobiles may be [5], and auctioned off. Nothing belongs to the [6] until the case is settled.” 

Second Passage: 

“At that time the white house was as serene as a resort hotel out of season. The corridors were [7]. In the various offices, [8] gray men in waistcoats talked to one another in low-pitched voices. The only color, or choler, curiously enough, was provided by President Eisenhower himself. Apparently, his [9] was easily set off; he scowled when he [10] the corridors. 

Q. 74 Blank Number [1] 

A. devious 

B. blunt 

C. tactful 

D. pretentious 

Answer: B. 

Explanation: 

The most appropriate option is blunt. Ramesh has made direct statement which can be said to be blunt. 

Q. 75 Blank Number [2] 

A. interest 

B. taxes 

C. principal 

D. returns 

Answer: A. 

Explanation: 

The principal is mentioned as 20,000. Since he hasn’t paid this amount, he has to pay the interest. 

Q. 76 Blank Number [3] 

A. sanctions 

B. refunds 

C. fees 

D. fines 

Answer: D. 

Explanation: 

Due to late payments, there were fines. 

Q. 77 Blank Number [4] 

A. closed 

B. detached 

C. attached 

D. impounded 

Answer: C. 

Explanation: 

Of the given options, only option C and D mean take hold of. Between the two, option C is used for funds and sources of income. Impounded is used for taking possession of physical assets and thus cannot be used for sources of income. Thus, the right answer is option C. 

Q. 78 Blank Number [5] 

A. smashed 

B. seized 

C. dismantled 

D. frozen 

Answer: B. 

Explanation: 

Before auctioning, automobiles are seized and not smashed or dismantled. 

Q. 79 Blank Number [6] 

A. purchaser 

B. victim 

C. investor 

D. offender 

Answer: D. 

Explanation: 

Those who evade income tax are offenders rather than a victim, purchaser or an investor. 

Q. 80 Blank Number [7] 

A. striking 

B. hollow 

C. empty 

D. white 

Answer: C. 

Explanation: 

The best word attached with the corridor is empty rather than hollow, striking or white because in the previous line it is mentioned that white house was serene. 

Q. 81 Blank Number [8] 

A. quiet 

B. faded 

C. loud 

D. stentorian 

Answer: A. 

Explanation: 

Since low-pitched voice is mentioned, the most appropriate word is quiet. 

Q. 82 Blank Number [9] 

A. laughter 

B. curiously 

C. humour 

D. temper 

Answer: D. 

Explanation: 

“Choler” means anger. The most appropriate word which goes with choler is temper. 

Q. 83 Blank Number [10] 

A. paced 

B. strolled 

C. stormed 

D. prowled 

Answer: A. 

Explanation: 

When someone is in a temper, he usually moves fast. “Paced” is the most appropriate word in this context. 

Instructions 

The painter is now free to paint anything he chooses. There are scarcely any forbidden subjects, and today everybody is prepared to admit that a painting of some fruit can be as important as painting of a hero dying. The Impressionists did as much as anybody to win this previously unheard of freedom for the artist. Yet, by the next generation, painters began to abandon tie subjects altogether, and began to paint abstract pictures. Today the majority of pictures painted are abstract. 

Is there a connection between these two developments? Has art gone abstract because the artist is embarrassed by his freedom? Is it that, because he is free to paint anything, he doesn’t know what to paint? Apologists for abstract art often talk of it as Inc art of maximum freedom. But could this be the freedom of the desert island? It would take too long to answer these questions properly. I believe there is a connection. Many things have encouraged the development of abstract art. Among them has been the artists’ wish to avoid the difficulties of finding subjects when all subjects are equally possible. 

I raise the matter now because I want to draw attention to the fact that the painter’s choice of a subject is a far more complicated question than it would at first seem. A. subject does not start with what is put in front of the easel or with something which the painter happens to remember. A. subject starts with the painter deciding he would like to paint such-and-such because for some reason or other he finds it meaningful. A. subject begins when the artist selects something for special mention. (What makes it special or meaningful may seem to the artist to be purely visual — its colours or its form.) When the subject has been selected, the function of the painting itself is to communicate and justify the significance of that selection. 

It is often said today that subject matter is unimportant. But this is only a reaction against the excessively literary and moralistic interpretation of subject matter in the nineteenth century. In truth the subject is literally the beginning and end of a painting. The painting begins with a selection (I will paint this and not everything else in the world); it is finished when that selection is justified (now you can see all that I saw and felt in this and how it is more than merely itself). 

Thus, for a painting to succeed it is essential that the painter and his public agree about what is significant. The subject may have a personal meaning for the painter or individual spectator; but there must also be the possibility of their agreement on its general meaning. It is at this point that the culture of the society and period in question precedes the artist and his art. Renaissance art would have meant nothing to the Aztecs — and vice versa. If, to some extent, a few intellectuals can appreciate them both today it is because their culture is an historical one: its inspiration is history and therefore it can include within itself, in principle if not in every particular, all known developments to date. 

When culture is secure and certain of its values, it presents its artists with subjects. The general agreement about what is significant is so well established that the significance of a particular subject accrues and becomes traditional. This is true, for instance, of reeds and water in China, of the nude body in Renaissance, of the animal in Africa. Furthermore in such cultures the artist is unlikely to be a free agent: he will be employed for the sake of particular subjects, and the problem, as we have just described it, will not occur to him. 

When a culture is in a state of disintegration or transitions the freedom of the artist increases — but the question of subject matter becomes problematic for him: he, himself, has to choose for society. This was at the basis of all the increasing crises in European art during the nineteenth century. It is too often forgotten how any of the art scandals of that time were provoked by the choice of subject (Gericault, Courbet, Daumier, Degas, Lautrec, Van Gogh, etc.). 

By the end of the nineteenth century there were, roughly speaking, two ways in which the painter could meet this challenge of deciding what to paint and so choosing for society. Either he identified himself with the people and so 

allowed their lives to dictate his subjects to him or he had to find his subjects within himself as painter. By people I mean everybody except the, bourgeoisie. Many painters did of course work for the bourgeoisie according to their copy book of approved subjects, but all of them, filling the Salon and the Royal Academy year after year, are now forgotten, buried under the hypocrisy of those they served so sincerely. 

Q. 84 When a culture is insecure, the painter chooses his subject on the basis of: 

A. The prevalent style in the society of his time. 

B. Its meaningfulness to the painter. 

C. What is put in front of the easel. 

D. Past experience and memory of the painter 

Answer: B. 

Explanation: 

Refer to the lines:”When a culture is in a state of disintegration or transitions the freedom of the artist increases — but the question of subject matter becomes problematic for him: he, himself, has to choose for society.” This implies option 2. 

Q. 85 In the sentence, “I believe there is a connection” (second paragraph), what two developments is the author referring to? 

A. Painters using a dying hero and using a fruit as a subject of painting. 

B. Growing success of painters and an increase in abstract forms. 

C. Artists gaining freedom to choose subjects and abandoning subjects altogether. 

D. Rise of Impressionists and an increase in abstract forms. 

Answer: C. 

Explanation: 

In first para we find the 2nd last sentence as ‘… by the next generation, painters began to abandon tie subject altogether, and began to paint abstract pictures…’ Then second para is continued on connection between these 2 developments. Hence option C is the correct answer. 

Q. 86 Which of the following is NOT necessarily among the attributes needed for a painter to succeed: 

A. The painter and his public agree on what is significant. 

B. The painting is able to communicate and justify the significance of its subject selection. 

C. The subject has a personal meaning for the painter. 

D. The painting of subjects is inspired by historical developments. 

Answer: C. 

Explanation: 

Refer to the lines of the para:”The subject may have a personal meaning for the painter or individual spectator; but there must also be the possibility of their agreement on its general meaning. It is at this point that the culture of the society and period in question precedes the artist and his art.” This implies that the subject need not necessarily have a personal meaning for the painter. 

Q. 87 In the context of the passage, which of the following statements would NOT be true? 

A. Painters decided subjects based on what they remembered from their own lives. 

B. Painters of reeds and water in China faced no serious problem of choosing a subject. 

C. The choice of subject was a source of scandals in nineteenth century European art. 

D. Agreement on the general meaning of a painting is influenced by culture and historical context. 

Answer: A. 

Explanation: 

Refer to the lines of the third paragraph:”A subject does not start with what is put in front of the easel or with something which the painter happens to remember.” 

Q. 88 Which of the following views is taken by the author? 

A. The more insecure a culture, the greater the freedom of the artist. 

B. The more secure a culture, the greater the freedom of the artist. 

C. The more secure a culture, more difficult the choice of subject. 

D. The more insecure a culture, the less significant the choice of the subject. 

Answer: A. 

Explanation: 

Refer to the following lines of the paragraph:”When a culture is in a state of disintegration or transitions the freedom of the artist increases — but the question of subject matter becomes problematic for him: he, himself, has to choose for society.” 

Instructions 

Recently I spent several hours sitting under a tree in my garden with the social anthropologist William Ury, a Harvard University professor who specializes in the art of negotiation and wrote the bestselling book, Getting to Yes. He captivated me with his theory that tribalism protects people from their fear of rapid change. He explained that the pillars of tribalism that humans rely on for security would always counter any significant cultural or social change. In this way, he said, change is never allowed to happen too fast. Technology, for example, is a pillar of society. Ury believes that every time technology moves in a new or radical direction, another pillar such as religion or nationalism will grow stronger in effect, the traditional and familiar will assume greater importance to compensate for the new and untested. In this manner, human tribes avoid rapid change that leaves people insecure and frightened. 

But we have all heard that nothing is as permanent as change. Nothing is guaranteed. Pithy expressions, to be sure, but no more than cliches. As Ury says, people don’t live that way from day-to-day. On the contrary, they actively seek certainty and stability. They want to know they will be safe. 

Even so we scare ourselves constantly with the idea of change. An IBM CEO once said: ‘We only re-structure for a good reason, and if we haven’t re-structured in a while, that’s a good reason.’ We are scared that competitors, technology and the consumer will put us Out of business — so we have to change all the time just to stay alive. But if we asked our fathers and grandfathers, would they have said that they lived in a period of little change? Structure may not have changed much. It may just be the speed with which we do things. 

Change is over-rated, anyway, consider the automobile. It’s an especially valuable example, because the auto industry has spent tens of billions of dollars on research and product development in the last 100 years. Henry Ford’s first car had a metal chassis with an internal combustion, gasoline-powered engine, four wheels with rubber types, a foot operated clutch assembly and brake system, a steering wheel, and four seats, and it could safely do 1 8 miles per hour. A. hundred years and tens of thousands of research hours later, we drive cars with a metal chassis with an internal combustion, gasoline-powered engine, four wheels with rubber tyres a foot operated clutch assembly and brake system, a steering wheel, four seats – and the average speed in London in 2001 was 17.5 miles per hour! 

 

That’s not a hell of a lot of return for the money. Ford evidently doesn’t have much to teach us about change. The fact that they’re still manufacturing cars is not proof that Ford Motor Co. is a sound organization, just proof that it takes very large companies to make cars in great quantities — making for an almost impregnable entry barrier. 

Fifty years after the development of the jet engine, planes are also little changed. They’ve grown bigger, wider and can carry more people. But those are incremental, largely cosmetic changes. 

Taken together, this lack of real change has come to man that in travel — whether driving or flying — time and technology have not combined to make things much better. The safety and design have of course accompanied the times and the new volume of cars and flights, but nothing of any significance has changed in the basic assumptions of the final product. 

At the same time, moving around in cars or aero-planes becomes less and less efficient all the time Not only has there been no great change, but also both forms of transport have deteriorated as more people clamour to use them. The same is true for telephones, which took over hundred years to become mobile or photographic film, which also required an entire century to change. 

The only explanation for this is anthropological. Once established in calcified organizations, humans do two things: sabotage changes that might render people dispensable, and ensure industry-wide emulation. In the 960s, German auto companies developed plans to scrap the entire combustion engine for an electrical design. (The same existed in the 1970s in Japan, and in the 1980s in France.) So for 40 years we might have been free of the wasteful and ludicrous dependence on fossil fuels. Why didn’t it go anywhere? Because auto executives understood pistons and carburettors, and would be loath to cannibalize their expertise, along with most of their factories 

Q. 89 According to the above passage, which of the following statements is true? 

A. Executives of automobile companies are inefficient and ludicrous. 

B. The speed at which an automobile is driven in a city has not changed much in a century. 

C. Anthropological factors have fostered innovation in automobiles by promoting use of new technologies. 

D. Further innovation in jet engines has been more than incremental. 

Answer: B. 

Explanation: 

In the fourth paragraph it is clearly mentioned that change is over-rated. Refer to the following lines:”Change is over rated, anyway, consider the automobile. It’s an especially valuable example, because the auto industry has spent tens of billions of dollars on research and product development in the last 100 years.” 

Q. 90 Which of the following views does the author fully support in the passage? 

A. Nothing is as permanent as change. 

B. Change is always rapid. 

C. More money spent on innovation leads to more rapid change. 

D. Over decades, structural change has been incremental. 

Answer: D. 

Explanation: 

Refer to the following lines of the paragraph:”But if we asked our fathers and grandfathers, would they have said that they lived in a period of little change? Structure may not have changed much. It may just be the speed with which we do things.” 

Q. 91 Which of the following best describes one of the main ideas discussed in the passage? 

A. Rapid change is usually welcomed in society. 

B. Industry is not as innovative as it is made out to be. 

C. We should have less change than what we have now. 

D. Competition spurs companies into radical innovation. 

Answer: B. 

Explanation: 

Clearly the second option is the main idea of the passage. In this passage the author talks about the over-exaggeration made by automobile industry and airplane industry while on the whole there are not many changes which have taken place. 

Q. 92 According to the passage, the reason why we continues to be dependent on fossil fuels is that: 

A. Auto executives did not wish to change. 

B. No alternative fuels were discovered. 

C. Change in technology was not easily possible 

D. German, Japanese and French companies could not come up with new technologies. 

Answer: A. 

Explanation: 

Refer to the following lines of the passage:”Why didn’t it go anywhere? Because auto executives understood pistons and carburettors, and would be loath to cannibalize their expertise, along with most of their factories”. This indicates that the auto executives do not want to change themselves. 

Instructions 

The viability of the multinational corporate system depends upon the degree to which people will tolerate the unevenness it creates. It is well to remember that the ‘New Imperialism’ which began after 1870 in a spirit of Capitalism Triumphant, soon became seriously troubled and after 1914 was characterized by war, depression, breakdown of the international economic system and war again, rather than Free Trade, Pax Britannica and Material Improvement. A. major reason was Britain’s inability to cope with the byproducts of its own rapid accumulation of capital; i.e., a class-conscious labour force at home; a middle class in the hinterland; and rival centres of capital on the Continent and in America. Britain’s policy tended to be atavistic and defensive rather than progressive — more concerned with warding off new threats than creating new areas of expansion. Ironically, Edwardian England revived the paraphernalia of the landed aristocracy it had just destroyed. Instead of embarking on a ‘big push’ to develop the vast hinterland of the Empire, colonial administrators often adopted policies to arrest the development of either a native capitalist class or a native proletariat which could overthrow them. 

As time went on, the centre had to devote an increasing share of government activity to military and other unproductive expenditures; they had to rely on alliances with an inefficient class of landlords, officials and soldiers in the hinterland to maintain stability at the cost of development. A. great part of the surplus extracted from the population was thus wasted locally. 

The New Mercantilism (as the Multinational Corporate System of special alliances and privileges, aid and tariff concessions is sometimes called) faces similar problems of internal and external division. The centre is troubled: excluded groups revolt and even some of the affluent are dissatisfied with the roles. Nationalistic rivalry between major capitalist countries remains an important divisive factor. Finally, there is the threat presented by the middle classes and the excluded groups of the underdeveloped countries. The national middle classes in the underdeveloped countries came to power when the centre weakened but could not, through their policy of import substitution manufacturing, establish a viable basis for sustained growth. They now face a foreign exchange crisis and an unemployment (or population) crisis—the first indicating their inability to function in the international economy and the second indicating their alienation from the people they are supposed to lead. In the immediate future, these national middle classes will gain a new lease of life as they take advantage of the spaces created by the rivalry between American and non-American oligopolists striving to establish global market positions. 

 

The native capitalists will again become the champions of national independence as they bargain with multinational corporations. But the conflict at this level is more apparent than real, for in the end the fervent nationalism of the middle class asks only for promotion within the corporate structure and not for a break with that structure. In the last analysis their power derives from the metropolis and they cannot easily afford to challenge the international system. They do not command the loyalty of their own population and cannot really compete with the large, powerful, aggregate capitals from the centre. They are prisoners of the taste patterns and consumption standards’ set at the centre. 

The main threat comes from the excluded groups. It is not unusual in underdeveloped countries for the top 5 per cent to obtain between 30 and 40 per cent of the total national income, and for the top one-third to obtain anywhere from 60 to 70 per cent. At most, one-third of the population can be said to benefit in some sense from the dualistic growth that characterizes development in the hinterland. The remaining two-thirds, who together get only one-third of the income, are outsiders, not because they do not contribute to the economy, but because they do not share in the benefits. They provide a source of cheap labour which helps keep exports to the developed world at a low price and which has financed the urban-biased growth of recent years. In fact, it is difficult to see how the system in most underdeveloped countries could survive without cheap labour since removing it (e.g. diverting it to public works projects as is done in socialist countries) would raise consumption costs to capitalists and professional elites. 

 

Q. 93 According to the author, the British policy during the “New Imperialism” period tended to be defensive because 

A. it was unable to deal with the fallouts of a sharp increase in capital. 

B. its cumulative capital had undesirable side-effects. 

C. its policies favoured developing the vast hinterland. 

D. it prevented the growth of a set-up which could have been capitalistic in nature. 

Answer: A. 

Explanation: 

Refer to the following lines of the paragraph:”A. major reason was Britain’s inability to cope with the byproducts of its own rapid accumulation of capital; i.e., a class-conscious labour force at home; a middle class in the hinterland; and rival centres of capital on the Continent and in America. Britain’s policy tended to be atavistic and defensive rather than progressive — more concerned with warding off new threats than creating new areas of expansion.” 

Hence, the answer is either A. or B. Option B. incorrectly places the blame on the capital – instead of on Britain’s inability to deal with it. Britain was defensive because it could not effectively deal with the problems that arose. Hence, option A. is better. 

Q. 94 Under New Mercantilism, the fervent nationalism of the native middle classes does not create conflict with the multinational corporations because they (the middle classes) 

A. negotiate with the multinational corporations 

B. are dependent on the international system for their continued prosperity. 

C. are not in a position to challenge the status quo. 

D. do not enjoy popular support. 

Answer: C. 

Explanation: 

Consider the second last paragraph for the answer:”In the last analysis their power derives from the metropolis and they cannot easily afford to challenge the international system. They do not command the loyalty of their own population and cannot really compete with the large, powerful, aggregate capitals from the centre. They are prisoners of the taste patterns and consumption standards’ set at the centre.” This implies that they are not in position to challenge the status-quo. 

Q. 95 In the sentence, “They are prisoners of the taste patterns and consumption standards set at the center.” (fourth paragraph), what is the meaning of “centre”? 

A. National government 

B. Native capitalists. 

C. New capitalists. 

D. None of the above. 

Answer: D. 

Explanation: 

Throughout the passage, the author uses the term “centre” to refer to the multinational corporations that are at the centre of power in the multi-national corporate system. Hence, the answer is none of the above. 

Q. 96 The author is in a position to draw parallels between New Imperialism and New Mercantilism because 

A. both originated in the developed Western capitalist countries. 

B. New Mercantilism was a logical sequel to New Imperialism 

C. they create the same set outputs . a labour force, middle classes and rival centers of capital. 

D. both have comparable uneven and divisive effects. 

Answer: D. 

Explanation: 

Through the first two paragraphs, the author highlights how New Imperialism created uneven systems which in turn created class divisions. The author then goes on to say that New Mercantilism suffers from the same issues. Hence, option D correctly captures the parallels between the two systems. 

Options A and B have not been implied in the passage. Thus, we can eliminate these options. 

Option C is partly true. It was true for New Imperialism but in the case of New Mercantilism there are divisions along multiple lines – nationalistic rivalries between countries, difference between middle classes and underdeveloped groups in developing vs developed countries etc. Moreover, these divisions are issues presented by the system. They are not outputs of the system. Hence, option C is incorrect. 

Thus, option D is the right answer. 

Instructions 

Fifty feet away three male lions lay by the road. They didn’t appear to have a hair on their heads. Noting the color of their noses (leonine noses darken as they age, from pink to black), Craig estimated that they were six years old — young adults. “This is wonderful!” he said, after staring at them for several moments. “This is what we came to see. They really are maneless.” Craig, a professor at the University of Minnesota, is arguably the leading expert on the majestic Serengeti lion, whose head is mantled in long, thick hair. He and Peyton West, a doctoral student who has been working with him in Tanzania, had never seen the Tsavo lions that live some 200 miles east of the Serengeti. The scientists had partly suspected that the maneless males were adolescents mistaken for adults by amateur observers. 

Now they knew better. 

The Tsavo research expedition was mostly Peyton’s show. She had spent several years in Tanzania, compiling the data she needed to answer a question that ought to have been answered long ago: Why do lions have manes? It’s the only cat, wild or domestic, that displays such ornamentation. In Tsavo she was attacking the riddle from the opposite angle. Why do its lions not have manes? (Some “maneless” lions in Tsavo East do have partial manes, but they rarely attain the regal glory of the Serengeti lions.) Does environmental adaptation account for the trait? Are the lions of Tsavo, as some people believe, a distinct subspecies of their Serengeti cousins? 

The Serengeti lions have been under continuous observation for more than 35 years, beginning with George Schaller’s pioneering work in the 1960s. But the lions in Tsavo, Kenya’s oldest and largest protected ecosystem, have hardly been studied. Consequently, legends have grown up around them. Not only do they look different, according to the myths, they behave differently, displaying greater cunning and aggressiveness. “Remember too,” Kenya: The Rough Guide warns, “Tsavo’s lions have a reputation of ferocity.” Their fearsome image became well-known in 1898, when two males stalled construction of what is now Kenya Railways by allegedly killing and eating 135 Indian and African laborers. A British Army officer in charge of building a railroad bridge over the Tsavo River, Lt. Col. J. H. Patterson, spent nine months pursuing the pair before he brought them to bay and killed them. Stuffed and mounted, they now glare at visitors to the Field Museum in Chicago. Patterson’s account of the leonine reign of terror, The Man-Eaters of Tsavo, was an international best seller when published in 1907. Still in print, the book has made Tsavo’s lions notorious. That annoys some scientists. “People don’t want to give up on mythology,” Dennis King told me one day. The zoologist has been working in Tsavo off and on for four years. “I am so sick of this maneater business. Patterson made a helluva lot of money off that story, but Tsavo’s lions are no more likely to turn man-eater than lions from elsewhere.” 

But tales of their savagery and wiliness don’t all come from sensationalist authors looking to make a buck. Tsavo lions are generally larger than lions elsewhere, enabling them to take down the predominant prey animal in Tsavo, the Cape buffalo, one of the strongest, most aggressive animals of Earth. The buffalo don’t give up easily: They often kill or severely injure an attacking lion, and a wounded lion might be more likely to turn to cattle and humans for food. 

And other prey is less abundant in Tsavo than in other traditional lion haunts. A. hungry lion is more likely to attack humans. Safari guides and Kenya Wildlife Service rangers tell of lions attacking Land Rovers, raiding camps, stalking tourists. Tsavo is a tough neighborhood, they say, and it breeds tougher lions. 

But are they really tougher? And if so, is there any connection between their manelessness and their ferocity? An intriguing hypothesis was advanced two years ago by Gnoske and Peterhans: Tsavo lions may be similar to the unmaned cave lions of the Pleistocene. The Serengeti variety is among the most evolved of the species — the latest model, so to speak — while certain morphological differences in Tsavo lions (bigger bodies, smaller skulls, and maybe even lack of a mane) suggest that they are closer to the primitive ancestor of all lions. Craig and Peyton had serious doubts about this idea, but admitted that Tsavo lions pose a mystery to science. 

Q. 97 The book Man-Eaters of Tsavo annoys some scientists because 

A. it revealed that Tsavo lions are ferocious. 

B. Patterson made a helluva lot of money from the book by sensationalism. 

C. it perpetuated the bad name Tsavo lions had. 

D. it narrated how two male Tsavo lions were killed. 

Answer: C. 

Explanation: 

Refer to the following lines of the third para:”Patterson’s account of the leonine reign of terror, The Man-Eaters of Tsavo, was an international best seller when published in 1907. Still in print, the book has made Tsavo’s lions notorious.” 

Q. 98 The sentence which concludes the first paragraph, “Now they knew better”, implies that: 

A. The two scientists were struck by wonder on seeing maneless lions for the first time. 

B. Though Craig was an expert on the Serengeti lion, now he also knew about the Tsavo lions. 

C. Earlier, Craig and West thought that amateur observers had been mistaken. 

D. Craig was now able to confirm that darkening of the noses as lions aged applied toTsavo lions as well. 

Answer: C. 

Explanation: 

Refer to the line just behind the given phrase:”The scientists had partly suspected that the maneless males were adolescents mistaken for adults by amateur observers. Now they knew better.” This implies option c. 

Q. 99 According to the passage, which of the following has NOT contributed to the popular image of Tsavo lions as savage creatures? 

A. Tsavo lions have been observed to bring down one of the strongest and most aggressive animals . the Cape buffalo. 

B. In contrast to the situation in traditional lion haunts, scarcity of non-buffalo prey in the Tsavo makes the Tsavo lions more aggressive. 

C. The Tsavo lion is considered to be less evolved than the Serengeti variety. 

D. Tsavo lions have been observed to attack vehicles as well as humans. 

Answer: C. 

Explanation: 

All other choices are there in 4th and 5th paragraph: Refer to these lines “But tales of their savagery and wiliness don’t all come from sensationalist authors looking to make a buck. Tsavo lions are generally larger than lions elsewhere, enabling them to take down the predominant prey animal in Tsavo, the Cape buffalo, one of the strongest, most aggressive animals of Earth. The buffalo don’t give up easily: They often kill or severely injure an attacking lion, and a wounded lion might be more likely to turn to cattle and humans for food. 

 

And other prey is less abundant in Tsavo than in other traditional lion haunts. A. hungry lion is more likely to attack humans. Safari guides and Kenya Wildlife Service rangers tell of lions attacking Land Rovers, raiding camps, stalking tourists. Tsavo is a tough neighborhood, they say, and it breeds tougher lions.” 

Q. 100 Which of the following, if true, would weaken the hypothesis advanced by Gnoske and Peterhans most? 

A. Craig and Peyton develop even more serious doubts about the idea that Tsavo lions are primitive. 

B. The maneless Tsavo East lions are shown to be closer to the cave lions. 

C. Pleistocene cave lions are shown to be far less violent than believed. 

D. The morphological variations in body and skull size between the cave and Tsavo lions are found to be insignificant. 

Answer: C. 

Explanation: 

Option c satisfies the criteria the most. If cave lions are far less violent, then Tsavo would also be less violent whereas in the paragraph, tsavo lions are shown to be more violent. 

Instructions 

Throughout human history the leading causes of death have been infection and trauma, Modem medicine has scored significant victories against both, and the major causes of ill health and death are now the chronic degenerative diseases, such as coronary artery disease, arthritis, osteoporosis, Alzheimer’s, macular degeneration, cataract and cancer. These have a long latency period before symptoms appear and a diagnosis is made. It follows that the majority of apparently healthy people are pre-ill. 

But are these conditions inevitably degenerative? A truly preventive medicine that focused on the pre-ill, analyzing the metabolic errors which lead to clinical illness, might be able to correct them before the first symptom. Genetic risk factors are known for all the chronic degenerative diseases, and are important to the individuals who possess them. At the population level, however, migration studies confirm that these illnesses are linked for the most part to lifestyle factors — exercise, smoking and nutrition. Nutrition is the easiest of these to change, and the most versatile tool for affecting the metabolic changes needed to tilt the balance away from disease. 

Many national surveys reveal that malnutrition is common in developed countries. This is not the calorie and/or micronutrient deficiency associated with developing nations (type A. malnutrition); but multiple micronutrient depletion, usually combined with calorific balance or excess (Type B. malnutrition). The incidence and severity of Type B. malnutrition will be shown to be worse if newer micronutrient groups such as the essential fatty acids, xanthophylls and falconoid are included in the surveys. Commonly ingested levels of these micronutrients seem to be far too low in many developed countries. 

There is now considerable evidence that Type B malnutrition is a major cause of chronic degenerative diseases. If this is the case, then t is logical to treat such diseases not with drugs but with multiple micronutrient repletion, or pharmaco-nutrition’. This can take the form of pills and capsules — ‘nutraceuticals’, or food formats known as ‘functional foods’, This approach has been neglected hitherto because it is relatively unprofitable for drug companies — the products are hard to patent — and it is a strategy which does not sit easily with modem medical interventionism. Over the last 100 years, the drug industry has invested huge sums in developing a range of subtle and powerful drugs to treat the many diseases we are subject to. Medical training is couched in pharmaceutical terms and this approach has provided us with an exceptional range of therapeutic tools in the treatment of disease and in acute medical emergencies. However, the pharmaceutical model has also created an unhealthy dependency culture, in which relatively few of us accept responsibility for maintaining our own health. Instead, we have handed over this responsibility to health professionals who know very little about health maintenance, or disease prevention. 

One problem for supporters of this argument is lack of the right kind of hard evidence. We have a wealth of epidemiological data linking dietary factors to health profiles/ disease risks, and a great deal of information on mechanism: how food factors interact with our biochemistry. But almost all intervention studies with micronutrients, with the notable exception of the omega 3 fatty acids, have so far produced conflicting or negative results. In other words, our science appears to have no predictive value. Does this invalidate the science? Or are we simply asking the wrong questions? 

Based on pharmaceutical thinking, most intervention studies have attempted to measure the impact of a single micronutrient on the incidence of disease. The classical approach says that if you give a compound formula to test subjects and obtain positive results, you cannot know which ingredient is exerting the benefit, so you must test each ingredient individually. But in the field of nutrition, this does not work. Each intervention on its own will hardly make enough difference to be measured. The best therapeutic response must therefore combine micronutrients to normalise our internal physiology. So do we need to analyse each individual’s nutritional status and then tailor a formula specifically for him or her? While we do not have the resources to analyze millions of individual cases, there is no need to do so. The vast majority of people are consuming suboptimal amounts of most micronutrients, and most of the micronutrients concerned are very safe. Accordingly, a comprehensive and universal program of micronutrient support is probably the most cost-effective and safest way of improving the general health of the nation. 

Q. 101 The author recommends micronutrient-repletion for large-scale treatment of chronic degenerative diseases because 

A. it is relatively easy to manage. 

B. micronutrient deficiency is the cause of these diseases. 

C. it can overcome genetic risk factors. 

D. it can compensate for other lifestyle factors. 

Answer: B. 

Explanation: 

Refer to the starting of the fourth paragraph:”There is now considerable evidence that Type B malnutrition is a major cause of chronic degenerative diseases.” 

Q. 102 Tailoring micronutrient-based treatment plans to suit individual deficiency profiles is not necessary because 

A. it very likely to give inconsistent or negative results. 

B. it is a classic pharmaceutical approach not suited to micronutrients. 

C. most people are consuming suboptimal amounts of safe-to-consume micronutrients. 

D. it is not cost effective to do so. 

Answer: C. 

Explanation: 

Refer to the start of the fifth paragraph:”While we do not have the resources to analyze millions of individual cases, there is no need to do so. The vast majority of people are consuming suboptimal amounts of most micronutrients, and most of the micronutrients concerned are very safe.” 

Hence, the correct option is option ‘C’ 

Q. 103 Type-B malnutrition is a serious concern in developed countries because 

A. developing countries mainly suffer from Type-A malnutrition. 

B. it is a major contributor to illness and death. 

C. pharmaceutical companies are not producing drugs to treat this condition. 

D. national surveys on malnutrition do not include newer micronutrient groups. 

Answer: B. 

Explanation: 

Refer to the first line of the fourth paragraph:”There is now considerable evidence that Type B. malnutrition is a major cause of chronic degenerative diseases. If this is the case, then this logical to treat such diseases not with drugs but with multiple micronutrient repletion, or pharmaco-nutrition’.” 

Hence, the correct option is option ‘B’ 

Q. 104 Why are a large number of apparently healthy people deemed pre-ill? 

A. They may have chronic degenerative diseases. 

B. They do not know their own genetic risk factors which predispose them to diseases. 

C. They suffer from Type-B. malnutrition. 

D. There is a lengthy latency period associated with chronically degenerative diseases. 

Answer: A. 

Explanation: 

Refer to the first paragraph. Here the author says that a large number of people may have chronic degenerative diseases but due to their latency period, they are not detected easily. That eliminates options B and C Options A and D are close. But as the question asks why the people are considered as pre-ill, the answer should be because they may have chronically degenerative diseases they don’t know about. Hence, A. is the better option. 

 

Instructions 

For the following questions answer them individually 

Q. 105 Identify the incorrect sentence or sentences 

A. It was a tough situation and Manasi was taking pains to make it better. 

B. Slowly her efforts gave fruit and things started improving. 

C. Everyone complemented her for her good work. 

D. She was very happy and thanked everyone 

A.

B.

C. B and C 

D. A and C

Answer: C. 

Explanation: 

Sentence B. should be “Her efforts bore fruit…” 

The word in sentence C should be “complimented” instead of “complemented”. 

The correct answer choice is c). 

 

Q. 106 Identify the incorrect sentence or sentences 

A. Harish told Raj to plead guilty. 

B. Raj pleaded guilty of stealing money from the shop. 

C. The court found Raj guilty of all the crimes he was charged with. 

D. He was sentenced for three years in jail 

A. A and C 

B. B and D 

C. A, C, and D 

D. B, C, and D 

Answer: B. 

Explanation: 

Sentences B and D are incorrect. 

B should be “plead guilty to…” 

D should be “sentenced to three years…” 

 

Q. 107 Identify the incorrect sentence or sentences 

A. Last Sunday, Archana had nothing to do. 

B. After waking up, she lay on the bed thinking of what to do. 

C. At 11 o’clock she took shower and got ready. 

E. She spent most of the day shopping 

A. B and C 

B.

C. A and B 

D. B, C, and D 

Answer: A. 

Explanation: 

Sentences B and C are incorrect. 

Sentence B should be “…thinking what to do”. 

Sentence C should be “..took a shower…”. 

 

Q. 108 Fill in the Blanks: Many people suggest _______ and still other would like to convince people not to buy pirated cassettes. 

A. to bring down audiocassette prices to reduce the incidence of music piracy, others advocate strong legal action against the offenders, 

B. bringing down audiocassette prices to reduce the incidents of music piracy, others are advocating strong legal action against offenders, 

C. bringing down audiocassette prices to reduce the incidents of music piracy, others advocate strong legal action against offenders, 

D. audio cassette prices to be brought down to reduce incidents of music piracy, others advocate that strong legal action must be taken against offenders, 

Answer: C. 

Explanation: 

The structure of the sentence should be “suggest bringing down…”. This eliminates options a) and d). In the second part of the sentence, the structure should be “others advocate” and not “others are advocating”. So, option c) is the correct answer. 

 

Q. 109 Fill in the Blanks: The ancient Egyptians believed ______ so that when these objects were magically reanimated through the correct rituals, they would be able to functions effectively. 

A. that it was essential that things they portrayed must have every relevant feature shown as clearly as possible 

B. it was essential for things they portray to have had every relevant feature shown as clearly as possible, 

C. it was essential that the things they portrayed had every relevant feature shown as clearly as possible. 

D. that when they portrayed things, it should have every relevant feature shown as clearly as possible 

Answer: C. 

Explanation: 

Options a), b) and d) are grammatically incorrect. 

Option A – In the sentence, “that it was essential that things they portrayed” ,definite article (the) should be used in front of the noun(things). 

Option B- This sentence implies that having every relevant detail shown as clearly as possible is important for the things that Egyptians portray. However, having every detail shown was important for the effective functioning of the things Egyptians portray. Hence this is grammatically incorrect. 

Option D – Consider the phrase “that when they portrayed things, it should have”. The correct phrasing would be “that when they portrayed things, they should have” or “that when they portrayed a thing, it should have”. There is a singular plural mismatch occurring here. Hence this option is incorrect as well. 

The correct sentence is “The ancient Egyptians believed it was essential that the things they portrayed had every relevant feature shown as clearly as possible, so that when these objects were magically reanimated through the correct rituals, they would be able to function effectively.” 

 

Q. 110 Fill in the Blanks: Archaeologists believe that the pieces of red-ware pottery excavated recently near Bhavnagar and ______ shed light on a hitherto dark 600-year period in the Harappan history of Gujarat. 

A. estimated with a reasonable certainty as being about 3400 years old, 

B. are estimated reasonably certain to be about 3400 years old, 

C. estimated at about 3400 years old with reasonable certainty, 

D. estimated with reasonable certainty to be about 3400 years old, 

Answer: D. 

Explanation: 

Option b) is wrong because because of the use of the word “are”. Option c) is wrong because it implies that the pieces are “3400 years old with reasonable certainty”. Option a) is incorrect because of the use of the phrase “as being about”. Option d) is grammatically correct. 

 

Q. 111 Choose the option in which the usage of BOLT is INCORRECT or INAPPROPRIATE? 

A. The shopkeeper showed us a bolt of fine silk. 

B. As he could not move, he made a bolt for the gate. 

C. Could you please bolt the door? 

D. The thief was arrested before he could bolt from the scene of the crime. 

Answer: B. 

Explanation: 

In the second sentence, the correct usage would be “As he could not move, he couldn’t make a bolt for the gate.” 

 

Q. 112 Choose the option in which the usage of FALLOUT is INCORRECT or INAPPROPRIATE? 

A. Nagasaki suffered from the fallout of nuclear radiation. 

B. People believed that the political fallout of the scandal would be insignificant. 

C. Who can predict the environmental fallout of the WTO agreements? 

D. The headmaster could not understand the fallout of several of his good students at the public examination. 

Answer: D. 

Explanation: 

Fallout generally means adverse results that occur due to a situation. It’s usage is wrong in the last sentence, and the correct sentence would be “The headmaster could not understand the failure of several of his good students at the public examination.”. 

 

Q. 113 Choose the option in which the usage of PASSING is INCORRECT or INAPPROPRIATE? 

A. She did not have passing marks in mathematics. 

B. The mad woman was cursing everybody passing her on the road. 

C. At the birthday party all the children enjoyed a game of passing the parcel. 

D. A passing taxi was stopped to rush the accident victims to the hospital. 

Answer: A. 

Explanation: 

She did not have PASS marks is the correct usage in this sentence. 

 

Q. 114 Choose the most logical order of sentences from among the given choices to construct a coherent paragraph? 

A. The two neighbours never fought each other. 

B. Fights involving three male fiddler crabs have been recorded, but the status of the participants was unknown 

C. They pushed or grappled only with the intruder. 

D. We recorded 17 cases in which a resident that was fighting an intruder was joined by an immediate neighbour, an ally. 

E. We therefore tracked 268 intruder males until be saw them fighting a resident male. 

A. BEDAC 

B. DEBAC 

C. BDCAE 

D. BCEDA 

Answer:

Explanation: 

Statement B is the perfect starting sentence. Also sentences D,A and C are connected. Hence DAC is mandatory. There is only one option with the above requirement. Hence option A. 

 

Q. 115 Choose the most logical order of sentences from among the given choices to construct a coherent paragraph? 

A. In the west, Allied Forces had fought their way through southern Italy as far as Rome. 

B. In June 1944 Germany’s military position in World War two appeared hopeless 

C. In Britain, the task of amassing the men and materials for the liberation of northern Europe had been completed. 

D. Red Army was poised to drive the Nazis back through Poland. 

E. The situation on the eastern front was catastrophic. 

A. EDACB 

B. BEDAC 

C. BDECA 

D. CEDAB 

Answer: B. 

Explanation: 

Statement B seems most logical starting sentence, as it introduces to the topic on position of germany in WW2 Statement A or E should follow B Also sentences ED and AC are pair. Only option B fulfils these conditions. 

 

Q. 116 Choose the most logical order of sentences from among the given choices to construct a coherent paragraph? 

A. He felt justified in bypassing Congress altogether on a variety of moves. 

B. At times he was fighting the entire Congress. 

C. Bush felt he had a mission to restore power to the presidency. 

D. Bush was not fighting just the democrats. 

E. Representatives democracy is a messy business, and a CEO of the white House does not like a legislature of second guessers and time wasters. 

A. CAEDB 

B. DBAEC 

C. CEADB 

D. ECDBA 

Answer: D. 

Explanation: 

Among given sentences E seems most perfect starting sentences as it introduces the topic and other options take it forward. Also statement B. should follow D thus we have DB as the pair. Option D is the only option which fulfils these requirements. 

 

Q. 117 Choose the option which summarizes the passage the best. 

The human race is spread all over world, from the polar regions to the tropics. The people of whom it is made up eat different kinds of food, partly according to the climate in which they live, and partly according to the kind of food which their country produces. 

In hot climates; meat and fat are not much needed; but in the Arctic regions they seem to be very necessary for keeping up the heat of the body. Thus, in India, People live chiefly on different kinds of grains, eggs, milk, or sometimes fish and meat. In Europe people eat more meat and less gain. In the Arctic regions, where no grains and fruits are produced, the Eskimo and others races live almost entirely on meat and fish. 

A. Food eaten by people in different regions of the world depends on the climate and produce of the region, and varies from meat and fish in the Arctic to predominantly grains in the tropics. 

B. Hot climates require people to eat grains while cold regions require people to eat meat and fish. 

C. In hot countries people eat mainly grains while in the Arctic, they eat meat and fish because they cannot grow grains. 

D. While people in Arctic regions like meat and fish and those in hot regions like India prefer mainly grains, they have to change what they eat depending on the local climate and the local produce. 

Answer: A. 

Explanation: 

The main idea of the paragraph is the different type of foods which is primarily consumed depending upon the climate.Option A clearly summarises this perfectly. 

Option B is incorrect as the climate doesn’t compel the people to eat different food. 

Option C doesn’t indicate the the main idea of the paragraph. It only states the facts given in the paragraph. Option D is incorrect as it is not mentioned in the paragraph that the people are required to change their food depending upon their location. 

 

Q. 118 Choose the option which summarizes the passage the best. 

You seemed at first to take no notice of your school-fellows, or rather to set yourself against them because they were strangers to you. They knew as little of you as you did of them; this would have been the reason for their keeping aloof from you as well, which you would have felt as a hardship. Learn never to conceive a prejudice against others because you know nothing of them. It is bad reasoning, and makes enemies of half the world. Do not think ill of them till they behave ill to you; and then strive to avoid the faults, which you see in them. This will disarm their hostility sooner than pique or resentment or complaint. 

A. The discomfort you felt with your school fellows was because both sides knew little of each other. You should not complain unless you find others prejudiced against you and have attempted to carefully analyze the faults you have observed in them. 

B. The discomfort you felt with your school fellows was because both sides knew little of each other. Avoid prejudice and negative thoughts till you encounter bad behavior from others, and then win them over by shunning the faults you have observed. 

C. You encountered hardship amongst your school fellows because you did not know then well. You should learn not to make enemies because of your prejudices irrespective of their behaviour towards you. 

D. You encountered hardship amongst your school fellows because you did not know them well. You should learn to not make enemies because of your prejudices unless they behave badly with you. 

Answer: B. 

Explanation: 

The paragraph begins with the experience of a 2nd person and then gives the advice. 

Option A is incorrect because it doesn’t tell anything about complaining and analysing 

Option C is incomplete as it only talks about the experience. 

Option D is incorrect as it is written”you did not know them well”. The para talks about both sides not knowing each other. 

 

Q. 119 Choose the most logical order of sentences from among the given choices to construct a coherent paragraph? 

A. But this does not mean that death was the Egyptians only preoccupation. 

B. Even papyri come mainly from pyramid temples. 

C. Most of our traditional sources of information about the Old Kingdom are monuments of the rich like pyramids and tombs. 

D. Houses in which ordinary Egyptian lived have not been preserved, and when most people died they were buried in simple graves. 

E. We know infinitely more about the wealthy people of Egypt than we do about the ordinary people, as most monuments were made for the rich. 

A. CDBEA 

B. ECDAB 

C. EDCBA 

D. DECAB 

Answer: C. 

Explanation: 

Out of statement C, E and D, E perfectly opens the topic . Also statement B should come after statement C, as C talks about how pyramids and temples are sources of information and B takes it forward by saying that even papyrus came from pyramids. SO we have C pair. Hence, Option C . 

Q. 120 Choose the most logical order of sentences from among the given choices to construct a coherent paragraph? 

A. Experts such as Larry Burns, head of research at GM, reckon that only such a full hearted leap will allow the world to cope with the mass motorization that will one day come to China or India. 

B. But once hydrogen is being produced from biomass or extracted from underground coal or made from water, using nuclear or renewable electricity, the way will be open for a huge reduction in carbon emissions from the whole system. 

C. In theory, once all the bugs have been sorted out, fuel cells should deliver better total fuel economy than any existing engines. 

D. That is twice as good as the internal combustion engine, but only five percentage points better than a diesel hybrid. 

E. Allowing for the resources needed to extract hydrogen from hydrocarbon, oil coal or gas, the fuel cell has an efficiency of 30%. 

A. CEDBA 

B. CEBDA 

C. AEDBC 

D. ACEBD 

Answer: A. 

Explanation: 

Befor finding the opening statement, first see if there is any link between sentences. We can clearly see that statement D should follow E Thus link ED is compulsary pair. Also statement A should follow B , as statement B infers from A . Thus only logical sequence is CEDBA. 

Q. 121 Choose the option that best captures the essence of the text. 

Local communities have often come in conflict with agents trying to exploit resources, at a faster pace, for an expanding commercial-industrial economy. More often than not, such agents of resource intensification are given preferential treatment by the state, through the grant of generous long leases over mineral or fish stocks, for example, or the provision of raw material at an enormously subsidized price. With the injustice so compounded, local communities at the receiving end of this process have no recourse expect direct action, resisting both the state and outside exploiters through a variety of protest techniques. These struggles might perhaps be seen as a manifestation of a new kind of class conflict. 

A. A new kind of class conflict arises from preferential treatments given to agents of resource-intensification by the state, which the local community sees as unfair. 

B. The grant of long leases to agents of resource-intensification for an expanding commercial-industrial economy leads to direct protests from the local community, which sees it as unfair. 

C. Preferential treatment given by the state to agents of resource-intensification for an expanding commercial industrial economy exacerbates injustice to local communities and leads to direct protests from them, resulting in a new type of class conflict. 

D.Local communities have no option but to protest against agents of resource intensification and create a new type of class conflict when they are given raw material at subsidized prices for an expanding commercial-industrial economy. 

Answer: C. 

Explanation: 

The main idea of the paragraph is the reasons behind the generation of class conflict.Option C correctly summarises this para.Option A and B talks nothing about the class conflict. 

Option D is incorrect as it does not mention the role of state in generation of class conflict. 

Q. 122 Choose the option that best captures the essence of the text. 

Although almost all climate scientists agree that the Earth is gradually warming, they have long been of two minds about the process of rapid climate shifts within larger periods of change. Some have speculated that the process works like a giant oven or freezer, warming or cooling the whole planet at the same time. Others think that shifts occur on opposing schedules in the Northern and Southern Hemisphere, like exaggerated seasons. Recent research in Germany examining climate patterns in the Southern Hemisphere at the end of the last Ice Age strengthens the idea that warming and cooling occurs at alternate times in the two hemispheres. A. more definitive answer to this debate will allow scientists to better predict when and how quickly the next climate shift will happen. 

A. Scientists have been unsure whether rapid shifts in the Earth’s climate happen all at once or on opposing schedules in different hemispheres; research will help find a definitive answer and better predict climate shifts in future. 

B. Scientists have been unsure whether rapid shifts in the Earth’s climate happen all at once or on opposing schedules in different hemispheres; finding a definitive answer will help them better predict climate shifts in future. 

C. Research in Germany will help scientists find a definitive answer about warming and cooling of the Earth and predict climate shifts in the future in a better manner. 

D. More research rather than debates on warming or cooling of the Earth and exaggerated seasons in its hemisphere will help scientists in Germany predict changes better in future. 

Answer: B. 

Explanation: 

No where is in mentioned in the paragraph that “a definitive answer” can be found. So, statement a) which says “research will help find a definitive answer”, option c) which says “Research will help scientists find a definitive answer” are ruled out. Option d) is misleading because the para doesn’t talk about more research than debates helping scientists. So, this option can also be ruled out. Option b) is the best summary. 

Q. 123 Choose the option that best captures the essence of the text. 

Modern bourgeois society, said Nietzsche, was decadent and enfeebled – a victim of the excessive development of the rational faculties at the expense of will and instinct. 

Against the liberal-rationalist stress on the intellect, Nietzsche urged recognition of the dark mysterious world of instinctual desires . the true forces of life. Smother the will excessive intellectualizing and you destroy the spontaneity that sparks cultural creativity and ignites a zest for living. The critical and theoretical outlook destroyed the creative instincts. For man’s manifold potential to be realized, he must forego relying on the intellect and nurture again the instinctual roots of human existence. 

A. Nietzsche urges the decadent and enfeebled modern society to forego intellect and give importance to creative instincts. 

B. Nietzsche urges the decadent and enfeebled modern society to smother the will with excessive intellectualizing and ignite a zest for living. 

C. Nietzsche criticizes the intellectuals for enfeebling the modern bourgeois society by not nurturing man’s creative instincts. 

D. Nietzsche blames excessive intellectualization for the decline of modern society and suggests nurturing creative instincts instead. 

Answer: A. 

Explanation: 

Option B,C and D doesn’t capture the essence as well the idea what the passage is about. OPtion A is clearly in line with the essence and also follows the last sentence of the passage. Hence option A 

×

Hello!

Click one of our representatives below to chat on WhatsApp or send us an email to info@vidhyarthidarpan.com

×